Download as pdf or txt
Download as pdf or txt
You are on page 1of 71

.

selfstudy Guide

SSC
CONSTABL�

ELEffiENTA�Y
ffiATHEffiATICS
Number System 3

CHAPTER 1

Number System
In the decimal number system, numbers are expressed by means of symbols 0, 1, 2, 3, 4, 5, 6, 7, 8, 9 called digits. Here, 0 is
called an insignificant digit whereas 1, 2, 3, 4, 5, 6, 7, 8, 9 are called significant digits. We can express a number in two ways
Notation Representing a number in figures is known as notation as 350.
Numeration Representing a number in words is known as numeration as ‘five hundred and forty five’.

Indian System of Numeration


Ten Thousand
Ten Arb Arb Ten Crore Crore Ten Lakh Lakh Thousand Place Hundred Place Ten Place Unit Place
Place
1010 10 9 10 8 10 7 10 6 105 10 4 10 3 10 2 101 10 0

Face Value and Place Value of a Digit Irrational Numbers Non-recurring and non- terminating
Face Value It is the value of the digit itself e.g., in 3452, decimals are irrational numbers.
face value of 4 is ‘four’, face value of 2 is ‘two’. p
These numbers cannot be expressed in the form of .
Place Value It is the face value of the digit multiplied by q
the place value at which it is situated e.g., in 2586, place e.g., 3, 5, 29, ............ .
value of 5 is 5 × 10 2 = 500. Real Numbers Real number includes both rational and
irrational numbers.
Categories of Numbers
Natural Numbers (N ) Numbers that are used in counting Different Types of Numbers
are called natural numbers.
If N is the set of natural numbers, then we write N = {1, 2, Even bers Numbers which are exactly divisible by 2
3, 4, 5, 6, ......... } are called even numbers.
The smallest natural number is 1. e.g., 2, 4, 6, ..... .
Whole Numbers (W ) When zero is included in the set of Odd Numbers Numbers which are not exactly divisible by
natural numbers, the numbers are known as whole 2 are called odd numbers.
numbers. If W is the set of whole numbers, then we write
e.g., 1, 3, 5, ..... .
W = {0,1,2, 3, 4, 5, .......... }
The smallest whole number is 0. Prime Numbers Prime numbers are divisible by one and
‘itself’ only. e.g. 2, 3, 5, 7, 11, ...... .
Integers (I ) Integers are whole numbers and negative of
whole numbers including zero. Relative Prime Numbers Two numbers are said to be
If I is the set of integers, then we write relatively prime, if they do not have any common factor
I = { ........... − 3, − 2, − 1, 0, 1, 2, 3, .......... } other than 1.
Rational Numbers Numbers which can be expressed in e.g., (3, 5), (4, 7), (11, 15), (15, 4), ......
p
the form of , where p and q are both integers and q ≠ 0 are Twin Primes Two prime numbers which differ by 2 are
q 3 −7 called twin primes.
called rational numbers. e.g., , , 5, − 2, ........... .
2 9
e.g., (3, 5), (5, 7), (11, 13), .... .
There exists infinite number of rational numbers between
any two rational numbers.
4 Self Study Guide SSC Constable (GD) REcruitment Exam

Most Important Points To Find the Unit’s Place Digit of A Given Exponential
In case of 0,1, 5, 6 The unit’s place digit is 0, 1, 5, 6 respectively.
■ 2 is the only even prime number. In case of 4 and 9
■ 1 is not a prime number because it has two equal factors. (a) if power is odd The unit’s place digit is 4 and 9 respectively.
■ Every prime number greater than 3 can be written in the (b) if power is even The unit’s place digit is 6 and 1 respectively.
form of (6k + 1) or (6k − 1) where k is an integer. In case of 2, 3, 7, 8 See the following example
■ There are 15 prime numbers between 1 and 50 and 10 To find the unit’s place digit of ( 134647 )553
prime numbers between 50 and 100. n Step I 553 ÷ 4 gives 1 as remainder, this
Composite Numbers Numbers which are not prime are remainder is taken as new power.
called composite numbers. e.g., 4, 6, 9, 15, ...... . n Step II (134647)553 ≡ (134647)1 ≡ 71 = 7
Note 1 is neither prime nor composite. ∴ The unit’s place digit is 7.
If on dividing the remainder obtained is zero,
Perfect Numbers A number is said to be a perfect take 4 as new power instead of zero.
number, if the sum of all its factors is equal to the number For example ( 134647 )552
itself.
e.g., Factors of 6 are 1, 2, 3 and 6. ( 134647 )552 ≡ ( 134647 ) 0 ≡ (7 0 ) ≡ (7 ) 4 = 2401

Sum of factors excluding 6 = 1 + 2 + 3 = 6 ∴The unit’s place digit is 1.

∴ 6 is a perfect number.
Most Important Formulae
Other examples of perfect numbers are 28, 496, 8128 etc.
n (n + 1)
■ Sum of first n natural numbers =
Divisibility Tests 2
■ Sum of first n even natural numbers = n (n + 1)
By 2 When the unit’s place digit is even or zero.
Sum of first even numbers upto n =  + 1

n n
By 3 When the sum of the digits is divisible by 3. 22 
■ Sum of first n odd natural numbers = n 2
By 4 When the number formed by the last two digits is divisible by 4. n (n + 1)(2n + 1)
■ Sum of squares of first n natural numbers =
By 5 When the unit’s place digit is 0 or 5. 6
2
n (n + 1) 
By 6 When the number is divisible by both 2 and 3. ■ Sum of cubes of first n natural numbers =  
 2 
By 7 See the following example. To check if 2961 is divisible by 7.
■ If the denominator of a rational number has no prime factors
n Step I 296 1 : 296 − 1 × 2 = 294 other than 2 or 5, then and only then it is expressible as a
n Step II 29 4 : 29 − 4 × 2 = 21 terminating decimal. The number of ways in which a number N
Since 21 is divisible by 7, hence the number. can be expressed as product of two factors which are relatively
prime to each other is 2m −1, where m is the number of different
If a number is formed by repeating a digit six times, the prime factors of N.
number is divisible by 7, 11 and 13, e.g., 666666. ■ The difference of squares of two consecutive numbers is an
If a number is formed by repeating a two-digit number three odd number and that odd number is the sum of these two
times, the number is divisible by 7. e.g., 676767. consecutive numbers.
If a number is formed by repeating a three-digit number two ■ The number of prime factors of a p ⋅ b q ⋅ c r ⋅ d s is p + q + r + s .
times, the number is divisible by 7, 11 and 13. e.g., 453453.
Ex. 1 Which one of the following number is divisible
By 8 When the number formed by the last three digits is divisible by 8. by 11?
By 9 When the sum of the digits of the number is divisible by 9. (a) 5697231 (b) 4832718 (c) 42922 (d) 239560
By 10 Solution (b) If the difference of the sum of the digits at odd
When the unit’s place digit is 0.
places and the sum of the digits at even places is either 0 or
By 11 Add the digits at even places and that at odd places and if the divisible by 11, the number is divisible by 11. Applying it in
difference of these two sums is zero or a multiple of 11, the all the numbers given, we find that in number 4832718.
number is divisible by 11. Sum of digits in odd places = 8 + 7 + 3 + 4 = 22
By 12 When the number is divisible by both 3 and 4. Sum of digits in even places = 1 + 2 + 8 = 11
By 13 See the following example. To check if 265213 is divisible by 13. Difference of sums = 22 − 11 = 11, which is divisible by 11.
n Step I 265 213 : 265 − 213 = 52 Ex. 2 Which one of the following prime numbers
while dividing 2176 leaves 9 as remainder?
Since 52 is divisible by 13, hence the number.
(a) 2176 (b) 1395 (c) 5739 (d) 7853
By 25 When the number formed by last two digits is divisible by 25. Solution (a) Clearly, when 2176 is divided by 197, the
By 27 When the sum of the digits of the number is divisible by 27. remainder is 9.
Number System 5

Practice Exercise
Face value and place value 11. What is the remainder when 75 3 is
and Types of Number divided by 4? (a) 12000 (b) 12100
(a) 1 (b) 4 (c) 122000 (d) 2400
1. Find the place value of 2 in
(c) 3 (d) 2 21. If 1 + 10 + 102 + upto n terms
832567.
(a) 200 (b) 2 12. What is the remainder when 10n − 1
(c)2000 (d) 20000 412 × 363 × 312 is divided by 15? = , then the sum of the
9
2. Which one is a rational number? (a) 2 (b) 6
series
(c) 4 (d) 3
(a) 4 (b) 3 4 + 44 + 444 + ..... upto n terms is
(c) 2 + 3 (d) 5 3 13. Find the largest four digit number 4 4n
(a) (10 n − 1) −
3. Which one is an irrational number? exactly divisible by 55. 9 9
22 (a) 9900 (b) 9955 4 4n
(a) (b) π (b) (10 n − 1) −
7 (c) 9945 (d) 9960 81 9
(c) 1.333 (d) 3.14 14. By what least number 250 must be 40 4n
(c) (10 n − 1) −
multiplied to get a multiple of 15. 81 9
4. The smallest odd prime number is
40 4n
(a) 0 (b)1 (a) 3 (b) 5 (d) (10 − 1) −
n

(c) 2 (d) 15 9 9
(c) 2 (d) 3
5. Which of the following is a prime 15. What is the sum of all numbers Previous Years’ Questions
number? between 250 and 550, which are
22. The sum of a natural number and
(a) 137 (b) 122 divisible by 19.
(a) 6635 (b) 6640
its square equals the product of the
(c) 144 (d) 125
(c) 6000 (d) 6695 first three prime numbers. The
6. Which of the following is not a number is [SSC Constable, 2012]
prime number? 16. In 337337, the unit digit is occupied (a) 2 (b) 3
(a) 17 (b) 19 by (c) 5 (d) 6
(c) 27 (d) 29 (a) 1 (b) 3
(c) 7 (d) 9 23. The sum of all those prime
7. Number of prime numbers between numbers which are not greater
0 and 100, is
Sum of consecutive than 17 is [SSC Constable, 2012]
(a) 31 (b) 29
(c) 25 (d) 23 numbers (a) 59 (b) 58
(c) 41 (d) 42
17. The sum of all even number from
Divisibility Test 1 to 60 is 24. 1 + 2 + 3 + K + 100 = x, then x is
(a) 870 (b) 960 [SSC Constable, 2011]
8. If 43p is divisible by 3, then what is
(c) 840 (d) 930 (a) 5050
the largest value p can take?
(b) 5000
(a) 8 (b) 2 18. The sum of all odd numbers from (c) 10100
(c) 5 (d) 0 1 to 32 is (d) 10000
9. If the number 6pq5 is divisible by (a) 256 (b) 128
both 3 and 5, which of the (c) 25 (d) 16 Answers
following digits can replace p and q? 19. The sum of natural numbers from 1 c 2 a 3 b 4 d 5 a
(a) 5, 9 (b) 9, 5 75 to 99, is 6 c 7 c 8 a 9 d 10 a
(c) 5, 7 (d) 9, 7 (a) 1598 (b) 1798 11 c 12 b 13 b 14 a 15 c
10. What is the remainder when 152 (c) 1958 (d) 2175
16 c 17 d 18 a 19 d 20 b
× 698 is divided by 5. 20. If 13 + 23 + .... + 103 = 3025, then 21 c 22 c 23 b 24 a
(a) 1 (b) 2
(c) 4 (d) 3
4 + 32 + 108 + ....+ 4000 =
6 Self Study Guide SSC Constable (GD) REcruitment Exam

Hints & Solutions


1. (c) Place value of 2 in 832567 = 2 × 1000 13. (b) The largest four digit number = 9999 18. (a)∴There are total 16 odd numbers
= 2000 55)9999(181 between 1 and 32.
2. (a) 4 = ± 2
55 ∴Required sum =(16)2 = 256
449
∴ It is clear that in given numbers, 4 is a 440 19. (d) 75 + 76 + ...+ 99
rational number. 99 = (1 + 2 + ....+99) − (1 + 2 + 3+ ...+74)
55 99( 99 + 1) 74 × (74 + 1)
3. (b) In given options, π is an irrational 44 = −
number. 2 2
The required number = 9999 − 44 = 9955 = 99 × 50 − 37 × 75
4. (d) The smallest prime number is 2 which is 14. (a) 15 = 3 × 5 = 4950 − 2775 = 2175
even but smallest odd prime number is 3.
∴Any multiple of 15 must be divisible by 3 20. (b) 4 + 32 + 108 + ....+ 4000
5. (a) In given numbers, 137 is a prime and 5. = 4 (1 + 8 + 27 + ....+1000)
number. = 4(13 + 2 3 + 3 3 + ....+10 3 )
250 = 2 × 5 × 5 × 5
6. (c) In given numbers, 27 is not prime. ∴ 250 is divisible by 5, but not by 3. = 4 × 3025 [Q 13 + 2 3 + ...+10 3 = 3025]
7. (c) The prime numbers between 0 and 100 So, 250 must be multiplied by 3 to make it a = 12100
are 25 in number these are 2, 3, 5, 7, 11, 13, multiple of 15. 21. (c) Expression = 4 + 44 + 444 + ... to n
17, 19, 23, 29, 31, 37, 41, 43, 47, 53, 59, 61, 15. (c) Since, 250 = 13 × 19 + 3 terms = 4 (1 + 11 + 111 + .... to n terms)
67, 71, 73, 79, 83, 89, and 97.
Terms divisible by 19 upto 250 = 13 Multiplying and dividing by 9.
8. (a) A number is divisible by 3 when sum of And 550 = 28 × 19 + 18 4
= ( 9 + 99 + 999 + …to n terms )
the digits is divisible by 3. ∴Term divisible by 19 upto 550 = 28 9
4+ 3+ p= 7 + p ∴Total terms divisible by 19 between 250 and
4
⇒ [(10 − 1) + (10 2 − 1) + (10 3 − 1) + .... n
So, p can be 2, 5, 8. So the largest value is 8. 9
550 = 28 − 13 = 15 4 terms ]
= (10 + 10 2 + 10 3 + ... n terms)
9. (d) If p is 9 and q is 7. The number is 6975. Midterm (Average) between 250 to 550 = 400 9
6975 is divisible by both 3 and 5. ∴Sum of 15 terms = 400 × 15 = 6000 4
− (1 + 1 + 1 + ... n terms)
10. (a) The remainder when 152 is divided by 16. (c) The given number is 337 337. The unit’s 9
4 4n
5=2
place digit is 7, so by applying the method we = (10 + 10 + 10 3 + ..... n terms) −
2

divide the power by 4, i.e., 9 9


The remainder when 698 is divided by 5 = 3 4 n−1 4
4)337(84 = × 10 [1 + 10 + 10 + ... + 10
2
]− n
Their product = 2 × 3 = 6 32 9 9
The remainder when 6 is divided by 5 = 1 17 40  10 n − 1 4 40 n 4
= − n= (10 − 1) − n
11. (c) The remainder when 75 is divided by 4 16 9  10 − 1  9 81 9
is 3. 1
The remainder is 1, so the new power is 1. 22. (c) From option (c),
∴The remainder when 75 × 75 × 75 is divided 5 + ( 5)2 = 5 + 25 = 30
by 4 is 3. ∴ (337 )337 ≡ (337 )1
Q 30 = 2 × 3 × 5
So, the required remainder is 3. Hence, the unit’s place digit is 7.
Hence, the number is 5.
12. (b) 41, 36 and 31 when divided by 15, 17. (d) We know that sum of all even natural
n n 23. (b) Required sum of all prime numbers
leave remainders 11, 6 and 1 respectively. numbers upto n = ( + 1) (which are not greater than 17)
∴412 × 36 3 × 312 when divided by 15, leaves 2 2
= 2 + 3 + 5 + 7 + 11 + 13 + 17 = 58
Here, n = 60
remainder n( n + 1)
60  60 24. (a) 1 + 2 + 3 + 4 + K + n =
112 × 6 3 × 12 = 26136. ∴ Required sum= + 1 2
As remainder cannot exceed the divisor. 2  2 
100 (100 + 1)
= 30 × 31 = 930 ⇒ = x
∴ Remainder is 6 i.e., remainder obtained on 2
dividing 26136 by 15. ⇒ x = 50 × 101 = 5050
Fractoion 7

CHAPTER 2

Fractions
Fraction Compound Fraction
x It is defined as a fraction of a fraction.
A number in the form of , where y ≠ 0, is known as
y 3 11 33
e.g., of = is a compound fraction.
fraction. 5 13 65

Here, x and y are respectively known as numerator and Complex Fraction


denominator of fraction. In it, numerator or denominator itself or both are fractions.
Again, a fraction is a rational number too. 5 1 1
+
Numerator N 7 , 2 5 etc.
Thus, Fraction = = e.g.,
3 4 7
Denominator D +
2 5 8

Important Points & Formulae Continued Fraction


A fraction which contains an addition or substraction
■ If N = D, then fraction = 1 based fraction either in numerator or in denominator or in
■ If N = 0, D ≠ 0, then fraction = 0 both is termed as continued fraction.
■ If D = 0, N take any value , then fraction = ∞ 1
When N and D are multiplied or divided by the same
e.g., is a continued fraction.
■ 1
number, then value of fraction remains unchanged. 2+
2
3+
If N and D have no common factor, then the fraction is said to 2
1+

be in its lowest terms. 5


x x×b x/ y x x/ y x×b Note To simplify a continued fraction start from bottom and
■ = ■ = ■ =
a/b a a y×a a/b y×a work upwards.

Types of Fraction Important Points


Proper Fraction ■ For addition or subtraction of fractions, if
When numerator is less than denominator, then fraction is ● denominators are same, then we write denominator once
called a proper fraction. and add or subtract the numerators.
1 15 21 ● denominators are different, then find a common
e.g., , , etc. denominator (by taking LCM of them) and then apply
2 17 47
above method.
Improper Fraction ■ To multiply fractions, the numerators are multiplied together
When numerator is greater than denominator, then fraction and denominators are multiplied together.
is called a improper fraction.

e.g.,
17 47 3
, , etc.
Decimal Fractions
15 21 2 The fractions, in which denominators has the powers of 10
Mixed Fraction are called decimal fractions. e.g., 10th part of unit = 1 = 0.1
Those fraction which are formed by using integer and 10
proper fraction are known as mixed fractions. For converting a decimal fraction into a simple fraction,
1 1 place 1 in the denominator under the decimal point. Then,
e.g., 3 , 7 etc. after removing the decimal point, place as many zeroes
4 2
after it as the number of digits after the decimal point.
Now, reduce the fraction in its smaller form.
Note Every mixed fractions can be expressed as improper
1 15 576 144
fractions and vice-versa. e.g., 7 = e.g., 5.76 = =
2 2 100 25
8 Self Study Guide SSC Constable (GD) REcruitment Exam

Addition and Subtraction of Decimal Fractions Recurring Decimals


For addition or subtraction of decimal fractions, The decimal fractions, in which one or more decimal digits are
write the decimal fractions in such a way that all the repeated again and again, are called recurring decimal fractions.
decimal points are in the align form, then these 2
e.g., = 0.6666 = 0.6 . Here, a line is drawn on the digits which
numbers can add or subtract in simple manner. 3
are repeated.
Ex. 1 353.5 + 2.32 + 43.23 is equal to
(a) 400.00 (b) 398.5 Pure and Mixed Recurring Decimal
(c) 399.05 (d) None of these In a recurring decimal, if all the figures are repeated after the
Solution (c) 353.50 decimal point, then it is called a pure recurring decimal and if
2.32
some figures are not repeated, while some of them are repeated,
+ 43.23
then it is called a mixed recurring decimal.
= 399.05
To Convert a Pure Recurring Decimal into
Multiplication of Two or More Decimal Fractions a Simple Fraction
For multiplication of two or more decimal fractions, Write the repeated figures only once in the numerator and place
multiply, then without considering the decimal as many nines in the denominator as the number of figures
points and then in the product, decimal point is 4
maked as many places of decimal as the sum of the repeating. e.g., 0.4 = 0.4 =
9
numbers of decimal places in the given numbers.
To Convert a Mixed Recurring Decimal into
Ex. 2 4.3 × 0.13 is equal to a Simple Fraction
(a) 0.599 (b) 0.622 Write the numerator take the difference between the numbers
(c) 5.99 (d) None of these
formed by all the digits after decimal point (repeated digits will
Solution (a) Q 43 × 13 = 599
be taken only once) and the number formed by non-repeating
Sum of the decimal places = (1 + 2) = 3
digits.
∴ Required product = 4.3 × 0.13 = 0.599
In the denominator place as many nines as there are repeating
Dividing a Decimal Fraction by an Integer digits and after nine put as many zeroes as the number of
If we divide a decimal number by an integer, then at non-repeating digits.
first divide the number ignoring the decimal point
( 36 − 3) 33 11
and at last put the decimal after the number of digits e.g., 0.36 = = =
(from right) according at in the given number. 90 90 30

Ex. 3 Divide 0.81 by 9 is equal to


(a) 0.9 (b) 0.09 Comparison of Fraction
(c) 9 (d) None of these l In a series of two or more fractions, if the difference of
81 numerator (N) and denominator (D) is same, then the fraction
Solution (b) Q =9
9 which
0.81 ¡ when for all fractions D > N, that fraction will be highest,
⇒∴ = 0.09 (Two places of decimal)
9 whose D will be greater and that fraction will be smallest
whose D will be smaller.
Note If we divide the decimal number by multiple of 10,
¡ when for all fractions D < N, that fraction will be highest
then we move the decimal point to the left by the respective
423 whose D will be smaller and fraction will be smallest whose
number of zeroes. e.g., = 423
. D will be greater.
100
l In a series of two or more fractions, if denominator (D) of all
Division of Decimal Fractions
fractions are same, then
If divisor and dividend both are decimal numbers, ¡ that fraction will be smallest whose N will be smaller.
then first convert them into the simple fraction by
¡ that fraction will be highest whose N will be greater.
putting number of zeroes in the denominators of
both. Now, divide them. l In a series of two or more fractions, if numerators (N) of all
fractions are same, then
Ex. 4 Divide 0.42 by 0.007 is equal to ¡ that fraction will be highest whose D will be smaller.

(a) 600 (b) 60 ¡ that fraction will be smallest whose D will be greater.
(c) 6000 (d) None of these a c
0.42 0.42 420 l Let and are two fractions. Then,
Solution (b) = × 100 = = 60 b d
0.007 7 7 a c a c
¡ if ad > bc, then > . ¡ if ad < bc, then < .
b d b d
Fraction 9

4 5 6 3 2 4 5 6 3
Ex. 5 Out of the fractions, , , , and , which Solution (c) = 0.57, = 0.38, = 0.54, = 0.6,
7 13 11 5 3 7 13 11 5
2
is the second smallest fraction? = 0.67
4 5 6 3 3
(a) (b) (c) (d) 6
7 13 11 5 Clearly, the second smallest fraction is .
11

Practice Exercise
10 12 5 1 3
1. Ascending order of , , and 6. Find the value of 1 + 12. of total students in a class are
13 17 6 1 5
1+
11 1 2
is 1+ girls and remaining are boys. If
21 1 3
10 12 5 11 11 12 10 5
1+ 1
2
(a) , , ,
13 17 6 21
(b) , , ,
21 17 13 6
1+ of girls and of boys are absent,
3 4
5 10 11 12 5 10 12 11 34 34 then which part of number of total
(c) , , , (d) , , , (a) (b)
6 13 21 17 6 13 17 21 21 22 students are present?
2. Which of the following fractions is 33 32 23 18
(c) (d) (a) (b)
3 5 21 9 30 49
greater than , but less than ? 23
4 6 1 (c) (d) None of these
1 2 4 9 7. A person spends part of his 36
(a) (b) (c) (d) 4
2 3 5 10 2 1
income on food, part on house 13. If th part of a number is 100
11 3 7
3. Sum of three fractions is 2 . If 1
24 rent and remaining ` 630 on other more than th part of a number,
the greatest fraction is divided by items. Find his house rent. 11
7 (a) ` 2050 (b) ` 5040 then the number is
the smallest fraction, the result is (a) 770 (b) 1925
6 (c) ` 1000 (d) None of these
which is greater than the middle (c) 1825 (d) 1200
8. If 0. 3 + 0. 6 + 0. 7 + 0. 8 = x, then x
1 14. A number is interchanging, if on
fraction by . Find the fraction. is equal to
3 3 2 3 adding the sum of the digits of the
4 4 7 7 (a) 2 (b) 2 (c) 2 .35 (d) 5
(a) (b) (c) (d) 10 3 10 number and the product of the
7 5 8 9 digits of the number. The result is
 1 1 1 1
4. If a fraction is multiplied by itself 9. 1 + 11 + 111 + 1111  is equal to the number. What fraction
 2 2 2 2
and then divided by the reciprocal of number between 10 and 100
of the same fraction, the result is equal to (both 10 and 100 included) is
26 1
(a) 1236 (b) 1234 interesting?
18 . Find the fraction. 2
27 (a) 0.1 (b) 0.11
(c) 618 (d) 617
8 1 (c) 0.16 (d) None of these
(a) (b) 1
27 3 3
10. A drum of kerosene is full. When
(c) 2
2
(d) 3
2 4 Previous Year‘s Questions
3 3 30 L of kerosene is drawn from it. 15. The decimal fraction 2.349 is equal
7 to
5. Find the sum of the largest and the It remains full. The capacity of [SSC Constable, 2012]
12 (a) 2326/999 (b) 2326/990
smallest fraction in 3 ,11 , 6 , 7 and the drum is (c) 2347/999 (d) 2347/990
7 13 11 8 (a) 120 L (b) 135 L (c) 150 L (d) 180 L
5
. 1 1 1 1 1 Answers
9 11. + + + + is equal to
75 73 71 75 3 15 35 63 99 1 b 2 c 3 c 4 c 5 b
(a) (b) (c) (d) 6 a 7 b 8 b 9 a 10 d
56 56 34 48 10 5 9 7
(a) (b) (c) (d) 11 b 12 d 13 b 14 d 15 b
11 11 11 11
10 Self Study Guide SSC Constable (GD) REcruitment Exam

Hints & Solutions


1. (b) Here,
10
= 0769
. ,
12
= 0705
. , So,
7 3
is largest fraction and is smallest 10. (d) Let the capacity of the drum = x L
13 17 8 7 3 7
5 11 fraction. ∴ x− x = 30
= 0.833 and = 0.523 4 12
6 21 Now, required sum 9x − 7 x
∴ Ascending order is 7 3 7 ×7 3× 8 ⇒ = 30
= + = + 12
0.523 < 0705
. < 0769
. < 0.833 8 7 8×7 7 × 8 30 × 12
11 12 10 5 49 24 73 ⇒ x= = 180 L
i.e., < < < = + = 2
21 17 13 6 56 56 56 1 1 1 1 1
3 5 1 11. (b) + + + +
2. (c) Here, = 075 . and = 0.833 6. (a) 1 + 3 15 35 63 99
4 6 1
1+
= +
1 1 1 1 1
1 + + +
Now, from options, 1 = 0.5, 2 = 0.666, 4 = 0.8 1+  3 15  35 63 99
1
2 3 5 1+
=
6 1 1 1
and
9
= 0.9 1+
2 + + +
 15 35  63 99
10 3
∴075. < 0.8 < 0.833 1 45 1 1 3 1 1
= 1+ = + + = + +
3 4 5 1+
1 105 63 99  7 63  99
i.e., < <
4 5 6 1+
1 28 1 4 1 44 + 1 45 5
= + = + = = =
3
3. (c) Let the fractions be x, y and z 1+ 63 99 9 99 99 99 11
5 12. (d) Let the total number of students = x
respectively in decreasing order. Then,
1
according to the question, = 1+ 3x
1 ∴ Number of girls =
11 59 1+ 5
x+ y+ z = 2 = ...(i) 5
24 24 1+ 3x 2 x
8 and number of boys = x − =
x 7 7 5 5
= ⇒ x= z ...(ii) 1
z 6 6 = 1+ 3 2 x 3x
8 Now, number of boys present = × =
7 1 7 −2 5 1+ 4 5 10
and y= − = = ...(iii) 13
6 3 6 6 1 3x x
13 34 and number of present girls = × =
On putting the values from Eqs.(ii) and (iii) in = 1+ = 3 5 5
21 21
Eq. (i), we get 3x x
7. (b) Let the income of person = ` x. Number of present students = +
7 5 59 10 5
z+ + z = x
6 6 24 Then, expenditure on food = ` 3x + 2 x 5x x
= = =
7 z + 6 z 59 5 59 − 20 4
10 10 2
⇒ = − = 2x
6 24 6 24 Expenditure on house rent = ` Hence, 1/2 part of total students were present.
13 z 39 39 6 3 3
⇒ = ⇒ z= × = 13. (b) Let the number = x
Remaining = x −  +  x 2 x x
6 24 24 13 4  =` x x 11x − 7 x
7 3 7 4 3 12 ∴ − = 100 ⇒ = 100
∴ From Eq. (ii), x = × = 7 11 77
6 4 8 According to the question,
100 × 77
x ⇒ x= = 1925
4. (c) Let the required fraction be x. According = 630 4
to the question, 12
x× x 26 ⇒ x = 630 × 12 14. (d) Let the number be expressed as
= 18 10a + b, where a ≠ 0, then
1 27 = ` 7560
2 10 a + b = ( a + b ) + ( a × b )
x Hence, house rent = × 7560 = ` 5040
3 3 ⇒ 9a−ab=0
512  8 
⇒ x3 = =  ⇒ a( 9 − b ) = 0
27  3 8. (b) 0.3 + 0.6 + 0.7 + 0.8 = x
3 6 7 8 ∴ b = 9 (Q a ≠ 0 )

8
x= =2
2 x= + + + Therefore, the possible numbers are
3 3 9 9 9 9
24 8 2 19, 29, 39, ..., 99. Thus, there are total 9 number
5. (b) Given fractions are x= = =2 out of 91 numbers.
9 3 3
3 11 6 ∴Required fraction = 9 / 91 = 0.0989
= 0.428, = 0.846, = 0.545, 1 1 1 1
7 13 11 9. (a) 1 + 11 + 111 + 1111 349 − 3
7 5
2 2 2 2 15. (b) 2.349 = 2 +
= 0.875 and = 0.555 990
= 1 + 11 + 111 + 1111 +  + + + 
1 1 1 1
8 9 1980 + 346 2326
2 2 2 2 = =
In decimal, 0.875 is largest and 0.428 is 990 990
smallest . = 1234 + 2 = 1236
HCF and LCM 11

CHAPTER 3

HCF and LCM


HCF of any given set of number is the greatest factor Ex. 1 The LCM of two numbers is 2079 and their HCF
common to them and LCM of two or more numbers is the is 27. If the first number is 189, find the second number.
smallest number which is common multiple of the given (a) 324 (b) 297
number. (c) 252 (d) 360
Solution (b) HCF × LCM = One number × Second number
Factor and Multiple ∴ 27 × 2079 = 189 × Second number
27 × 2079
When two or more numbers are multiplied together to give ⇒ Second number = = 297
189
a product, each is called a factor of the product.
The product of two or more numbers is said to be a 25 15
Ex. 2 Find the LCM and HCF of and .
multiple of each of those numbers. e.g., 4 × 5 = 20 6 4
73 5 75 7
Here, 4 and 5 are factors of 20 and 20 is a multiple of both 4 (a) , (b) ,
and 5. 2 2 2 12
75 5 77 5
(c) , (d) ,
HCF (Highest Common Factor) 2 12 2 12
25 15 LCM of (25,15) 75
HCF of two or more numbers is the greatest number which Solution (c) LCM of and = =
6 4 HCF of (6,4) 2
divides each of them exactly.
25 15 HCF of (25,15) 5
e.g., 6 is the HCF of 12 and 18 as there is no number greater HCF of and = =
6 4 LCM of (6,4) 12
than 6 that divides both 12 and 18. HCF is also known as
GCD and GCM (Greatest Common Divisor and Greatest Ex. 3 Find the LCM of 0.6, 9.6 and 0.12.
Common Measure). (a) 9.60 (b) 9.70
(c) 8.9 (d) 10.12
LCM (Least Common Multiple) Solution (a) The given numbers are equivalent to 0.60, 9.60
The LCM of two or more given numbers is the least and 0.12 Now, we have to find the LCM of 60, 960 and 12.
number to be exactly divisible by each of them. LCM of 60, 960 and 12 = 960
e.g., we can obtain LCM of 4 and 12 as follows ∴ The required LCM = 9.60
Multiple of 4 = 4, 8, 12, 16, ....... Ex. 4 Find the largest number that will divide 260,
Multiple of 12 = 12, 24, 36, 48, ....... 720 and 145 leaving 7 as remainder in each case.
Common multiple of 4 and 12 = 12, 24, 36, .... (a) 25 (b) 27
(c) 29 (d) 23
∴ LCM of 4 and 12 = 12
Solution (d) Required number = HCF of (260 − 7), (720 − 7),
LCM is also known as least common dividend. (145 − 7) = HCF of 253, 713 and 138

Keep in Mind 138)713(5 23)253(11


690 23
■ HCF × LCM = One number × Second number 23)138(6 23
LCM of numerators 138 23
■ LCM of fractions = × ×
HCF of denominators
HCF of numerators Hence, HCF of 253, 713 and 138=23
■ HCF of fractions =
LCM of denominators
12 Self Study Guide SSC Constable (GD) REcruitment Exam

Practice Exercise
1. How many numbers between 400 9 12 18 21 22. Four numbers are in the ratio of 10
11. The HCF of , , and is
and 500 are exactly divisible by 12, 10 25 35 40 : 12 : 15 : 18. If their HCF is 3, find
15 and 20? 3 252 3 52 their LCM
(a) (b) (c) (d)
(a) One (b) Two 5 7 1400 140 (a) 420 (b) 540 (c) 620 (d) 680
(c) Three (d) None of these
2 3 4 9 23. Five bells first begin to toll
2. The numbers between 200 and 300, 12. The LCM of , , , is together and then at intervals of 3,
3 5 7 13
which when divided by 6, 8 or 9 1 1 12 5, 7, 8 and 10 s. Find after what
leaves remainder 8 in each case, are (a) 36 (b) (c) (d)
36 1365 455 interval they will again toll
(a) 216, 288 (b) 224, 296 together. How many times does
(c) 210, 240 (d) 224, 288 13. The LCM of 5, 8, 12, 20 will not be they toll together in one hour?
a multiple of (a) 14 min, 3 times (b) 12 min, 4 times
3. The HCF and LCM of two numbers (a) 3 (b) 9
are 21 and 4641 respectively. If one (c) 14 min, 4 times (d) 12 min, 3 times
(c) 8 (d) 5
of the numbers lies between 200 24. Four bells ring at the interval of 6s,
and 300, the two numbers are 14. Find the LCM of 2.5, 1.2, 20 and 7.5. 8s, 12s and 18 s. They start ringing
(a) 273, 357 (b) 210, 340 (a) 60 (b) 65 (c) 70 (d) 50 together at 12’o clock. After how
(c) 215, 314 (d) 210, 252 15. The LCM of two numbers is 48. many seconds will they ring
4. Find the greatest number which The numbers are in the ratio 2 : 3. together again?
can divide 284, 698 and 1618 Find the sum of the numbers. (a) 72 (b) 84 (c) 60 (d) 48
leaving the same remainder 8 in (a) 28 (b) 32 (c) 40 (d) 64 25. Six bells commence tolling
each case. 16. The LCM of two numbers is (a + b) together and toll at intervals of 2,
(a) 23 (b) 46 and their HCF is P(a − b). If one of 4, 6, 8, 10 and 12 s respectively. In
(c) 25 (d) 29 30 min, how many times do they
the numbers is P, then other
5. Find the largest number which number is toll together?
divides 62, 132 and 237 to leave Pa (a) 4 (b) 10 (c) 15 (d) 16
(a) (b) Pab
the same remainder in each case. b
(a + b ) 26. An electric wire is sold only in
(a) 21 (b) 30 (c) a 2 − b 2 (d) multiples of 1 m and a person
(c) 35 (d) 40 P( a − b )
required several lengths of wire,
6. Find the largest number of four 17. What is the least number which each 85 cm long. To avoid any
digits exactly divisible by 12, 15, 18 when diminished by 7, is divisible wastage and to minimise labour,
and 27. by each one of 21, 28, 36 and 45? he should purchase minimum
(a) 9720 (b) 9937 (a) 1255 (b) 1177 lengths of
(c) 9999 (d) 9921 (c) 1265 (d) 1267
(a) 8.5 m (b) 17 m (c) 85 m (d) 1 m
7. Find the smallest number of five 18. Find the least number which when 27. Find the greatest possible length of
digits exactly divisible by 16, 24, 36 divided by 16, 18 and 20 leaves a the planks, if three pieces of timber
and 54. remainder 4 in each case, but is 42 m, 49 m and 63 m long have to
(a) 10244 (b) 10296 completely divisible by 7 be divided into planks of the same
(c) 10368 (d) 10291 (a) 2884 (b) 2256 length
(c) 865 (d) 3332 (a) 8 (b) 49
8. Find the smallest whole number
19. The least number which should be (c) 7 (d) 63
which is exactly divisible by
1 1 1 1 added to 2497, so that the sum is 28. Monica, Veronica are Rachat begin
1 , 2 , 3 and 4 . exactly divisible by 5, 6, 4 and 3, is to jog around a circular stadium.
3 4 2 5
(a) 3 (b) 13 (c) 23 (d) 33 They complete their revolutions in
(a) 252 (b) 154
42 s, 56 s and 63 s, respectively.
(c) 322 (d) 454 20. The smallest fraction which is
After how many seconds will they
1 exactly divisible by each of be together at the starting point?
9. The difference of two numbers is 6 5 10
9 , , is (a) 366
of their sum. Their sum is 45. Find 7 14 21 (b) 252
the LCM. 30 30 60 50 (c) 504
(a) (b) (c) (d) (d) Cannot be determined
(a) 225 (b) 100 7 98 147 294
(c) 150 (d) 200 29. The capacity of two pots are 240 L
21. How many numbers are there
10. The sum of two numbers is 528 between 4000 and 6000 which are and 112 L, respectively. Find the
and their HCF is 33. The number of exactly divisible by 32, 40, 48 and capacity of a container which can
pairs of such numbers satisfying exactly measure the contents of
60?
the above condition is the two pots
(a) 2 (b) 3
(a) 6 (b) 12 (c) 8 (d) 4 (a) 9000 cc (b) 12000 cc
(c) 4 (d) 5
(c) 16000 cc (d) 8000 cc
HCF and LCM 13

30. A least number of 4 digits when 32. A milk vendor has 21 L of cow 34. The LCM of two numbers is 520
increased by 5 is completely milk, 42 L of toned milk and 63 L and their HCF is 4. If one of the
divisible by 12, 15, 20 and 35. The of double toned milk. If he wants numbers is 52, then the other
number is [SSC Constable, 2013] to pack them in cans, so that each number is [SSC Constable, 2011]
(a) 1275 (b) 1265 can contains same number of litres (a) 40 (b) 42
(c) 1235 (d) 1255 of milk and does not want to mix (c) 50 (d) 52
31. A, B, C start running at the same any two kinds of milk in a can,
time and at the same point in the then the least number of cans Answers
same direction in a circular required, is [SSC Constable, 2012] 1 b 2 b 3 a 4 b 5 c
stadium. A completes a round in (a) 3 (b) 6 (c) 9 (d) 12 6 a 7 c 8 a 9 b 10 d
252 s, B in 308 s and C in 198 s. 11 c 12 a 13 b 14 a 15 c
33. The number of integers in between
After what time will they meet 16 c 17 d 18 a 19 c 20 a
100 and 600, which are divisible by
again at the starting point? 21 c 22 b 23 c 24 a 25 d
[SSC Constable, 2012] 4 and 6 both, is 26 b 27 c 28 c 29 c 30 b
[SSC Constable, 2012]
(a) 26 min 18 s (b) 42 min 36 s 31 d 32 b 33 c 34 a
(a) 40 (b) 42 (c) 41 (d) 50
(c) 45 min (d) 46 min 12 s

Hints & Solutions


1. (b) LCM of 12, 15,20 = 60. 8. (a) LCM of given numbers 15. (c) Let numbers are 2 x and 3 x.
1 1 1 1
Number 60 is divisible by each of 12, 15 and 1 ,2 , 3 , 4 Then, according to the question, 6 x = 48 ⇒
20 and every multiple of 60 will also be 3 4 2 5 x = 8 (LCM = 6 x)
4 9 7 21
divisible by 12,15 and 20. = LCM of , , , ∴Required sum = (2 x + 3 x ) = 5 x
3 4 2 5 = 5 × 8 = 40
120,180, 240, 300, 360, 420, 480, 540, ..... Two
LCM of (4, 9, 7, 21) 252
numbers 420 and 480 lie between 400 and = = = 252 16. (c) Product of two numbers = LCM × HCF
500. HCF of (3,4, 2, 5) 1 ( a + b ) × P( a − b )
Other number = = (a2 − b 2 )
2. (b) LCM of 6, 8, 9 = 72 9. (b) If the numbers are x and y, P
Required number = (Multiple of 72 + 8) which Then, x + y = 45 ...(i) 17. (d) LCM of 21, 28, 36, 45
lies between 200 and 300. x− y = 5 ...(ii)
By adding Eqs. (i) and (ii), we get 2 21, 28, 36, 45
= (72 × 3 + 8, 72 × 4 + 8) = (224, 296)
3. (a) Since, the HCF of two numbers is 21, x + y = 45 2 21, 14, 18, 45
hence let the numbers be 21 x and 21 y x− y = 5 3 21, 7, 9, 45
respectively. We know that, product of two 2 x = 50 3 7, 7, 3, 15
numbers ∴ x = 25
= HCF × LCM 7 7, 7, 1, 5
From Eq. (i), x + y = 45
21x × 21y = 21 × 4641 ∴ y = 45 − x 1, 1, 1, 5
∴ x y = 221 = (13 × 17 ) ⇒ y = 45 − 25 = 20
LCM = 2 × 2 × 3 × 3 × 7 × 5 = 1260
because x and y are co-prime. Now, LCM of 25 and 20 = 100 ∴Required number = LCM +7
Therefore, the numbers are (21 × 13,21 × 17 ) 10. (d) Let the required numbers be 33a and = 1260 + 7 = 1267
⇒ (273, 357) 33b. 18. (a) LCM of 16, 18, 20 = 720
4. (b) Required number = HCF of (284 − 8), Then, 33a + 33b = 528 ∴Required number = 720k + 4, where k is a
( 698 − 8) and (1618 − 8) ⇒ a + b = 16 natural number to be divisible by 7, (720k + 4)
= HCF of 276,690 and 1610 = 46 Now, co-primes with sum 16 are will be multiple of 7. Smallest value of k = 4.
5. (c) Required number = HCF of (132 − 62 ), (1, 15), (3, 13), (5, 11) and (7, 9). ∴ Required number = 720 × 4 + 4 = 2884
(237 − 132 ) and (237 − 62 ) Hence, four such pairs can be formed. 19. (c) LCM of 5, 6, 4 and 3 = 60
= HCF of 70,105 and 175 = 35 9 12 18 21 On dividing 2497 by 60, the remainder is 37
11. (c) HCF of , , ,
6. (a) Largest four digit number = 9999 10 25 35 40 ∴Number to be added = 60 − 37 = 23
LCM of 12,15,18, 27 = 540 HCF of (9, 12, 18, 21) 3 6 5 10
= = 20. (a) Required fraction = LCM of , ,
LCM of (10, 25, 35, 40) 1400 7 14 21
If 9999 is divided by 540, it leaves a remainder
LCM of 6, 5, 10
of 279. 2 3 4 9 =
12. (a) LCM of , , , HCF of 7, 14, 21
Hence, the required number 3 5 7 13
30
= ( 9999 − 279) = 9720 =
LCM of (2, 3, 4, 9) 36
= = 36 =
HCF of (3, 5, 7, 13) 1 7
7. (c) Smallest number of five digits = 10000
21. (c) LCM of 32, 40, 48 and 60 = 480
LCM of 16, 24, 36, 54 = 432 13. (b) LCM of 5, 8, 12, 20 = 120 The number divisible by 480 between 4000
If we divide 10000 by 432, we get a remainder 120 is not a multiple of 9. and 6000 are 4320, 4800, 5280 and 5760.
of 64.
14. (a) Required LCM = (LCM of 25, 12, 200 Hence, required number of numbers are 4.
∴ Required number and 75)×01. = 600 × 01
. = 60 22. (b) If numbers be 10 x, 12 x, 15 x and 18 x,
= 10000 + ( 432 − 64) = 10368
14 Self Study Guide SSC Constable (GD) REcruitment Exam

Then, this LCM = 180 x 28. (c) Required time = LCM of 42, 56 and 63 s 31. (d) Required time will they meet again at
As HCF = x LCM of 42, 56, 63 is the starting point = LCM of 252, 308 and 198 s
Hence, required LCM = 180 × 3 = 540 = 2772 s = 46 min 12 s
2 42, 56, 63
23. (c) Required time interval = LCM of 3, 5, 32. (b) Required number of cans
3 21, 28, 63 LCM of 21, 42 and 63
7, 8 and 10 =
= 840 s 7 7, 28, 21 HCF of 21, 42 and 63
= 14 min 1, 4, 3 2 × 3× 3×7
= =6
Number of times they will toll together in one 21
hour ∴Required time = 2 × 3 × 7 × 4 × 3 = 504 s
33. (c) LCM of 4 and 6 = 12
60 29. (c) Required capacity = HCF of 240 L and
= = 4 times [ignoring the fraction part] 112 L Possible numbers = 108,116,124.....588
14
∴ HCF of 240 and 112 Where, a = 108, n=?, d=12, Tn=588
24. (a) LCM of 6, 8, 12, 18 = 72
112)240(2 By formula, Tn = a +(n-1)d
They will ring together after 72 s.
224 588 =108+(n-1)12
25. (d) LCM of 2, 4, 6, 8, 10, 12 = 120.
16)112(7 480
So, the bells will toll together after every 120 s = n−1
112 12
i.e., 2 min.
× n = 40+1=41
In 30 min, they will toll together in
∴Required capacity = 16 L = 16000 cc 34. (a) First number × Second number
 30  30. (b) A least number of 4 digits = 1000
  + 1 = 16 times = HCF × LCM
 2 
LCM of 12, 15, 20, 35 = 420 ⇒ 52 × Second number = 4 × 520
26. (b) Required minimum length
The required number is 420 × n + 5 4 × 520
= LCM of 100 cm and 85 cm ⇒ Second number =
= 420 × 3 + 5 (for least) 52
= 1700 cm = 17 m
= 1260 + 5 = 1265 = 40
27. (c) Required length = HCF of 42 m, 49 m,
63 m = 7 m

CHAPTER 4

Square Root, Cube Root and Surds


Squaring and cubing of a number is largly used in To Find the Square Root of an Integer
mathematical calculations.
(i) By the Method of Prime Factors
The method of squaring is intimately connected with a
When a given number is a perfect square, we resolve it into
procedure of ‘‘Duplex combination’’ and the method of
prime factors and take the product of prime factors,
cubing is connected with a procedure of the product with
Duplex combination. Also, in this chapter we deals with choosing one out of every two.
questions based on surds of a number. Ex. 1 Find the square root of 4356.
(a) 67 (b) 68
Square (c) 63 (d) 66
Solution (d)
When a number is multiplied by itself, we get the square of 2 4356
that number. e. g., 4 × 4 = 16 i. e., square of 4 is 16. 2 2178
3 1089
Square Root 3 363
11 121
The square root of a number is that number the product of 11 11
which itself gives the given number i.e., the square root of 400
1
is 20, the square root of 625 is 25.
The square root of a number is denoted by the symbol 4356 = 2 × 2 × 3 × 3 × 11 × 11 = 2 2 × 3 2 × 112
called the radical sign. The expression ‘ 9’ is read as ‘root ∴ 4356 = 2 × 3 × 11 = 66
nine’, ‘radical nine’ or ‘the square root of nine’.
Square Root, Cube Root and Surds 15

l Thus, from the above example, it is clear that in order Solution (a) Method 1
to find the complete square root of a given number
1.35
every prime factor of that number should be repeated
1 1. 82 25
twice. Thus, we can make a number which is not a
perfect square, a perfect square by multiplying or 1
dividing the number by those factors of it which are 23 82
not contained in pairs. 69
Ex. 2 Find the least number by which 1800 be 265 1325
multiplied or divided to make it a perfect square. 1325
(a) 2 (b) 3 ×
(c) 5 (d) 7
.
18225 = 135
.
Solution (a) 1800 = 2 × 2 × 2 × 3 × 3 × 5 × 5
18225 18225 135
The least number by which the given number be multiplied Method 2 .
18225 = = = = 135
.
or divided is 2. 10000 10000 100
(ii) By the Method of Long Division To Find the Square Root of a Fraction
This method can be used when the number is large and
13
the factors cannot be determined easily. This method can Ex. 5 Find the square root of 1 .
also be used when we want to add a least number or to 36
5 1 1 15
subtract a least number from a given number so that the (a) 1 (b) 1 (c) 2 (d)
resulting number may give a perfect square of some 6 6 6 6
number. 13 49 49 7 1
Solution (b) 1 = = = =1
36 36 36 6 6
Ex. 3 Find the square root of 156816.
(a) 382 (b) 516 Important Facts
(c) 396 (d) 424 ■ The square of a number other than unity is either a multiple of
Solution (c) 4 or exceeds a multiple of 4 by 1.
396 ■ A perfect square can never end with
3 15 68 16 (a) an odd number of zeroes. (b) 2, 3, 7 and 8.
9 ■ The square root of an integer is not always an integer i.e.,
69 668 3, 5, 11 are not integers.
621 ■ ab = a× b ■
a
=
a
786 4716 b b
4716 ■ a + b ≠ a+b ■ a − b ≠ a −b
×
Firstly, mark off the digits in pairs starting from the unit’s
digit. Each pair is called a period. Cube
Now, 3 2 = 9 and 4 2 = 16. So, we take 3 2 = 9 and on
subtracting 9 from 15, we get 6 as remainder. When we multiply a number by its square then we get the
Now, bring down the next period i.e., 68.
cube of that number.
Now, double the root figure already found which is 3 and e. g., 2 × 2 2 = 8 i. e., 8 is the cube of 2.
write it to the left.
Now, from trial and error, we find 69 × 9 = 621 which is closet Cube Root
and least to 668. So, place 9 to right of 6 changing it to 69. The cube root of a number is that number the cube of
We also put another 9 to the right of the quotient 3 making it which itself gives the given number i.e., the cube root of 64
39. Now, we subtract 621 from 668. We get a remainder of is 4. The cube root of a number is denoted by the symbol
47. 3 . The expression 3 8 is read as ‘cube eight’ or the ‘cube
Now, repeat the whole process till there is no period left root of eight’.
over to be brought down. So, 156816 = 396
To Find the Cube Root of an Integer
To Find the Square Root of a Decimal By the Method of Prime Factors When a given
number is a perfect cube, we resolve it into prime
Ex. 4 Find the square root of 1.8225. factors and take the product of prime factors, choosing
(a) 1.35 (b) 1.75 one out of every three.
(c) 0.95 (d) 1.15
16 Self Study Guide SSC Constable (GD) REcruitment Exam

Ex. 6 Find the cube root of 3375. Important Facts


(a) 15 (b) 25 (c) 35 (d) 45 Let n be a positive integer and a be a real number.
Solution (a) 3 3375 ■ If a is an irrational, then n a is not a surd.
3 1125
■ If a is rational, then n a is a surd.
3 375
5 125 Laws of Radicals
5 25 As surds can be expressed with fractional exponents, the
5 5 laws of indices are therefore, applicable to surds.
1 (i) ( n a) n = a (ii) n
ab = n a n b
3375 = 3 × 3 × 3 × 5 × 5 × 5 = 3 × 5 3 3
a na
(iii) mn
a = mn
a= n m
a (iv) n =
∴ 3
3375 = 3 × 5 = 15 b nb

To Find the Cube Root of a Decimal (v) ( n a) m = n am

Ex. 7 Find the cube root of 19.683. Types of Surds


(a) 0.27 (b) 2.7 (c) 3.7 (d) 1.7
(i) Pure Surds A surd which has unity only as rational
19683
Solution (b) 3 19.683 = 3 factor, the other factor being irrational is called a pure
1000
surd.
3
19683 3 3 9 33 27 (ii) Mixed Surds A surd which has a rational factor other
= 3
= = = = 2.7
3
1000 10 3 10 10 than unity, the other factor being irrational, is called a
⇒ 19683 = 3 9 mixed surd.

To Find the Cube Root of a Fraction Comparison of Two Surds


If two surds are of same order, then the one whose
61 radicand is larger, is the larger of the two. If two surds of
Ex. 8 Find the cube root of 1 .
64 different order are to be compared, then we reduce them to
3 1 3 1 the same but smallest order and then compare.
(a) 1 (b) 2 (c) 2 (d) 1
4 4 4 4
61 3 125 3 125 3 53 5 1 Rationalizing Factors
Solution (d) 31 = = 3 = = =1
64 64 64 3 3 4 4 When the product of two surds is rational, then each one of
4
them is called the rationalizing factor of the other.
Important Facts

3
ab = 3 a × 3 b ■ 3
a 3a
=
Rationalization of Surds
b 3b
The process of converting a surd to a rational number by

3
a + 3b ≠ 3a+b ■
3
a − 3b ≠ 3a −b multiplying it to a suitable rationalizing factor is called the
rationalization of the surd.
Ex. 9 Find the smallest number by which 2400 be
divided to make it a perfect cube. Ex. 11 The value of
(a) 225 (b) 45 (c) 300 (d) 325 x 2 + xy + y 2 y 2 + yz +z 2
Solution (c) 2400 = 25 × 52 × 3  ax   ay 
 y × z
To make it a perfect cube, it must be divided by2 2 × 5 2 × 3 = 300 a  a 
z 2 + xz + x 2
Ex. 10 Find the smallest number by which 2400 be  az 
× x  is
multiplied to make it a perfect cube. a 
(a) 30 (b) 90 (c) 45 (d) 36 (a) 0 (b) 1
Solution (b) 2400 = 2 × 5 × 3 5 2
(c) a (d) xyz
To make it a perfect cube, it must be multiplied by x 2 + xy + y 2 y 2 + yz+ z2 z2 + xz+ x 2
 x  y  z
2 × 5 × 3 2 = 90 Solution (b)  y  × a  × x 
a a

a a z  a 
     
Surds or Radicals = ( ax − y ) x
2
+ xy + y 2
× ( ay − z) y
2
+ yz+ z2
×( az− x) z2 + xz+ x 2

If n a is irrational, where a is a rational number and n is a = ( a) x


3
−y 3
× ( a) y
3
− z3
× ( a) z
3
− x3
positive integer, then n a or a1/ n is called a surd or radical of 3
− y 3 + y 3 − z3 + z3 − x 3
order n and a is called the radicand. = ax = a0 =1
Practice Exercise
Square root and Cube root 13. Find the smallest number that 126 × 63 × 45
23. If x = , then the
must be added to 103141 to make 147 × 243
1. Evaluate 234 − 73 + 69 − 25 it a perfect square. value of x is
(a) 15 (b) 18 (c) 12 (d) 17 (a) 543 (b) 545 (c) 532 (d) 536 (a) 5 (b) 10 (c) 10 (d) 5
484 + 121 14. Find the smallest number that 24. Find the value of
2. Evaluate must be subtracted from 9100 to
121 20 + 20 + 20 + ... .
1 make it a perfect square.
(a) (b) 0.03 (c) 0.3 (d) 3 (a) 25 (b) 30 (c) 50 (d) 75
3 (a) 5 (b) 4
15. A general wishing to draw up his (c) 6 (d) Cannot be determined
3. Evaluate 3 0. 000729
4094 men in the form of a solid
(a) 0.09 (b) 0.9 (c) 9 (d) 0.009 square found that he had 2 men 25. Find the value of 4 4 4 4 4
4. What least fraction must be less. If he may get 2 more men and 31
13 form a solid square, the number of (a) 0 (b) 4 32 (c) 1 (d) 4
subtracted from the square root1 men in each row is
36 3+ 2
to make the result a whole number? (a) 68 (b) 64 (c) 66 (d) 62 26. If a = , then the value of
3− 2
1 1 1 1 16. The sum of squares of two
(a) (b) (c) (d) 1
5 6 9 8 numbers is 114 and the difference a+ is
of their squares is 14. What is the a
5. What least fraction must be added (a) 13 (b) 10
1 product of the two numbers?
to the square root of 2 to make (c) 6 (d) 9
4 (a) 40 2 (b) 40
(c) 36 2 (d) 36 2 +1 2 −1
the result a whole number? 27. If x = and y = , then
(a) 1/2 (b) 1/8 (c) 1/16 (d) 1/4 2 −1 2 +1
17. If 3 = 6561, then the value of n is
n

0. 064 × 0. 4 the value of (x 2 + y 2) is


(a) 66 (b) 16
6. Find the value of (a) 34 (b) 36 (c) 32 (d) 38
. × 6. 25
016 (c) 243 (d) 27
(a) 0.016 (b) 16 (c) 0.16 (d) 1.6 18. If 0. 09 × 0. 9 × x = 0. 9 × 0. 09 × y, 3 +1 3 −1
28. If a = and b = , then
7. What is the ascending order of 12, x 3 −1 3 +1
then the value of is
12 24 y  a 2 + ab + b 2 
, ? (a) 0.081 (b) 0.81 the value of  2  is
5 2  a − ab + b 2 
24 12 12 24 (c) 8.1 (d) 0.0081
(a) , , 12 (b) , , 12 15 16 11 12
2 5 5 2 19. If 18496 = 136, then the value of (a) (b) (c) (d)
12 24 12 24 13 13 13 13
(c) 12 , ,
5 2
(d)
5
, 12 ,
2 18496 + 184.96 + 1.8496 is
29. If x = 12 + 12 + 12 + 12 + ...
x 14 (a) 1497.36 (b) 149.736
8. If 1 + = , then x is equal to (c) 14.9736 (d) 1.49736 and
169 13
(a) 9 (b) 24 (c) 16 (d) 27 20. If 29791 = 31, then the value of
3
y = 12 − 12 − 12 − 12 − ... , then
9. Find the least number by which
3
29791 + 3 29. 791 + 3 0. 029791 is
the value of x − y will be
31752 be divided to make it a (a) 34.41 (b) 33.12 (c) 344.1 (d) 331.2 (a) 0 (b) 1 (c) 4 (d) 2
perfect square.
(a) 2 (b) 3 (c) 4 (d) 7  127  30. If x = 64, then the value of
21. Find 3 1 − .
 343 x −3 x
10. Find the least number by which is
69984 be multiplied to make it a (a)
6
(b)
7
(c)
6
(d)
5 2
perfect square. 7 6 5 6 (a) 4 (b) 2
(a) 6 (b) 4 (c) 2 (d) 3 (c) 16 (d) 8
22. The students present in an
11. Find the least number by which auditorium are asked to stand in 31. Value of the expression
3
175760 be multiplied to make it a rows. If there were 4 students
extra in a row there would be 4 ( 4)5 × ( 4)−3  2
perfect cube.   is
(a) 100 (b) 160 less rows. However, if 2 students  ( 4)−2 
(c) 80 (d) 27 were less in a row, there would be (a) 2 6 (b) 2 2
4 more rows. The number of (c) 2 8 (d) 2 4
12. Find the least number by which
students present in the auditorium
234375 be divided to make it a
is 32. The value of (122 + 52)1 / 2 is
perfect cube. (a) 11 (b) 13
(a) 80 (b) 96
(a) 20 (b) 8 (c) 15 (d) 10 (c) 12 (d) 15
(c) 100 (d) 128
18 Self Study Guide SSC Constable (GD) REcruitment Exam

33. If 9 x = 12 + 147, then the value 37. The smallest number by which 41. 64 − 36 is equal to
of x is 243000 be divided so that the
[SSC Constable, 2011]
(a) 1 (b) 2 (c) 3 (d) 4 quotient is a perfect cube is
(a) − 2 (b) 2
34. The value of (7 2 + 5) (7 2 − 5) is [SSC Constable, 2015] (c) 0 (d) 1
(a) 1 (b) 3 (c) 27 (d) 9
(a) 37 (b) 171 42. If 0.42 × 100k = 42, then the value
(c) 73 (d) 14 2 38. The square root of
of k is [SSC Constable, 2011]
0.324 × 0.081 × 4.624
Previous Year‘s Questions is (a) 4 (b) 2
1.5625 × 0.0289 × 72.9 × 64 (c) 1 (d) 3
35. 1008 divided by which single digit
[SSC Constable, 2012]
number gives a perfect square? Answers
[SSC Constable, 2015] (a) 24 (b) 2.4 (c) 0.024 (d) 1.2
(a) 8 (b) 7 1 a 2 d 3 a 4 b 5 a
39. The least integer which should be
(c) 4 (d) 9 6 c 7 b 8 d 9 a 10 a
added to 1000, so as to make it a
11 a 12 c 13 a 14 d 15 b
36. The value of 0.000441 is equal to perfect quare is
16 a 17 b 18 a 19 b 20 a
[SSC Constable, 2012]
[SSC Constable, 2015] (a) 10 (b) 18 (c) 24 (d) 89 21 a 22 b 23 b 24 a 25 b
(a) 0.0021 26 b 27 a 28 a 29 b 30 b
(b) 0.21
40. The simplified value of 0.25 × 2.25 31 a 32 b 33 c 34 c 35 b
(c) 0.00021 is [SSC Constable, 2012] 36 d 37 d 38 c 39 c 40 c
(d) 0.021 (a) 0.075 (b) 0.705 (c) 0.750 (d) 7.500 41 b 42 c

Hints & Solutions


1. (a) Given expression x 14 14. (d)
8. (d) Given, 1 + = 95
169 13
= 234 − 73 + 69 − 25 9 9100
Squaring both sides 81
= 234 − 73 + 69 − 5 1+
x
=
196 185 1000
169 169 925
= 234 − 73 + 64 x 196 27 75
⇒ = − 1=
= 234 − 73 + 8 = 234 − 81 169 169 169
Now, ( 95)2 = 9025
= 234 − 9 = 225 = 15 ⇒
x
=
27
⇒ x = 27 ∴ Required number to be subtracted = 75
484 + 121 22 + 11 33 169 169
2. (d) = = =3 15. (b) Number of men in each row
121 11 11 9. (a) 31752 = 2 3 × 34 × 7 2
= 4094 + 2 = 4096 = 64
729 3 729 To make it a perfect square, it must be divided
3. (a) 3 0.000729 = 3 6 = 16. (a) Let the two numbers be x and y.
10 3
10 6 by 2.
x 2 + y 2 = 114 …(i)
3
93 9 x 2 − y 2 = 14
= = = 0 .09 10. (a) 69984 = 2 5 × 37 …(ii)
3 6 100
10
To make it a perfect square, it must be Adding 2 x 2 = 128
13
4. (b) 1 =
49
=
49 7
= =1
1 multiplied by 2 × 3 = 6. ⇒ x 2 = 64 ⇒ x = 8
36 36 36 6 6 11. (a) 175760 = 2 4 × 5 × 133 Substituting x = 8 in Eq. (i), we get y = 5 2
∴ Fraction to be subtracted is 1/6. Product of the numbers = 8 × 5 2 = 40 2
To make it a perfect cube, it must be multiplied n
1 9 9 3 1 by 2 2 × 5 2 = 100 17. (b) 3n = 38 ⇒ 3 2 = 38
5. (a) 2 = = = =1
4 4 4 2 2 12. (c) 234375 = 57 × 3 ∴
n
⇒ 8 ⇒ n = 16
1
∴ Fraction to be added is . To make it a perfect cube it must be divided by 2
2 5 × 3 = 15 18. (a) Given,
0 .064 × 0 .4 64 × 4 13. (a) 0 .09 × 0 .9 × x = 0 .9 × 0 .09 × y
6. (c) = 321
0 .16 × 6 .25 16 × 625 Squaring both sides,
3 103141
8× 2 16 0 .09 × 0 .9 × x = 0 .9 × 0 .9 × 0 .09 × 0 .09 × y
= = = 016
. 9
4 × 25 100 62 131
x
= 0.9 × 0.09 = 0.081
7. (b) 12 = 4 × 3 = 2 × 3 y
124
= 2 × 1.732 = 3.464 641 741 19. (b) 18496 + 184.96 + 18496
.
12 24 4× 6 641 18496 18496
= 2.4, = = 6 = 2.45 = 18496 + +
5 2 2 100 100 10000
So, ascending order 136 136
∴ Number to be added = ( 322 )2 − 103141 = 136 + +
12 24 10 100
= , , 12 = 103684 − 103141 = 543
5 2 = 136 + 13 .6 + 0136
. = 149 .736
Square Root, Cube Root and Surds 19

( 2 + 1)2 2 + 1+ 2 2
3
20. (a) 3 29791 + 3 29 .791 + 3 0 .029791
= =  ( 4 )5 × ( 4 )−3  2
29791 3 29791 ( 2 )2 − (1)2 2 −1 31. (a) Given expression =  
= 3 29791 + 3 +  ( 4 )−2 
1000 1000000 = 3+ 2 2 3 3
31 31 ∴ x 2 = ( 3 + 2 2 )2 = 9 + 8 + 12 2  2 5 × 2 −3  2  2 2  2
3 3
= 31 + + 2+2 2
10 100 = 17 + 12 2
= −2  =  −2  = (2 ) = (2 4 )2 = 2 6
 2   2 
= 31 + 3 .1 + 0.31 = 34 .41 ( 2 − 1) ( 2 − 1) ( 2 − 1)2
and y = × = 32. (b) (12 2 + 5 2 )1/ 2 = (144 + 25)1/ 2
( 2 + 1) ( 2 − 1) ( 2 )2 − (1)2
21. (a) Given expression = 3  1 −
127 
 = (169)1/ 2
 343  2 + 1− 2 2
= = 3−2 2
343 − 127 216 3
216 6 2 −1 = (13 2 )1/ 2 = 13
= 3 = 3 = =
343 343 3
343 7 ∴ y = ( 3 − 2 2 ) = 9 + 8 − 12 2
2 2
33. (c)Q 9 x = 12 + 147
22. (b) Let there are r rows and there are x = 17 − 12 2 ⇒ 9 x = 2 ×2 × 3 + 3×7 ×7
students in each rows. Now, x 2 + y 2 = (17 + 12 2 ) + (17 − 12 2 )
⇒ 9 x =2 3+7 3
∴ ( x + 4) × ( r − 4) = x × r = 34
⇒ 9 x = 3(2 + 7 )
x × r − 4 x + 4r − 16 = x × r ( 3 + 1) ( 3 + 1)
28. (a) a = × ⇒ 9 x =9 3
− 4 x + 4r = 16 ...(i) ( 3 − 1) ( 3 + 1)
On comparing, we get x = 3
( 3 + 1) 2
3 + 1+ 2 3
( x − 2 ) × ( r + 4) = x × r = = 34. (c) (7 2 + 5) (7 2 − 5) = (7 2 ) 2− ( 5) 2
( 3 )2 − (1)2 3−1
x × r + 4x − 2r − 8 = x × r [Q( x + y) ( x − y) = ( x )2 − ( y)2]
4+ 2 3
4x − 2r = 8 = =2+ 3
...(ii)
2 = 98 − 25 = 73
Adding Eqs. (i) and (ii), we get ∴ a 2 = (2 + 3 )2 = 4 + 3 + 4 3 35. (b) Factor of 1008
2 r = 24 = 2 ×2 ×2 ×2 × 3× 3×7
= (7 + 4 3 )
So, 1008 is divided by 7 to get a perfect
r = 12 ( 3 − 1) ( 3 − 1)
and b = × square.
∴ 4 x − 24 = 8 ( 3 + 1) ( 3 − 1)
36. (d) Let x = 0.000441
4 x = 32 ( 3 − 1)2 3 + 1− 2 3 21 × 21
= = x=
441
,x= ,
( 3 )2 − (1)2 3−1 1000 × 1000
x=8 1000000
4−2 3 x=
21
, x = 0.021
∴ Total number of students = 8 × 12 = 96 = =2 − 3
2 1000
126 × 63 × 45 ∴ b = (2 − 3 ) = 4 + 3 − 4 3
2 2 37. (d) Factors of
23. (b)Q x =
147 × 243 = (7 − 4 3 ) 243000 = 3 × 3 × 3 × 2 × 2 × 2
2 × 3× 3×7 × 3× 3×7 × 3× 3× 5 ( 3 + 1) ( 3 − 1) × 5× 5× 5× 3 × 3
= ab = × =1 So, 243000 is divided by 9 to get a perfect
3×7 ×7 × 3× 3× 3× 3× 3 ( 3 − 1) ( 3 + 1)
cube.
2 × 3× 3×7 × 3× 3×7 × 3× 3× 5 a 2 + ab + b 2 a 2 + b 2 + ab 0.324 × 0.081 × 4.624
= Now, = 38. (c)
3×7 ×7 × 3× 3× 3× 3× 3 a − ab + b
2
a 2 + b 2 − ab
2
15625
. × 0.0289 × 72.9 × 64
(7 + 4 3 ) + (7 − 4 3 ) + 1 15 324 × 81 × 4624
3 × 3 × 3 × 7 10 = = = =
9
= = 10 (7 + 4 3 ) + (7 − 4 3 ) − 1 13
3× 3× 3×7 15625 × 289 × 729 × 64 15625
24. (a) Let the given expression be x. 29. (b) x = 12 + 12 + 12 + 12 + ... ∴ The square root =
9
⇒ 20 + x = x is satisfied for x = 5 15625
25. (b) General rule = 12 + x ⇒ 12 + x = x 3
= = 0.024
If x is repeated n times, then its value is is satisfied for x = 4 125
n n 39. (c)Q 312 < 1000 < 32 2
given by x( 2 − 1)÷2
y = 12 − 12 − 12 − 12 − ... 961 < 1000 < 1024
∴ Value of the given expression
31
5
− 1)÷25 ⇒ 12 − y = y Hence, 24 should be added to 1000, so as to
= 4( 2 = 4 32 make it a perfect square.
is satisfied for y = 3
( 3 + 2) ( 3 + 2) 40. (c) Required value = 0.25 × 2.25
26. (b) a = ×
( 3 − 2) ( 3 + 2) ∴ x− y = 4− 3 = 1
= 0.5 × 15
. = 075
.
( 3 + 2) 3+ 2 + 2 6
2 30. (b) Given expression 41. (b) 64 − 36 = 8 − 6 = 2
= = = (5 + 2 6 )
( 3 )2 − ( 2 )2 3−2 x−3 x 64 − 3 64
= = 42. (c) 0.42 × 100k = 42
2 2
1 3− 2 ⇒
42
× 100 k = 42
Similarly, = = 5−2 6 1 1 1 1
a 3+ 2 ( 64)2 − ( 64)3 (2 6 )2 − (2 6 )3 100
= = 42 × 100
 1 2 2 ⇒ 100 k = = 1001
∴  a +  = ( 5 + 2 6 ) + ( 5 − 2 6 ) = 10 42
 a 23 − 22 8 − 4 4
= = = =2 ∴ k=1
( 2 + 1) ( 2 + 1) 2 2 2
27. (a) x = ×
( 2 − 1) ( 2 + 1)
20 Self Study Guide SSC Constable (GD) REcruitment Exam

CHAPTER 05

Fundamental Arithmetical Operations

Ex. 1 Simplify 25 − [10 − {9 − 2 ( 9 − 3 + 2)}]


‘VBODMAS’ Rule (a) 16 (b) 12 (c) 25 (d) 30
This rule gives the correct sequence in which the Solution (a) Given expression
mathematical operations are to be executed so as to find = 25 − [10 − {9 − 2 (9 − 3 + 2)}]
out the value of a given expression. = 25 − [10 − {9 − 2 (9 − 5)}]
Here, ‘V’ stands for Vinculum (or Bar), ‘B’ stands for = 25 − [10 − {9 − 2 ( 4)}] = 25 − [10 − {9 − 8}]
‘Bracket’, ‘O’ stands for ‘Of ’, ‘D’ stands for ‘Division’, ‘M’ = 25 − [10 − 1] = 25 − 9 = 16
stands for ‘Multiplication’, ‘A’ stands for ‘Addition’ and ‘S’
x 3 x2 + y2
stands for ‘Subtraction’. Ex. 2 If = , find the value of 2 ⋅
(A) Here, ‘VBODMAS’ gives the order of simplification. y 2 x − y2
Thus, the order of performing the mathematical 1 2 1 3
(a) 1 (b) (c) 1 (d) 2
operations in a given expression is 5 3 2 5
First : Vinculum or bar x2
+1
Second : Bracket x +y 2
y2
2
Solution (d) 2 2 = 2
Third : Of x −y x
−1
Fourth : Division y2
Fifth : Multiplication [dividing both numerator and denominator by y 2 ]
2
Sixth : Addition  x  3
2
9 13
  +1   +1 +1
Seventh : Subtraction  y 2 4 4 = 13 = 2 3
= 2
= 2
= =
 x  3 9 5 5 5
The above order should strictly be followed. −1
  −1   −1 4
 y 2 4
(B) There are three types of brackets.
1. Square brackets [ ] 2. Curly brackets { } 2
Ex. 3 Find the value of 3 − .
3. Circular brackets ( ) 1
1+
Thus, in simplifying an expression all the brackets must be 1
1+
removed in the order, ‘( )’, ‘{}’ and ‘[]’. 2
1+
5
Most Important Formulae 1 3 7 14
(a) 1 (b) 2 (c) (d) 1
■ (a + b) = a + 2ab + b
2 2 2
2 5 9 19
■ (a − b)2 = a 2 − 2ab + b 2 Solution (d) Given expression
2 2 2
■ (a + b)2 − (a − b)2 = 4 ab =3− =3− =3−
1 1 1
■ (a + b)2 + (a − b)2 = 2 (a 2 + b 2 ) 1+ 1+ 1+
1 1 5
1+ 1+ 1+
■ (a 2 − b 2 ) = (a + b) (a − b) 2  7 7
1+  
■ (a + b + c)2 = a 2 + b 2 + c 2 + 2 (ab + bc + ca) 5  5
(a 3 + b 3 ) = (a + b)(a 2 − ab + b 2 ) 2 2 2

=3− =3− =3−
(a − b ) = (a − b) (a + ab + b )
3 3 2 2
1+
1
1+
7  19

 
12 12 12
(a 3 + b 3 + c 3 − 3 abc) = (a + b + c)  
 7

(a 2 + b 2 + c 2 − ab − bc − ca)
2 × 12 57 − 24 33 14
■ a 3 + b 3 + c 3 = 3 abc, if a + b + c = 0 =3− = = =1
19 19 19 19
Fundamental Arithmetical Operations 21

Ex. 4 Simplify ( 69.98) 2 − ( 30.02) 2 . a 5 8a + 3b


Ex. 6 If = , then find the value of .
(a) 3996 (b) 4386 (c) 1111 (d) 1000 b 4 8a − 3b
Solution (a) Given expression is of the form 2 6
(a) 1 (b)
a2 − b 2 = ( a + b) ( a − b) 3 7
6 3
∴ a = 69.98 and b = 30.02 (c) 1 (d) 2
So,(69.98) 2 − (30.02) 2 = (69.98 + 30.02) (69.98 − 30.02) 7 5
 a
= (100 × 39.96) = 3996 8  +3
8a + 3b  b
Solution (c) =
0. 3 × 0. 3 + 0.03 × 0.03 − 0.6 × 0.03 8a − 3b  a
Ex. 5 Simplify 8   −3
0.54  b
(a) 0.45 (b) 0.135 (c) 0.145 (d) 0.27 [dividing numerator and denominator by b]
Solution (b) 0.3 × 0.3 + 0.03 × 0.03 − 0.6 × 0.03  5
8  +3
= 0.3 × 0.3 + 0.03 × 0.03 − 2 × 0.3 × 0.03 is of the form  4
=
( a2 + b 2 − 2ab) = ( a − b) 2  5
8   −3
 4
Let a = 0.3 and b = 0.03 Then, (0.3 − 0.03) 2 = (0.27) 2
0.27 × 0.27 13 6
∴ Given expression = = 0.135 = =1
0.54 7 7

Practice Exercise
1. Simplify [1 − 2 (3 − 4)−1 ]−1 (256)2 − (144)2 13. Find the value of
7. Find the value of .
(a) 1/4 (b) 1/3 (c) 1/2 (d) 1/6 256 − 144 (214)3 − (186)3
.
1 (a) 600 (b) 400 (c) 300 (d) 100 (214) + (214)(186) + (186)2
2
2. Simplify 1 +
1 8. Find the value of x in (a) 28 (b) 26 (c) 22 (d) 24
2+
1 (25. 25)2 − (14 .75)2 3 3
3+ = 20  4  3
3   − 
x  7  7
(a)
19
(b)
17
(c)
33
(d)
11 (a) 26 (b) 23 (c) 28 (d) 21 14. Simplify 2 2
.
23 23 23 23  4  3
(0.7) − (0.6)
4 4   − 
9. Simplify .  7  7
(0.7)2 + (0.6)2 37 39 33 36
   3  (a) (b) (c) (d)
3. Find x, if 6 − 5 −  x −  2 −   = 3 (a) 0.13 (b) 1.3 (c) 1.1 (d) 0.11 49 49 41 49
   2 
5 3 7 9 10. Find the value of 15. Simplify
(a) (b) (c) (d)
2 2 2 2  0.049 × 0.049 × 0.049 + 0.051   1   1   1  1
1 +  1 −  1 +  1 − 
1  × 0.051 × 0.051   4  4  5  5
4. Simplify 2 +  
 0.049 × 0.049 − 0.049 × 0.051 
1  1  1
1+ 1 +  1 −  .
1+
1  + 0.051 × 0.051   6  6
2 3 5 7 1
8 7 9 13 (a) 0.1 (b) 0.01 (c) 0.5 (d) 0.05 (a) (b) (c) (d)
(a) (b) (c) (d) 8 8 8 8
5 5 5 5 11. Find the value of
2125 × 2125 − 2125 × 875 + 875 16. A person on tour has ` 360 for his
5. What least fraction should be
daily expense. He decides to
added to × 875 extend his tour programme by 4
1 1 1 1 .
+ + + K+ , 2125
. × 2125
. × 2125
. + 0.875 days which leads to cutting down
2× 3 3× 4 4 × 5 21 × 22
× 0.875 × 0.875 daily expense by ` 3 a day. The
so that the result is unity. number of days of his tour
(a) 2 × 10 3 (b) 3 × 10 3
(a) 3/11 (b) 6/11 (c) 5/11 (d) 7/11 1 programme is
(c) × 10 6 (d) 4 × 10 3
a 2 b 9 3 (a) 15 (b) 20 (c) 18 (d) 16
6. If = and = , then find 17. At a party, there are 43 persons in
b 3 c 16 12. Find the value of
c 2 − a2 all. The number of women is two
. (4 . 256 − 1.744)2 + (4 . 256 + 1.744)2 more, than men but the number of
c 2 + a2 .
55 55 55 55
4 . 256 × 4 . 256 + 1.744 × 1.744 children is 4 less than men. How
(a) (b) (c) (d) (a) 8 (b) 2 (c) 4 (d) 6 many women are in the party?
72 76 73 79
(a) 17 (b) 13 (c) 11 (d) 25
22 Self Study Guide SSC Constable (GD) REcruitment Exam

18. Out of a group of swans, 7/2 times 20. Ram went to a market and bought 1
21. 1 + is equal to
the square root of the number are one copy of a Mathematics book 1
playing on the shore of the pond. 1+
and two pencils for `165. Rahim 5 [SSC Constable, 2011]
The two remaining are inside the went to the same market and (a) 11/6 (b) 13/6
pond. What is the total number of bought another copy of the same (c) 15/6 (d) None of these
swans?
book and ten pencils of the same
(a) 10 (b) 14 (c) 12 (d) 16
brand for `169. The price of each Answers
19. The fuel indicator in a car shows pencil was
1 5th of the fuel tank as full. When 1 b 2 c 3 a 4 d 5 c
[SSC Constable, 2012]
22 more litres of fuel are poured 6 c 7 b 8 d 9 a 10 a
(a) ` 0.50
into the tank, the indicator rests at (b) ` 1
11 c 12 b 13 a 14 a 15 c
the three-fourth of the full mark. (c) ` 075
. 16 b 17 a 18 d 19 d 20 a
Find the capacity of the fuel tank. (d) ` 2 21 a
(a) 25 L (b) 35 L (c) 30 L (d) 40 L

Hints & Solutions


1. (b) Given expression = [1 − 2 ( 3 − 4)−1 ]−1  1 1  1 1  1 1 [( 0 ⋅ 7 )2 ]2 − [( 0 ⋅ 6)2 ]2
= − + − +  −  9.(a) Given expression =
−1 −1 −1  2 3  3 4  4 5 ( 0 ⋅ 7 )2 + ( 0 ⋅ 6)2
= [1 − 2 ( −1) ] = [1 − 2 ( −1)]
1  1 1 . ) + ( 0.6) ][( 07
[( 07 2
. ) − ( 0.6)2 ]
2 2
= [1 + 2 ]−1 = 3 −1 = + ... +  −  =
3  21 22  . )2 + ( 0.6)2 ]
[( 07
1
2. (c) Given expression = 1 + =
1 1
− =
10
=
5 [using the algebraic expression
1
2+ 2 22 22 11 a 2 − b 2 ( a + b )( a − b )
1 = = ( a − b )]
3+ 5 6 a+ b (a + b )
3 Thus, least fraction to be added = 1 − =
11 11 = ( 07
. )2 − ( 0.6)2 = ( 07
. + 0.6)( 07
. − 0.6)
1 1
= 1+ = 1+ a 2 b 9
1 3 6. (c) Given, = and = = 013
.
2+ 2+ b 3 c 16
10 10 10. (a) Given expression
a b a a 2 9 3
3 ⇒ × = ; = × = ( 0.049)3 + ( 0.051)3
1 10 33
b c c c 3 16 8 =
= 1+ = 1+ =  a2  ( 0.049) − 0.049 × 0.051 + ( 0.51)2
2
23 23 23 1−  2
10 c −a
2 2
c  Here,
=
   c 2 + a2  a2  ( 0.049 + 0.051) [( 0.049)2 − 0.049
3  1+  2
3. (a) Given, 6 − 5 −  x − 2 −   = 3 c  × 0.051 + ( 0.051)2 ]
   2   =
 a
2
 3
2
( 0.049) − 0.049 × 0.051 + ( 0.051)2
2
 1 
6 −  5 −  x −   = 3 1−   1−   9
⇒ 1−
 2  c  8
 = = = 64 [Using the algebraic expression
2 2 9
 2 x − 1   a  3 1+ a 3 + b 3 = ( a + b )( a 2 − ab + b 2 ]
⇒ 6 −  5 −   =3 1+   1+  
 2   c  8 64
 = ( 0.049 + 0.051) = 01
.
55
11 − 2 x 
⇒ 6−  =3 55 64 55 11. (c) Given expression
 2  = 64 = × =
73 64 73 73 2125 × 2125 − 2125 × 875
1+ 2 x 5 + 875 × 875
⇒ = 3 ⇒ 1 + 2 x = 6 or 2 x = 5 ⇒ x = 64 =
2 2 2.125 × 2.125 × 2.125 + 0.875
7. (b) The expression is of the form
4. (d) Given expression a 2 − b 2 ( a + b )( a − b ) × 0.875 × 0.875
1 1 = = (a + b )
=2+ =2+ a−b (a − b ) [(2.125)2 − 2.125 × 0.875
1 1
1+ 1+ = (256 + 144) = 400
1 3 + ( 0.875)2 ] × 10 6
1+ =
(25 .25)2 − (14 .75)2 (2.125)3 + ( 0.875)3
2 2 8. (d) Given, = 20
x
=2+
1
=2+
1 10 6 × [(2.125)2 − 2.125
2 5 . )2 − (14 .75)2
(2525
1+ ⇒ x= ×0.875 + ( 0.875)2 ]
3 3 20 =
3 13 . + 14 .75)(2525
(2525 . − 14 .75) (2.125 + 0.875)[(2.125)2
=2+ = ⇒ x=
5 5 20 − (2.125 × 0.875) + ( 0.875)2 ]
5. (c) Given expression is [using the algebraic expression [using the algebraic expression
1 1 1 1 a 2 − b 2 = ( a + b )( a − b )] a 3 + b 3 = ( a + b )( a 2 − ab + b 2 )]
+ + +K +
2 × 3 3× 4 4× 5 21 × 22 40 × 10.5
⇒ x= = 21 =
1
× 10 6
20 3
Fundamental Arithmetical Operations 23

12. (b) Given expression is of the form 16 12 9 18. (d) Let the total number of swans be x.
+ +
Let 4.256 = a, 1744 =b 49 49 49 37 7
. = = The number of swans playing on shore = x
7 49
( a − b )2 + ( a + b )2 2
= 7
a2 + b 2 Number of remaining swans = 2
15.(c) Given expression is 7 7
a 2 + b 2 − 2 ab + a 2 + b 2 + 2 ab ∴ x= x + 2 ⇒ (x − 2) = x
=  1  1  1  1  1  1 2 2
a2 + b 2 1 +  1 −  1 +  1 −  1 +  1 − 
 4  4  5  5  6  6 Now, among all the options, x = 16
2 (a + b )
2 2
5 3 6 4 7 5 7 i.e., option (d) satisfies above equation.
= =2 = × × × × × =
(a2 + b 2 ) 4 4 5 5 6 6 8 19. (d) Let the capacity of the fuel tank
13. (a) Given expression 16. (b) Person’s daily expenses = ` x be x L.
(214)3 − (186)3 Number of days tour last = y days Given,
x
+ 22 = x
3
=
(214) + (214)(186) + (186)2
2 So, x × y = 360 ...(i) 5 4
( x − 3) ( y + 4) = 360 ...(ii)  3 1 11
 a3 − b 3  ∴  −  x = 22 ⇒ x = 22 ⇒ x = 40 L
 2 = a − b Solving Eqs. (i) and (ii), we get  4 5 20
 a + ab + b 2
 y = 20, − 24 20. (a) According to the question,
= [214 − 186] = 28 Discarding negative value number of days, Cost price of →
[using the algebraic expression y = 20 days (1 book + 2 pencils) = ` 165 …(i)
a 3 − b 3 = ( a − b )( a 2 + ab + b 2 )] 17. (a) Let the number of men in the party be Cost price of →
14. (a) Given expression is of the form M. (1 book + 10 pencils) = ` 169 …(ii)
a3 − b 3 ( a − b )( a 2 + ab + b 2 ) ∴Number of women = M + 2 On subtracting Eq. (i) from Eq. (ii), we get
= and number of children = M − 4
a2 − b 2 ( a + b )( a − b ) 8 pencils = ` 4
Now, According to the question, 4
a 2 + ab + b 2 ∴ 1 pencil = ` = ` 0.50
= M + M + 2 + M − 4 = 43 8
a+ b
3 3 2 2 ⇒ 3M = 43 + 2 = 45 21. (a) Expression = 1 +
1
= 1+
1
 4  3  4  4  3  3 45 5+1
  −    +    +   ∴ M = = 15 1+
1
7 7 7 7 7 7
∴ 2 2
= 3 5 5
 4  3 4 3 ∴Number of women in party = 15 + 2 = 17 6 + 5 11
+ = 1+
5
= =
  − 
7 7 7 7 6 6 6

CHAPTER 06

Ratio and Proportion


Ratio Solution (c) Here, we have 80 paise
The number of times one quantity contains another and ` 5 = 5 × 100 = 500 paise
80
quantity of the same kind is called ratio of two unit. ∴ Their ratio = = 4 : 25
500
Or
The ratio of two quantities in the same units is the fraction
Ex. 2 Two natural numbers are in the ratio 3 : 5 and
that one quantity is of the other.
a their product is 2160. The smaller number is
Thus, the ratio a to b is the fraction written as a : b. (a) 36 (b) 24
b
(c) 18 (d) 12
In the ratio a : b , the first term a is antecedent and second
Solution (a) Let the natural numbers be 3x and 5x, then
term b is consequent.
3x × 5x = 2160 ⇒15x2 = 2160
Ex. 1 What is the ratio of 80 paise and ` 5? ⇒ x2 = 144 ⇒ x = 12
(a) 2 : 25 (b) 4 : 35 Hence, smaller number = 3x = 3 × 12 = 36
(c) 4 : 25 (d) 6 : 35
24 Self Study Guide SSC Constable (GD) Recruitment Exam

Properties of Ratio 5. If A : B = N1 : D1 and B : C = N2 : D2


Then A : B : C = N1 N2 : D1 N2 : D1D2
Some properties of ratio are as follows
6. If A : B = N1 : D1 , B : C = N2 : D2 and C : D = N 3 : D 3
1. The value of a ratio remains unchanged, if each one
of its terms is multiplied or divided by a same Then A : B : C : D = N1 N2 N 3 : D1 N2 N 3 : D1D2 N 3: D1D2D 3
non-zero number. a c a+b c+ d a−b c− d
7. If = , then = and =
2. a2 : b 2 is the duplicate ratio of a : b. b d a−b c− d a+b c+ d
3. a : b is the sub-duplicate ratio of a : b. [componendo and dividendo]
4. a : b 3 is the triplicate ratio of a : b.
3
Ex. 4 The mean proportional of 9 and 16 will be
5. 3
a : 3 b is the sub-triplicate ratio of a : b. (a) 11 (b) 12 (c) 13 (d) 14
6. b : a is the inverse ratio of a : b. Solution (b) Mean Proportional = 9 × 16 = 3 × 4 = 12
7. If a : b and c : d are two ratios, then the compounded
Ex. 5 If A : B = 5 : 4 and B : C = 3 : 2, then A : B : C = ?
ratio is ac : bd.
(a) 12 : 15 : 8 (b) 15 : 12 : 8 (c) 9 : 6 : 5 (d) 8 : 12 : 5
Comparison of Ratios Solution (b) Here, A : B = 5 : 4 and B : C = 3 : 2
There are some rules to compare different ratios. Then, A : B : C = N1N2 : D1N2 : D1D2
a c = 5 × 3 : 4 × 3 : 4 × 2 = 15 : 12 : 8
If the given ratios are and , then
b d Ex. 6 The sum of value of coins of ` 1, 50 paise and
(i) a : b > c : d , if ad > bc 25 paise is ` 210 which are in the ratio 5 : 6 : 8
(ii) a : b < c : d, if ad < bc respectively, then the number of coins of ` 1 is
(iii) a : b = c : d , if ad = bc (a) 168 (b) 105 (c) 100 (d) 63
Solution (b) Let the number of coins of ` 1, 50 paise and 25
Ex. 3 Which of the following is right relation Paise are 5x, 6x and 8x , respectively.
5 9 6x 8x
between and ? ∴ 5x + + = 210
8 14 2 4
5 9 5 9 ⇒ 5x + 3x + 2x = 210 ⇒ x = 21
(a) < (b) >
8 14 8 14 Hence, number of coins of ` 1= 5 × 21 = 105
5 9
(c) = (d) None of these Ex. 7 The sum of three numbers is 315. If the ratio
8 14 between 1st and 2nd is 2 : 3 and the ratio between 2nd
a 5 c 9 and 3rd is 4 : 5, then find the 2nd number.
Solution (a) Let = and =
b 8 d 14 (a) 170 (b) 108
Now, ad = 5 × 14 = 70 and bc = 8 × 9 = 72Q ad < bc; (c) 84 (d) 115
5 9 Solution (b) 1st number : 2nd number
∴ <
8 14 = 2 : 3 = ( 2 × 4) : ( 3 × 4) = 8 : 12
2nd number : 3rd number
Proportion = 4 : 5 = ( 4 × 3) : (5 × 3) = 12 : 15
The equality of two ratios is called proportion. ∴Ist number : 2nd number : 3rd number = 8 :12 :15
12 12
Let a , b , c and d are four quantities, then the proportion ∴2nd number = × 315 = × 315 = 108
8 + 12 + 15 35
are a : b :: c : d.
Ex. 8 The ratio between the present ages of A and B is
Properties of Proportion 4 : 5. If the ratio between their ages 4 yr, hence becomes
Some properties of proportion are as follows. 14 : 17, then what is B’s age at present?
1. In the proportion a : b :: c : d; a and d are extreme (a) 30 yr (b) 28 yr
values and b and c are mean values. (c) 34 yr (d) Data inadequate
Solution (a) Let the present ages of A and B be 4x and 5x yrs,
i.e. Product of extreme = Product of means
respectively.
⇒ a×d =b ×c According to the question,
2. If x is the third proportional to a, b, then a : b :: b : x. 4x + 4 14
∴ =
3. The fourth proportional to a, b and c will be, i.e 5x + 4 17

a : b :: c : d ⇒ a × d = b × c ⇒ d =
bc ⇒ 68x + 68 = 70 x + 56 ⇒ 2x = 12 ⇒ x = 6
a Hence, present age of B = 5 × 6 = 30 yr
4. Mean proportional between a and b is ab.
Ratio and Proportion 25

Practice Exercise
Ratio and Proportion 12 The value of ? in 2 : ? : : ? : 32 is 23. The present ages of Reena and
(a) 64 (b) 34 (c) 30 (d) 8 Usha are 24 and 36 yr, respectively.
1. The ratio of 40 m and 2 km is What was the ratio between the
(a) 1 : 50 (b) 2 : 35 13. Third proportional of 10 and 30 is ages of Usha and Reena
(c) 1 : 40 (d) 3 : 25 (a) 40 (b) 30 (c) 90 (d) 33 respectively 8 yr ago?
2. If x : y = 3 : 1, then the ratio of 14. The fourth proportion of the (a) 7 : 4 (b) 4 : 7
(c) 11 : 8 (d) 8 : 11
x 3 − y 3 : x 3 + y 3 is numbers 12, 16, 18 is
(a) 28 (b) 30 (c) 20 (d) 24 24. At present, Meena is eight times
(a) 12 : 11 (b) 13 : 21
(c) 13 : 14 (d) 21 : 42 15. If 182 is divided in the ratio her daughter’s age. 8 yr from now,
3 : 5 : 4 : 1, then the lowest part is the ratio of the ages of Meena and
3. If two numbers are in the ratio of her daughter will be 10 : 3
5 : 8 and if 9 be added to each, the (a) 28 (b) 15 (c) 14 (d) 7
respectively. What is Meena’s
ratio becomes 8 : 11. The lower 16. If a : b = 3 : 2, b : c = 1 / 4 : 1 / 5, present age?
number is 1 1 (a) 32 yr
(a) 10 (b) 13 (c) 12 (d) 15 c : d = 1 : 3 then, what is a : d ?
2 2 (b) 40 yr
4. If x : y = 3 : 4 , then the value of (a) 45 : 56 (b) 15 : 14 (c) 36 yr
(d) Cannot be determined
5x − 2y (c) 15 : 22 (d) 45 : 67
is 25. At present, Anil is 1.5 times of
7x + 2y 17. If
1 1 1 1
: = : , then the value of Purvi’s age. 8 yr hence, the
7 7 7 7 5 x x 1.25
(a) (b) (c) (d) respective ratio between Anil and
25 23 29 17 x is
Purvi’s ages will be 25 : 18. What is
(a) 1.5 (b) 2
5. Ratio of boys to the girls in a class Purvi’s present age?
(c) 2.5 (d) 3.5
is 5 : 4. Which of the following (a) 50 yr (b) 28 yr
cannot be the number of students a 2 b 4 (c) 42 yr (d) 36 yr
18. If = and = , then
in the class? b 3 c 5 26. The sum of the present ages of
(a) 45 (b) 72 (c) 108 (d) 98 (a + b):(b + c) = ? Varun and Kapil is 42 yr. The ratio
(a) 3 : 4 (b) 4 : 5 of their ages after 5 yr will be
6. The two numbers are in the ratio
(c) 5 : 9 (d) 20 : 27 15 : 11. What is the present age of
2 : 3 and their product is 96. The
sum of the numbers is a 3 b 5 c 7 Kapil?
19. If = , = , = , then
(a) 5 (b) 20 (c) 101 (d) 102 b 4 c 6 d 8 (a) 17 yr (b) 24 yr
a : b : c : d is (c) 25 yr (d) 22 yr
7. The present ratio of ages of A and
(a) 105 : 120 : 135 : 180
B is 4 : 5. 18 yr ago, this ratio was
(b) 105 : 140 : 168 : 192 Previous years’ Questions
11 : 16. The sum of total of their (c) 105 : 140 : 158 : 212 27. The ratio of ages of two persons is
present ages is (d) 115 : 140 : 168 : 182 5 : 9 and the age of one of them is
(a) 90 yr (b) 105 yr (c) 110 yr (d) 80 yr
20. The monthly salary of A , B , C are in greater than the other by 40 yr. The
8. If a : b = c : d , then the value of the ratio of 2 : 3 : 5. If C’s monthly sum of their ages in year is
a2 + b2 salary is ` 1200 more than that of [SSC Constable, 2015]
is (a) 140 (b) 180
c2 + d 2 A, then B’s annual salary is
(c) 150 (d) 160
1 a+ b a−b b 2 (a) ` 14400 (b) ` 24000
(a) (b) (c) (d) (c) ` 1200 (d) ` 2000 28. The mean proportion of 1.21 and
2 c+d c −d d2 0.09 is [SSC Constable, 2015]
21. The ratio of coins of `1, 50 paise
9. If p : q = 3 : 4 and q : r = 8 : 9. Then, (a) 0.33 (b) 3.03
and 25 paise is 4 : 5 : 6. If total cost (c) 3.3 (d) 0.033
p : r is equal to
of all coins is ` 64, then the 29. 22 carat gold is mixed with 18
(a) 1 : 3 (b) 3 : 2 (c) 2 : 3 (d) 1 : 2
number of coins of 25 paise is carat gold in the ratio 1 : 2 to get x
10. Ratio between the monthly (a) 48 (b) 40
carat gold. Then, the value of x is
incomes of A and B is 9 : 8 and the (c) 32 (d) None of these [SSC Constable, 2013]
ratio between their expenditures is 2 1
(a) 20 (b) 15 (c) 14 (d) 19
8 : 7. If they save ` 500 each, then Ages 3 3
A’s monthly income is 22. The ratio of ages of two students is
(a) ` 3500 (b) ` 4000
30. A man has some hens and cows. If
3 : 2. One is older, than the other by the number of heads: number of
(c) ` 4500 (d) ` 5000
5 yr. What is the age of the feet = 12: 35, find out the number of
11 The greatest ratio between 2 : 3 younger student? hens, if the number of heads alone
and 4 : 5 is (a) 2 yr (b) 10 yr is 48. [SSC Constable, 2012]
(a) 2 : 3 (b) 4 : 5 1 (a) 28 (b) 26
(c) 2 yr (d) 15 yr
(c) Cannot be determined 2 (c) 38 (d) 40
(d) Both are equal
26 Self Study Guide SSC Constable (GD) Recruitment Exam

31. Two numbers are such that the 33. The ratio of successful and
ratio between them is 4 : 7. If each unsuccessful examinees in an Answers
is increased by 4, the ratio becomes examination in a school is 6 : 1.
3 : 5. The larger number is The ratio would have been 9 : 1, if 1 a 2 c 3 d 4 c 5 d
[SSC Constable, 2012] 6 more examinees had been 6 b 7 b 8 a 9 d 10 c
(a) 36 (b) 48 (c) 56 (d) 64 successful. The total number of
11 c 12 b 13 c 14 d 15 c
32. If the annual income of A, B and C examinees is [SSC Constable, 2012]
(a) 140 (b) 120 (c) 200 (d) 160 16 a 17 c 18 d 19 b 20 a
are in the ratio 1 : 3 : 7 and the
total annual income of A and C is 21 a 22 b 23 a 24 a 25 b
34. Marks of two candidates P and Q
`800000, then the monthly salary are in the ratio of 2 : 5. If the 26 a 27 a 28 a 29 d 30 b
of B (in `) is [SSC Constable, 2012] marks of P are 120 than marks of 31 c 32 b 33 a 34 c
(a) 20000 (b) 25000 Q are [SSC Constable, 2011]
(c) 30000 (d) 15000 (a) 120 (b) 240 (c) 300 (d) 360

Hints & Solutions


1. (a) Here, we have 40 m a c 15. (c) Required lowest part
8. (d) Let = =k
and 2 km = 2 × 1000 = 2000 m b d
=
1
× 182
So that, ratio of 40 m and 2000 m Then, a = kb and c = kd ( 3 + 5 + 4 + 1)
= 40 : 2000 = 1 : 50 a 2 + b 2 k 2b 2 + b 2 1
∴ 2 = = × 182 = 14
2. (c) Let x = 3k and y = k c + d 2 k 2d 2 + d 2 13
Then, x 3 − y 3 = 27 k 3 − k 3 = 26k 3 b 2( k 2 + 1) b 2 Alternate Method
= 2 2 =
d ( k + 1) d 2 Let the quantities be 3 x, 5 x, 4 x and x,
and x 3 + y 3 = 27 k 3 + k 3 = 28k 3 respectively.
9. (c) Q p : q = 3 : 4 and q : r = 8 : 9
26k 3 Then, 3 x + 5 x + 4 x + 1x = 182
∴ Their ratio = = 13 : 14 p q 3 8 2
28k 3 ∴ p: r = × = × = =2:3 ⇒ 13 x = 182
q r 4 9 3
3. (d) Let the numbers be 5 x and 8 x. ∴ x = 14
10. (c) Let A’s monthly income = ` 9 x
5x + 9 8 Hence, lowest part = x = 14
By given condition, = and B’s monthly income = ` 8 x
8 x + 9 11 a b c 3 1 5 3 2
According to question, 16. (a) × × = × × × ×
⇒ 55 x + 99 = 64 x + 72 b c d 2 4 1 2 7
Ratio between their expenditures = 8 : 7
⇒ 9 x = 27 ⇒ x = 3 9 x − 500 8 a 45
⇒ = ⇒ =
Hence, lower number = 5 x = 5 × 3 = 15 d 56
8 x − 500 7
x 3 17. (c) We have,
4. (c)Q = = k (let) ⇒ x = 3k, y = 4k ⇒ 63 x − 3500 = 64 x − 4000
y 4 1 1 1 1
⇒ x = 500 : = :
5x − 2 y 5( 3k ) − 2( 4k ) 5 x x 125
.
Now, = ∴ A’s monthly income = 9 × 500
5 x + 2 y 5 ( 3k ) + 2( 4k ) 1
×
1
= 2
1
= ` 4500
15k − 8k 7k 7 5 125 . x
= = = a 2
11. (b) Let = and =
c 4
15k + 8k 29k 29 ⇒ x 2 = 5 × 125
.
b 3 d 5
5. (d) The total number of students should be and ad = 2 × 5 = 10 ∴ x= . = 2.5
625
divisible by ( 5 + 4) = 9, a 2 b 4
and bc = 3 × 4 = 12 18. (d) Given, = , =
b 3 c 5
i.e. 98 is the only number, which is not divisible Q ad < bc
⇒ a : b :c = 8 :12 :15
by 9. ∴ a : b < c :d = 2 : 3< 4: 5
6. (b) Let numbers are 2 x and 3x. Let a = 8k, b = 12 k and c = 15k
Hence, 4 : 5 is the greatest ratio.
a+ b 8k + 12 k 20
96 12. (d) Q 2 : ? : : ? : 32 ⇒ (?)2 = 2 × 32 = 64 ∴ = =
∴ 2 x × 3x = 96 ⇒ x 2 = = 16 b + c 12 k + 15k 27
6
∴ ? =8 19. (b) a : b : c : d = N1N2N3 : D1N2N3
⇒ x=4
13. (c) Let third proportional of 10 and 30 is
Hence, required sum = (2 x + 3 x ) = 5 x = 20 b, then
: D1D2N3 : D1D2D3
7. (a) Let the present ages of A and B are 4 x = 3× 5×7 : 4× 5×7 : 4× 6×7: 4× 6× 8
( 30)2 = 10 × b
and 5 x, respectively. = 105 : 140 : 168 : 192
4 x − 18 11 30 × 30
By given condition, = ⇒ b= = 90 20. (a) Let the monthly salary of A, B and C
5 x − 18 16 10
be ` 2 x, ` 3 x and ` 5 x.
⇒ 64 x − 18 × 16 = 55 x − 18 × 11 14. (d) Let x be the fourth proportion of 12,
According to the question,
16 and 18, then
⇒ 9 x = 18(16 − 11) 5 x = 2 x + 1200
16 × 18
⇒ x = 10 x= ⇒ x = 24 ⇒ x = ` 400
12
∴The sum of their present ages So, annual salary of B
= 40 + 50 = 90 yr = 3 × 400 × 12 = ` 14400
Ratio and Proportion 27

21. (a) Let number of coins of ` 1, 50 paise x + 5 15 30. (b) Let the number of heads = 12 K
After 5 yr, =
and 25 paise be 4 x, 5 x and 6 x , respectively. y + 5 11 and the number of feet = 35K
5x 6x ⇒ 11x + 55 = 15 y + 75 Then, 12 K = 48
∴ 4x + + = 64
2 4 ⇒ 11x − 15 y = 20 … (ii) K= 4
⇒ 16 x + 10 x + 6 x = 64 × 4 On solving Eqs. (i) and (ii), we get ∴Number of feet = 35 × 4 = 140
64 × 4 11x + 11y = 462 Again, let the number of hens = x
⇒ x= =8
32 [multiply by 11 in Eq. (i)] and number of cows = y
Hence, number of coins of 25 paise Now, subtracting Eq. (ii) from Eq. (i) we get ∴ x + y = 48 .......(i)
= 6 × 8 = 48 26 y = 442 and 2 x + 4 y = 140 .......(ii)
22. (b) Let the age of students be 3 x yr and 442 ⇒ 2 x + 4 ( 48 − x ) = 140 [from Eq. (i)]
∴ y= = 17 yr
2 x yr, respectively. 26 ⇒ 2 x + 192 − 4 x = 140
∴ 3x − 2 x = 5 ⇒ x = 5 Hence, present age of Kapil = 17 yr ⇒ 2 x = 52
∴The age of younger student = 2 × 5 27. (a) Ratio of age of two persons =5 : 9 ∴ x = 26
= 10 yr Difference between their ages = 40 31. (c) Let the numbers be 4 K and 7 K,
36 − 8 28
23. (a) Required ratio = = =7:4 Let age of first person = 5 x and second person respectively.
24 − 8 16 = 9x 4K + 4 3
Then, =
24. (a) Let age of daughter = x yr According to the question, 7K + 4 5
∴ Age of Meena = 8 x yr 9 x − 5 x = 40 , 4 x = 40 , x = 10 ⇒ 20K + 20 = 21K + 12 ⇒ K =8
According to the question, Age of first person = 5 x = 5 × 10 = 50 ∴ Larger number = 7 × 8 = 56
8 x + 8 10 Age of second persons = 9 x = 9 × 10 = 90 32. (b) Let the annual income of A, B and C
=
x+ 8 3 Sum of ages of both the persons = ( 50 + 90) be ` x, ` 3x and ` 7x, respectively.
= 140
⇒ 24 x + 24 = 10 x + 80 Then, x + 7x = 800000
28. (a) Mean proportion = ab here, a = 1.21
⇒ 14 x = 56 ⇒ x = 100000
and b = 0.09 ∴ B’s monthly salary
∴ x=4
Hence, Meena’s Present age = 4 × 8 = 32 yr So, Mean proportion = 121 . × 0.09 3 × 100000
121 9 = = `25000
25. (b) Let age of Purvi = x yr = × 12
100 100 33. (a) Let the successful and unsuccessful
Then, age of Anil = 15
. x yr
11 × 11 3× 3 11 × 3 33 examinees in an examination be 6x and x,
According to the question, = × = = = 0.33
. x + 8 25
15 10 × 10 10 × 10 100 100 respectively.
= 29. (d) Using the rule of alligation 6x + 6 9
x+ 8 18 Then, =
x−6 1
⇒ 27 x + 144 = 25 x + 200 Gold x Gold y
56 22 18 ⇒ 6 x + 6 = 9 x − 54
∴ x= = 28 ⇒ 3 x = 60 ⇒ x = 20
2 x
Hence, present age of Purvi = 28 yr ∴ Total number of examinees
(x – 18) (22 – x) = 6x + x = 7x
26. (a) Let the present age of Varun be x yr
and present age of Kapil be y yr. x − 18 1 = 7 × 20 = 140
= ⇒ 2 x − 36 = 22 − x 5
According to the question, 22 − x 2 34. (c) Marks of Q = × 120 = 300
2
x + y = 42 … (i) 1
3 x = 58 ⇒ x = 19
Also, 3
28 Self Study Guide SSC Constable (GD) Recruitment Exam

CHAPTER 07

Average
Average Age and Average
If the average age of n persons decreases by x yr. Then, the
An average or arithmetic mean of given data is the sum of
total age of n persons decreased by (n × x) yr. Also, if the
the given observations divided by number of observation.
average age of n persons increases by x yr. Then, the total
If we have to find out the average of 10, 15, 25 and 30, then age of n persons increases by (n × x) yr.
the required average will be
10 + 15 + 25 + 30 80 Ex. 2 The average age of 6 persons is increased by
= = = 20 2 yr. When one of them, whose age is 26 yr is replaced
4 4
by a new man. What is the age of new person?
Therefore, we can write the formula (a) 26 yr (b) 36 yr (c) 38 yr (d) 40 yr
Sum of given observations ( S) Solution (c) Total age increased = 6 × 2 = 12 yr
Average (A) =
Number of observations ( N) ∴Age of new person = (26 + 12) = 38 yr
S The increase in the total age of 6 persons is due to the
or A=
N replacement of a person aged 26 yr.
With a person who is 12 yr older to him.
Ex. 1 If the age of 4 students are 20 yr,
22 yr, 18 yr and 24 yr, then what is the average age of Ex. 3 The average age of 24 boys and a teacher is
the students? 15 yr. When the teachers age is excluded, the average
(a) 20 yr (b) 21 yr (c) 22 yr (d) 23 yr decreased by 1 what is the age of teacher?
20 + 22 + 18 + 24 (a) 40 yr (b) 39 yr (c) 41 yr (d) 43 yr
Solution (b) Average age =
4 Solution (b) Total original age of 25 members
84 = 15 × 25 = 375 yr
= = 21yr
4 Total new age of 24 members = 14 × 24 = 336 yr
∴ Age of teacher = (375 – 336) = 39 yr
Weighted Average
Average of Some Important Series of Number
The concept of weighted average is used when we have
two or more groups whose individual averages are known. The average of odd numbers 1 to n is,
Last odd number +1
Suppose in a class, there are 2 students of 20 yr, = ,
3 of 21 yr, 4 of 22 yr, and 5 of 23 yr, then their average age 2
in given by where n = last odd number
(2 × 20) + (3 × 21) + (4 × 22) + (5 × 23) The average of even number from 2 to n is
2+ 3+ 4+ 5
last even number +2
2 3 4 5 306 = , where n = last even number
= × 20 + × 21 + × 22 + × 23 = yr 2
14 14 14 14 14
2 3 4 5 Ex. 4 Find the average of all the odd numbers and
Here, , , and are called the weights of each
14 14 14 14 average of all the even number from 1 to 45.
category of students. (a) 22, 23 (b) 23, 23 (c) 24, 23 (d) 25, 24
45 + 1 46
In other words, weights are the fractions of the numbers in Solution (b) Average of 1 to 45 odd number = =
2 2
that category with respect to the total students in that
= 23
class. This average is also called the weighted average of 44 + 2 46
that class. and Average of 2 to 44 even number = = = 23
2 2
Average 29

Practice Exercise
1. Find the average of the set of scores 10. The average of 13 results is 60. If 19. Three years ago, the average age of
566, 455, 231, 678, 989, 342, 715 the average of first 7 results is 59 a family of 5 members was 17 yr. A
(a) 590 (b) 555 (c) 568 (d) 513 and that of last 7 results is 61, baby having been born, the
2. The average of two numbers is M. what will be the seventh result? average age of the family is the
If one number is N, then other (a) 90 (b) 50 (c) 75 (d) 60 same today. The present age of the
number is 11. The average of 3 numbers is 60. Ist baby is
(a) 2 yr (b) 2.4 yr (c) 3 yr (d) 1.5 yr
(a) 2 N (b) 2 M number is double the 2nd number
(c) M – N (d) 2 M – N while the 2nd number is double 20. Five years ago, the average age of P
3. The average of marks of a students the 3rd number. Find out the 1st and Q was 15 yr. Average age of P,
in 7 subjects is 75. His average in 6 number Q and R today is 20 yr. How old will
subjects excluding science is 72. 820 620 720 920 R be after 10 yr?
(a) (b) (c) (d)
How many marks did he get in 8 7 7 9 (a) 35 yr (b) 40 yr (c) 30 yr (d) 50 yr
science? 12. If 48m + 48n = 2880, what is the 21. The average weight of 3 men A, B
(a) 72 (b) 90 average of m and n? and C is 84 kg. Another man D
(c) 93 (d) None of these (a) 90 (b) 45 (c) 35 (d) 30 joins the group and the average
4. The average age of 30 students of a now becomes 80 kg. If another
13. The average income of employees man E, whose weight is 3 kg more
class is 12 yr. The average age of a in an office is ` 150 per month. The
group of 5 of the students is 10 yr than that of D, replaces A, then
average income of officers is ` 450 the average weight of B, C, D and E
and that of another group of 5 of and that of non-officers is ` 80. If
them is 14 yr. What is the average becomes 79 kg. The weight of A is
the number of officers is 14 what is (a) 70 kg (b) 72 kg (c) 75 kg (d) 80 kg
age of the remaining students? the number of non-officers?
(a) 8 yr (b) 10 yr (c) 12 yr (d) 14 yr (a) 65 (b) 55 (c) 60 (d) 70 22. The average age of 5 members of a
5. The average of 4 consecutive even family is 25 yr. If the servant of the
14. The average of 50 numbers is 38. If family is included, the average age
numbers is 27. What is the greatest two numbers, namely 45 and 55
number? increases by 40%. What is the age
are discared, the average of of servant?
(a) 28 (b) 26 (c) 32 (d) 30 remaining numbers is (a) 85 yr (b) 90 yr
6. The average weight of 9 mangoes (a) 36.5 (b) 37 (c) 37.5 (d) 37.52 (c) 73 yr (d) 55 yr
increases by 20 g, if one of them 15. A motorist travels to a place 150
weighing 120 g is replaced by 23. The average of 1, 3, 5, 7, 9, 11,
km away at an average speed of 50 ………, 25 terms is
another. The weight of new mango km/h and returns at 30 km/h. His [SSC Constable, 2015]
is average speed for the whole (a) 625 (b) 25 (c) 125 (d) 50
(a) 180 g (b) 200 g journey in km/h is
(c) 260 g (d) 300 g 24. A batsman makes a score of 87
(a) 35 (b) 37 (c) 37.5 (d) 40
runs in the 17th inning and thus
7. The average age of boys is twice 16. The average age of 30 students is 9 increased his average by 3. Find his
the number of girls in a class. If yr. If the age of their teacher is average after 17th inning.
the ratio of boys and girls in the included, it becomes 10 yr. The age [SSC Constable, 2015]
class of 36 is 5 : 1, what is the total of the teacher (in yr) is (a) 84 (b) 87 (c) 90 (d) 39
ages of the boys in class (in yr)? (a) 27 (b) 31 (c) 35 (d) 40 25. Average income of A and B is ` 200
(a) 490 (b) 196
(c) 420 (d) 360 17. The average weight of 29 students and average income of C and D is
is 28 kg. By the admission of a new ` 250. The average income of A , B ,C
8. The average age of a class is 15.8 student, the average weight is and D is [SSC Constable, 2015]
yr. The average age of the boys in reduced to 27.8 kg. The weight of (a) ` 200 (b) ` 10625
.
the class is 16.4 yr. while that of the new student is (c) ` 125 (d) ` 225
the girls is 15.4 yr. What is the (a) 22 kg (b) 21.6 kg
ratio of boys to girls in the class? 26. The average age of a class of
(c) 22.4 kg (d) 21 kg 20 boys is 12 yr. What will be the
(a) 1 : 2 (b) 3 : 4
(c) 3 : 5 (d) None of these 18. A team of 8 persons joins in a average age of the class, if 5 new
shooting competition. The best boys join the class whose average
9. The average of 100 observations marksman scored 85 points. If he age is 7 yr? [SSC Constable, 2013]
was calculated as 40. It was found had scored 92 points, the average (a) 10 yr (b) 11 yr
later on that one of the observations score for the team would have (c) 10.5 yr (d) 11.5 yr
was misread as 83 instead of 53. been 84. The number of points, the
The correct average is 27. The average of the first 9 positive
team scored was even numbers is
(a) 39 (b) 39.7
(a) 672 (b) 665 [SSC Constable, 2013]
(c) 40.3 (d) 42.7 (a) 12 (b) 9 (c) 10 (d) 10.5
(c) 645 (d) 588
30 Self Study Guide SSC Constable (GD) Recruitment Exam

28. The average of 7 consecutive 30. Out of four numbers, the average 32. If average of 20 observations
numbers is 20. The largest of these of the first three is 18 and that of x1 , x 2 , ..... , x 20 is y, then the average
numbers is [SSC Constable, 2012] the last three is 16. If the last of x1 −101, x 2 −101,
(a) 24 (b) 23 number is 19, the first is x 3 −101, K , x 20 − 101 is
(c) 22 (d) 20 [SSC Constable, 2012]
(a) 19 (b) 18 (c) 20 (d) 25 [SSC Constable, 2011]
29. A man bought 13 articles at ` 70 (a) y − 20 (b) y − 101
each, 15 at ` 60 each and 12 at ` 65 31. B was born when A was 4 yr 7 (c) 20 y (d) 101y
each. The average price per article months old and C was born when
is [SSC Constable, 2012] B was 3 yr 4 months old. When C Answers
(a) ` 60.25 was 5 yr 2 months old, then their 1 c 2 d 3 c 4 c 5 d
(b) ` 64.75 average age was 6 d 7 d 8 d 9 b 10 d
(c) ` 65.75 [SSC Constable, 2012] 11 c 12 d 13 c 14 c 15 c
(a) 8 yr 9 months 16 d 17 a 18 b 19 a 20 c
(d) ` 62.25 21 c 22 a 23 b 24 d 25 d
(b) 7 yr 3 months
26 b 27 c 28 b 29 b 30 d
(c) 8 yr 7 months
31 d 32 b
(d) 8 yr 11 months

Hints & Solutions


1. (c) Required average whose weight is 180 g more than the mango 180
x=
replaced. Therefore, weight of new mango 7
566 + 455 + 231 + 678 + 989 + 342 + 715 = (120 + 180) = 300 g. ∴First number, 4 x =
180
×4=
720
=
7 7. (d) Number of boys = 30 7 7
=
3976
= 568 and number of girls = 6 12. (d) We have, 48m + 48n = 2880
7 Average age of boys = 12 yr ⇒ m+ n =
2880
= 60
N+ x Hence, total age of boys = 12 × 30 = 360 yr 48
2. (d) Given, M = , where x is the other
2 Boys Girls m + n 60
8. (d) ∴Average of m and n = = = 30
number. 16.4 15.4 2 2
Then, x = 2 M – N 13. (c) Let number of non-officers = x
15.8
3. (c) Average Then, 80 x + 450 × 14 = 150 (14 + x )
Sum of numbers in all subjects ⇒ 8 x + 45 × 14 = 15 (14 + x )
= 0.4 0.6
Number of subjects ⇒ 8 x + 630 = 210 + 15 x
Sum of numbers in 7 subjects = 75 × 7 = 525 Required ratio = 2 : 3 ⇒ 7 x = 630 – 210
Sum of numbers in 6 subjects = 72 × 6 = 432 9. (b) Total of 100 observations = 4000 420
⇒ 7 x = 420 ∴ x = = 60
∴Marks in science = ( 525 – 432 ) = 93 Correct total of 100 observations 7
4. (c) Total age of 30 students = ( 4000 – 83 + 53) = 3970 14. (c) Average of remaining numbers
3970
. ( 38 × 50) – ( 45 + 55) 1800
= 30 × 12 = 360 yr Correct average = = 397. = = = 37.5
100
Total age of 10 students = (10 × 5 + 14 × 5) 48 48
= 120 yr 10. (d) According to the fundamental formula, Total distance
S
15. (c) Average speed =
Total age of remaining 20 students A= Total time
= ( 360 – 120) = 240 yr N 300
=
240 [A = Average, S = Sum, N = Number] 150 150
Average age of 20 students = = 12 yr +
20 From the question, 30 50
S 300
5. (d) Series of consecutive even numbers 60 = = = 37.5 km/h
goes like x, ( x + 2 ), ( x + 4), ( x + 6),..... and so 13 8
on. ∴ S = 60 × 13 = 780 16. (d) Total age of 30 students
According to the formula, Sum of first seven results = 59 × 7 = 413 = 30 × 9 = 270 yr
Average of consecutive numbers Sum of last seven results = 61 × 7 = 427 Total age of 31 persons including one teacher
First number + Last number ∴7th results = 31 × 10 = 310 yr
=
2 = Sum of first seven results + Sum of last ∴Age of teacher = ( 310 – 270)
x + ( x + 6) seven results – Sum of all the results = 40 yr
⇒ 27 = ⇒ x + 3 = 27
2 = ( 413 + 427 – 780) = ( 840 – 780) = 60 17. (a) Total weight of 29 students
∴ x = 24 11. (c) Sum of three numbers = 60 × 3 = 180 = 29 × 28 = 812 kg
∴Greatest number = 4th Consecutive even Let 3rd number = x Total weight of 30 students
number ∴ 2nd number = 2 x = 30 × 27.8 = 834 kg
= ( x + 6) = 24 + 6 = 30 and 1st number = 2 × 2 x = 4 x ∴Weight of new student = ( 834 – 812 ) = 22 kg
6. (d) Total increase in weight of 9 mangoes Now, according to the question, 18. (b) Let the number of points scored by 7
(20 × 9) = 180 g. This increase in the weight is 4 x + 2 x + x = 180 persons be x.
due to replacement of a mango with a new
Average 31

x + 92 = 25 × 25 = 625 ⇒ 7 x = 119
New average = = 84
8 625 119
Average = = 25 ⇒ x= = 17
∴ x = 580 25 7
Number of points scored by the team was 24. (d) A batsman makes a score of 87 runs ∴ Largest number = x + 6 = 17 + 6 = 23
= ( 580 + 85) = 665 in 17th inning and his average increases by 3. 29. (b) The average price per article
19. (a) Total age of family three years ago Score of batsman before 17th inning = x 13 × 70 + 15 × 60 + 12 × 65
=
= 17 × 5 = 85 yr Score of batsman after 17th inning = x + 87 13 + 15 + 12
Total age of family today = 85 + 5 × 3 = 100 yr x + 87 x 910 + 900 + 780 2590
= − =3 = = = ` 6475
.
100 + x 17 16 40 40
Let the age of the baby be x yr = = 17
6 16 ( x + 87 ) − 17 x = 3 × 16 × 17
30. (d)Let the numbers be x1, x2, x3 and x4 ,
⇒ x = 2 yr 16 x + 87 × 16 − 17 x = 3 × 16 × 17 ,
respectively.
20. (c) Total age of P and Q today 87 × 16 − 3 × 16 × 17 = x
Then, x1 + x2 + x3 = 18 × 3 = 54 ...(i)
= (15 × 2 + 5 × 2 ) = 40 yr 16 ( 87 − 51) = x , x = 16 × 36 = 576
and x2 + x3 + x4 = 16 × 3 = 48 ...(ii)
Total age of P, Q and R today Now, score of batsman after 17th inning
On subtracting Eq. (ii) from Eq. (i), we get
= 20 × 3 = 60 yr = 576 + 87 = 663
663 x1 − x4 = 54 − 48
Therefore, present age of R Now, average after 17th inning = = 39 ⇒ x1 − x4 = 6
= 60 – 40 = 20 yr 17
25. (d) Average income of A and B is ` 200 Q x4 = 19
Age of R after 10 yr ∴ x1 = 19 + 6 = 25
A+ B
= 20 + 10 = 30 yr = 200
2 31. (d) A’s, present age
21. (c) Total weight of A, B, and C = (4 + 3 + 5) yr and (7 +4 + 2) months
A + B = 400 …(i)
= 84 × 3 = 252 kg = 12 yr and 13 months
Average Income of C and D is ` 250
Total weight of A, B, C and D = 13 yr and 1 month
C+D
= 80 × 4 = 320 kg = 250
2 B’s present age = (3 + 5) yr
Weight of D = ( 320 – 252 ) = 68 kg
C + D = 500 …(ii) and (4 + 2) months
Weight of E = 68 + 3 = 71 kg = 8 yr and 6 months
From Eqs. (i) and (ii), we get
Total weight of B, C, D and E = 79 × 4 = 316 kg C’s present age = 5 yr and 2 monts
A + B + C + D = 400 + 500 = 900
Total weight of B, C, D = ( 316 – 71) = 245 kg ( A + B + C )’ s age
Now average income of A, B, C and D is ∴ Required average =
∴Weight of A = ( 320 – 245) = 75 kg A+ B+ C + D 3
22. (a) Total age of 5 members =
4 (13 + 8 + 5) yr and (1 + 6 + 2) months
= 25 × 5 = 125 yr 900 =
= = ` 225 3
Total age of 5 members and the servant 4
26 × 12 months + 9 months
 40  20 × 12 + 5 × 7 =
=  25 + 25 × ×6 26. (b) Required average = 3
 100  20 + 5
= 35 × 6 = 210 yr 240 + 35 275 321
= = = 11 yr = months
∴Age of the servant = 210 – 125 = 85 yr 25 25 3
23. (b) Here a = 1, d = 2, n = 25 27. (c) The average of the first n positive = 107 months
Tn = a + ( n − 1) d = 1 + (25 − 1) × 2 = 1 + 24 × 2 even number = n + 1 = 8 yr and 11 months
= 49 So, the required average = x + 1 32. (b) Required average
Since, the nth term is 49, so l = 49 = 9 + 1 = 10 x + x2 + K + x20 101 × 20
= 1 −
n 25 28. (b) Suppose, first number = x 20 20
S n = [a + l ] = [1 + 49]
2 2 Then, x + x + 1 + x + 2 + x + 3 + x = y − 101
25 + 4 + x + 5 + x + 6 = 20 × 7
= × 50
2 ⇒ 7 x + 21 = 140
32 Self Study Guide SSC Constable (GD) Recruitment Exam

CHAPTER 8

Percentage
Per cent Solution (b) Original price = `300
The meaning of the term per cent is ‘for every hundred’. It  20 
∴ New price = 300 1 +  = ` 360
can be defined as  100 

‘‘A per cent is a fraction whose denominator is 100 and the Ex. 4 The wages of a worker is first increased by 20%
numerator of the fraction is called the rate per cent.’’ and subsequently reduced by 20%. If the original
Per cent is denoted by the sign ‘%’. monthly wages is ` 500. What is the final wages?
(a) `500 (b) `450 (c) `480 (d) `640
Formula to Calculate Per cent
Solution (c) Final wages = 500 1 +
20   20 
 1 − 
 100   100 
It we have to find y% of x, then y% of
6 4
y = 500 × × = ` 480
x=x× 5 5
100
Successive increase/decrease
Ex. 1 Quantity of water in milk constitutes 6 parts of
every 20 parts of mixture. What is the percentage of All successive changes in percentage (increase or decrease)
water in the mixture? can be represented as a single percentage which is given by
(a) 30% (b) 60% (c) 40% (d) 25%  ab 
Solution (a) Percentage of water in the mixture a + b + 100 %, where a and b shows the first and second
 
6
= × 100 = 30% percentage changes.
20

T Note To convert any fraction a/b to rate percentage, multiply it T Note In case of increment we take positive values and in case
by 100 and % sign. Alternatively to convert a rate per cent to a of decrement we take negative values.
fraction, divide it by 100 and remove the % sign.
Ex. 5 The length of a rectangle is increased by 20%
Percentage Increase/ Percentage Decrease and breadth is decreased by 10%. Calculate the
percentage change in the area.
Percentage increase/decrease in calculated with respect to (a) 6% (b) 7% (c) 12% (d) 8%
the base (Previous) value unless mentioned otherwise.
Solution (d) Percentage change in area
Increase / Decrease  ab   200 
Percentage increase/decrease = × 100 = a + b +  % = 20 − 10 −  % = 8%
Base value  100   100 
Ex. 2 If A’s income is 20% more than that of B, then Example 6 If the price of a commodity is increased by
how much per cent is B’s income less than that of A ? 20%, find by how much per cent must a house wife
2 reduce her consumption, so as not to increase the
(a) 16 % (b) 20% (c) 25% (d) 16%
3 expenditure?
2 1
Solution (a) Let B’s income be `100. (a) 20% (b) 25% (c) 16 % (d) 33 %
3 3
(100 + 20)
∴ A’s income = × 100 = `120 Sol. (c) Let the original price of the commodity be ` 100
100
120 − 100 2 ∴ Increased price of commodity
∴ Required percentage = × 100 =16 % 100 + 20 
120 3 =  × 100 = `120
 100 
Ex. 3 The price of a product is increased by 20%. If Now, reduction in consumption for same expenditure
the original price is ` 300. What is the final price of the = ` (120 − 100) = `20
product? 20 2
Required percentage = × 100 = 16 %
(a) `400 (b) `360 (c) `420 (d) `170 120 3
Practice Exercise
1. If 20% of x = 90, find x. 10. The price of a television includes 17. Two numbers are 20% and 25% less
(a) 350 (b) 450 (c) 250 (d) 550 the manufacturing cost, 10% sales than the third number. How much
tax and 10% profit. What is the percentage is the second number
3
2. What is 32% of th of 5000? manufacturing cost, if the price is less than the first?
8 `14400? (Sales tax and profit are to 4 20
(a) 5 % (b) %
(a) 606 (b) 610 (c) 580 (d) 600 be calculated on manufacturing 5 3
3. P’s income has increased by 10% costs). 1 1
(c) 6 % (d) 6 %
over last year and will be 20% (a) ` 10000 (b) ` 12000 2 4
higher next year. If his last year’s (c) ` 12500 (d) ` 9000 18. Due to increase of 30% in the price
income is ` 7000 per month what 11. A class has girls and boys in the of a colour TV the sale is reduced
will it be next year? ratio 4 : 5. Among the girls, the by 40%, then what will be the
(a) ` 9240 (b) ` 7500 ratio of Mathematics to Physics percentage change in income?
(c) ` 9000 (d) ` 7850 students is 3 : 1. If the ratio of (a) 10%, increase
4. A mixture contains sand and Mathematics and Physics students (b) 10%, decrease
in the entire class is 3 : 2. What (c) 35%, decrease
gravel in the ratio 1 : 4. The sand
percentage of class comprises girls (d) 22%, decrease
has 20% impurity. If total mixture
weights 10 kg, how much impurity studying Mathematics? 19. Two numbers are respectively 20%
is there? (Consider that gravel has (a) 33.3% (b) 30% and 50% more than a third
(c) 25% (d) 18% number. What is the percentage of
no impurity).
(a) 300 g (b) 450 g 12. Asha’s income is `12000 per second with respect to first?
(c) 400 g (d) 500 g month. She pays 20% tax on (a) 125% (b) 90%
monthly income above ` 2000 and (c) 64% (d) 75%
5. The population of a city is
estimated to be 432000 after 2 yr. If she spends 30% of the remaining 20. From 2008 to 2009, the sales of a
income. How much does she save book decreased by 80%. If the sales
the population growth is 20% per
annually. in 2010 was the same as in 2008,
annum. What is the current
population? (a) ` 70000 by what percent did it increase
(a) 310000 (b) 285000 (b) ` 60000 from 2009 to 2010?
(c) 380000 (d) 300000 (c) ` 84000 (a) 80% (b) 100%
(d) ` 50000 (c) 120% (d) 400%
6. The price of a house increased by
13. A single discount which is 21. Anand spends 75% of his income.
25% after 10 yr, reduces by 25%
equivalent to successive discounts His income in increased by 20%
during the subsequent 10 yr. If the
25% and 10% is and he increased his expenditure
present cost is `10 lakh, what will
be its cost after (a) 35% (b) 34.5% by 10%. His savings are increased
(c) 33% (d) 32.5% by
20 yr?
14. A reduction of ` 2 per kg enables a (a) 10% (b) 25%
(a) ` 937500 (b) ` 900000
1
(c) ` 850000 (d) ` 725000 man to purchase 4 kg more sugar (c) 37 % (d) 50%
2
7. The price of a product of a company for `16. Find the original price of
increases by 10% and the turnover sugar. 22. In an examination, 35% candidates
increases by 10%. What is the (a) ` 4 per kg failed in one subject and 42%
change in quantity sold? (b) ` 6 per kg failed in another subject while 15%
(a) 10% (b) 8% (c) ` 3 per kg failed in both the subjects. If 2500
(c) 20% (d) No change (d) ` 4.5 per kg candidates appeared at the
15. A reduction of 25% in the price of examination, how many passed in
8. The monthly income of a person is
rice, enables a person to buy 5 kg either subject but not in both?
` 5000. If his income is increased
more for `120 . Find the original (a) 325 (b) 1175
by 30%, what is his monthly
price of rice per kg. (c) 2125 (d) None of these
income now?
(a) ` 7000 (b) ` 5500 (a) ` 8 (b) ` 10 23. In an election between two
(c) ` 4500 (d) ` 6500 (c) ` 12 (d) ` 15 candidates, a candidate who got
16. A litre of water evaporates from 6 L 40% of total votes is defeated by
9. The price of a certain article is
of sea water containing 4% salt. 15000 votes. The number of votes
`15000 but due to slump in the
Find the percentage of salt in the received by the winning
market, its price decreases by 8%.
remaining solution candidates is
Find the new price of the article.
1 1 (a) 6000
(a) ` 14000 (b) ` 13800 (a) 5 % (b) 3 %
2 2 (b) 10000
(c) ` 16500 (d) ` 12600
4 (c) 22500
(c) 3% (d) 4 % (d) 45000
5
34 Self Study Guide SSC Constable (GD) Recruitment Exam

24. The price of sugar increases by travel, 10% on clothes and the yz zx
(a) % of a (b) % of a
20%. By what per cent must a remaining amount of ` 22500 is x y
housewife reduce the saved. What is Anish’s salary? xy y
(c) % of a (d) % of a
consumption of sugar, so that the (a) ` 40000 (b) ` 40500 z z
expenditure on sugar is the same (c) ` 45000 (d) ` 50000 34. The value of a machine depreciates
as before? 29. Madan’s salary is 25% of Ram’s by 5% every year. If its present
2 value is ` 200000, its value after
(a) 16 % (b) 20% salary and Ram’s salary is 40% of
3 Sudin’s salary. If the total salary of 2 yr will be [SSC Constable, 2012]
1
(c) 80% (d) 83 % all the three for a month is `12000. (a) ` 180500
3
How much did Madan earn that (b) ` 199000
25. The salary of an employee of a month? (c) ` 180000
company increases every month by (a) ` 800 (b) ` 8000 (d) ` 210000
4%. If his salary in August was (c) ` 600 (d) ` 850 35. If y % of 1 h is 1 min 12 s, then y is
` 6300, what would be his equal to [SSC Constable, 2012]
approximate salary in month of Previous Years’ Questions (a) 2 (b) 1
October of the same year? 30. If the cost of an article is ` P after 1 1
(c) (d)
(a) ` 6552 (b) ` 6967 two successive reductions of 20% 2 4
(c) ` 6814 (d) ` 6627 36. A number, on subtracting 15 from
and 25%, the original price of the
26. Bina’s monthly income is 90% of article was [SSC Constable, 2015] it, reduces to its 80%. What is 40%
Anita’s monthly incomes. The total 4P 5P of the number?
(a) ` (b) `
of both their monthly income is 5 4 [SSC Constable, 2012]
equal to Mr Sen’s monthly income. 5P 3P (a) 75 (b) 60
(c) ` (d) `
Mr Sen’s annual income is 3 5 (c) 30 (d) 90
` 775200. What is Bina’s monthly 31. In a college election, a candidate 37. One-third of 1206 is what per cent
income? secured 62% of the votes and is of 134? [SSC Constable, 2011]
(a) ` 34000 (b) ` 36000 elected by a margin of 144 votes. (a) 100 (b) 150
(c) ` 30600 (d) ` 30000 The total number of votes polled is (c) 200 (d) 300
27. In a test, Rajesh got 112 marks [SSC Constable, 2015]
which was 32 more than the (a) 600 (b) 800
(c) 1200 (d) 925 Answers
passing marks. Sonal got 75%
marks which was 70 more than the 32. If 90% of x is 315 km, then the 1 b 2 d 3 a 4 c 5 d
passing marks. What is the value of x is [SSC Constable, 2013] 6 a 7 d 8 d 9 b 10 b
minimum passing percentage of (a) 325 km (b) 350 km 11 a 12 c 13 d 14 a 15 a
the test? (c) 405 km (d) 340 km 16 d 17 d 18 d 19 a 20 d
(a) 35% (b) 45% 33. If x% of a is the same as y% of b, 21 d 22 b 23 d 24 a 25 c
(c) 40% (d) 48% 26 c 27 c 28 d 29 a 30 c
then z% of b will be
28. Anish spends 25% of his salary on [SSC Constable, 2012] 31 a 32 b 33 b 34 a 35 a
house rent, 5% on food, 15% on 36 a 37 d

Hints & Solutions


1. (b) 20% of x = 90 4. (c) Quantity of sand in the mixture 6. (a) Single percentage change
1  (25) × ( − 25)
⇒ x×
20
= 90 = × 10 = 2 kg =  25 − 25 +  % = − 6.25%
100 5  100 
⇒ x = 90 × 5 = 450 20
Impurity in sand = 2 × = 0.4 kg = 400 g Price of house after 20 yr
3 32 3 100
2. (d) 32% of th of 5000 = × × 5000  6.25 
8 100 8 5. (d) Let the current population be P. = 1000000 ×  1 − 
 100 
= 4 × 3 × 50 = 600 According to the question, = 1000000 × 0.9375 = ` 937500
3. (a) Single percentage change  20   20  7. (d) Let the price per piece and quantity in
P  1+   1+  = 432000
 ab   100   100 
= a + b + % piece be ` 10 and 10 respectively.
 100 
 120   120  Then, original turnover = 10 × 10 = ` 100
 10 × 20  ⇒ P   = 432000
=  10 + 20 +  % = 32%  100   100  [Q Turnover = Price × Quantity]
 100 
432000 × 100 × 100 Now, new turnover = ` 110 and new price = ` 11
∴Next year income ⇒ P=
120 × 120 Turnover 110
 32  Then, Quantity = = = 10
= 7000  1 +  = 7000 × 132
. = ` 9240
 100  = 300000 Price 11
Hence, there is no change in the quantity sold.
Percentage 35

8. (d) New income = 130% of 5000 25 22. (b)Failed in first subject


∴ × 120 = 5
130 100 35
= × 5000 = ` 6500 = 2500 × = 875
100 Now cost of 5 kg rice = ` 30 100
9. (b) New price = 92% of 15000 or New cost of 1 kg rice = ` 6 42
Failed in second subject = 2500 × = 1050
92 Let original cost be ` x per kg 100
= × 15000 = ` 13800  25  15
100 ∴  1−  x= 6 Failed in both the subjects = 2500 × = 375
 100  100
10. (b) Since, sales tax and profit are to be
6 × 100 Failed in first subject only = Passed in second
calculated on manufacturing cost, therefore x= = ` 8 per kg
subject only
single percentage change = 20% 75
16. (d) Quantity of salt in 6 L of sea water = ( 875 − 375) = 500
Now, manufacturing cost × 1.2 = 14400
14400 6×4 Failed in second subject only
∴ Manufacturing cost = = ` 12000 = = 0.24 L
1.2 100 = Passed in first subject only
Percentage of salt in 5 L of sea water = (1050 − 375) = 675
11. (a) Let girls and boys in the class 4 x and
.
024 4 ∴Passed in either subjects but not in both
5 x respectively. = × 100 = 4 %
5 5 = ( 500 + 675) = 1175
∴Total students = 4 x + 5 x = 9 x
3 17. (d) Let the third number be 100, then first 23. (d) Percentage votes of Ist candidate
Girls students of Mathematics = × 4 x = 3 x number and second number would be 80 and = 40%
4
1 75 respectively. ∴percentage votes of IInd candidate = 60%
Girls students of Physics = × 4 x = x Second number is 5 less than the first number. Let the total number of voters be `‘x’
4
Less (decrease) 60 40
Percentage of girls studying Mathematics Percentage less = × 100 ∴ x− x = 15000
3x Base value 100 100
= × 100 = 33 . 3% 5 1 20
9x = × 100 = 6 % ⇒ x = 15000
20 80 4 100
12. (c) Tax paid = (12000 − 2000) × 18. (d) Method I Single percentage change ∴ x = 75000
100
= ` 2000  ab  ∴ number of votes of winning candidate
= a + b + %
Amount after tax paid  100  =
60
× 75000 = 45000
= (12000 − 2000) = ` 10000 ( 30) × ( − 40) 100
= 30 − 40 +
30 100 24. (a) Let the original price of sugar be ` x
Other expenses = 10000 × = ` 3000
100 = − 10 − 12 = − 22% per kg. After increase the price of sugar
Saving = 10000 − 3000 = ` 7000 ∴ Income will decrease by 22%. becomes ` 120. x per kg.
Annual saving= 12 × 7000 = ` 84000 Method II Since, the expenditure of sugar should not
ab Let the price per TV and sale of TV be ` 10 and increase person has to spend ` x. Hence, the
13. (d) Equivalent percentage = a + b +
100 10 piece respectively. quantity of sugar bought for ` x after increases
Here, a = − 25% b = − 10% Then, income = 10 × 10 = ` 100 =
1
× x=
1 5
= kg
(for discount we take negative values) New price = ` 13, New sale = 6 piece 1.20 x 1.20 6
25 × 10 New income = 13 × 6 = ` 78 Percentage reduction in consumption
∴ Required percentage = − 25 − 10 + Change income = (78 − 100) = − 22%
100 5
1−
(−ve shows the decrease) 6 × 100 = 50 % = 16 2 %
= − 35 + 23 = − 32. 5% =
∴ Single discount = 32. 5%
19. (a) III II I 1 3 3
14. (a) Let the original price of sugar be ` x 100 150 120 25. (c) Required salary in October
150  104 104 
per kg. ∴ Required percentage = × 100% = 125% = `  6300 × × 
16
120  100 100 
Then, quantity purchased for ` 16 = kg 20. (d) Let sale in 2008 = 100
x = ` ( 63 × 104 × 104. )
After reduction of ` 2 per kg, quantity Sale in 2009 = 20 = ` 6814.08 ≈ ` 6814
16 Sale in 2010 = 100 26. (c) Mr Sen’s monthly income
purchased for ` 16 = kg
(x − 2) ∴ Required increase per cent = (Bina + Anita’s) monthly income
16 16 100 − 20 775200
From the given condition, = + 4 = × 100% = 400% = = ` 64600
(x − 2) x 20 12
16 16 21. (d) Method I Let the income of Anand According to the question,
⇒ − =4 be ` 100.
(x − 2) x Bina = 90% of Anita
He spends = ` 75, savings = ` 25 Bina 90 9
 1 1 ⇒ = =
⇒ 4 − =1 Income after increase = ` 120 Anita 100 10
( x − 2 ) x  Expenses after increase = ` 82. 50 ⇒ Bina : Anita = 9 : 10
x − x + 2 Savings = ` 37. 50 9
⇒ 4 2 =1 ∴ Bina’s salary = × 64600 = ` 30600
 x − 2x  Increase in saving = 37.50 − 25 = ` 12 . 50 19
⇒ x2 − 2 x − 8 = 0 Increase percentage =
Increase
× 100
27. (c) Let maximum marks be x.
Base value According to the question,
∴ x = ` 4 per kg [Q x ≠ − 2 ]
12 . 50 75 x
15. (a)Q After reduction of 25% the man is = × 100 = 50% − 70 = 112 − 32
25 100
able to purchase 5 kg more rice for `120
36 Self Study Guide SSC Constable (GD) Recruitment Exam

75 x 5x − x 4x ⇒ ax = by …(i)
⇒ = 80 + 70 ⇒ ⇒
100 5 5 z×b
∴ z %of b =

75 x
= 150 ∴ x = 200
4 x 25  4x 100
Second reduction of 25% = − of  
100 5 100  5  From Eq. (i),we get
Passing marks = 112 − 32 = 80 4x 1  4x ax z xz
⇒ − ×  z % of b = × = % of a
(As Rajesh got 112 marks which is 32 more 5 4  5 y 100 y
than the passing marks) 4x x 3x 34. (a) The cost price of machine after 2 yr
∴Minimum passing marks percentage ⇒ − ⇒ 2
5 5 5  5 
80 = 200000 ×  1 − 
= × 100% = 40% According to the question after two successive  100 
200 reduction the cost of article is P 19 19
28. (d) Remaining salary per cent 3x = 200000 × × = ` 180500
So, P= 20 20
= [100 − (25 + 5 + 15 + 10)]% 5
35. (a)Q 1 h = 60 min or 3600 s
= [100 − 55]% = 45% 5
Therefore, original price ( x ) = ` P and 1 min + 12 s = (60 + 12) s = 72 s
According to the question, 45% of x = 22500 3
(x is total salary) Then, y % of 3600 = 72
31. (a) A candidate secured 62% of the votes.
y
⇒ x=
22500
× 100 = ` 50000 Remaining votes = 100% − 62% = 38% ⇒ 3600 × = 72
45 100
Candidate is won by a margin of 144 votes. 72 × 100
Anish’s salary = ` 50000 So, 62% − 38% = 144 , 24% = 144 ⇒ y= ⇒y=2
29. (a) Let Sudin’s salary = ` x 3600
144
1% = , 36. (a) Let the number be x.
2x 24
∴ Ram’s salary = 40% of x = ` Then, x − 15= 80% of x
5 144
100% = × 100 = 600 80
 25 2 x x 24 ⇒ x − 15 = x ×
Madan’s salary = `  ×  =` 100
 100 5 10 32. (b) 90% of x = 315
4x
2x x 90 ⇒ x − 15 =
∴ x+ + = 1200 x× = 315 5
5 10 100
⇒ x = ` 8000 315 × 100 ⇒ 5 x − 75 = 4 x
x= = 350 km ⇒ x = 75
8000 90
Hence, Madan’s salary = ` = ` 800 1 1
10 33. (b) According to the question, 37. (d) of 1206 = 1206 × = 402
30. (c) Let the cost price = x 3 3
x %of a = y %of b 402
First reduction of 20% = x −
20 1
× x = x− x ax by ∴ Required per cent = × 100 = 300%
⇒ = 134
100 5 100 100

CHAPTER 9

Profit, Loss and Discount


Whenever a purchased article is sold, then either profit is Loss If selling price of an article is less than its cost price
earned or loss is incurred. (CP), there is loss. In fact,
The following basic terms are very useful to solve the problems Loss = CP − SP
based on profit and loss.
Ex. 1 A person sells an article for a price which gives
Cost Price (CP) The cost price of an article is the price at
him a profit of 20% on cost price of ` 500. Calculate the
which the article is purchased or manufactured.
selling price of the article.
Selling Price (SP) The selling price of an article is the price (a) 520 (b) 600 (c) 720 (d) 680
at which the article is sold. Sol. (b) SP = CP + Profit
Profit or Gain If selling price of an article is more than its = CP + 20% of CP
cost price, there is profit or gain.  20  500 × 120
= 500 1 + = = 500 × 12
. = ` 600
Profit = SP − CP  100  100
∴ Selling price of the article = ` 600
Profit, Loss and Discount 37

Discount Ex. 2 A person marks his goods 20% more than the
cost price and allows some discount on it. He still makes
Discount is a term used during a business. a profit of 10%. Find the discount percentage.
It is the rebate given to the buyer by the seller to increase (a) 8.33% (b) 16.66%
the sale. (c) 10% (d) 8.5%
Discount is always given on marked price or printed price Sol. (a) Let the CP of the article be ` 100.
of the article. Then, MP of the article = ` 120 Since, profit = 10%
i.e. Discount = Marked/Print Price − Selling ∴ SP = CP + Profit =100 +10 = ` 110
Price The difference between marked price and selling price is the
discount.
Impartant Formulae ∴ Discount = MP − SP
Profit SP − CP = 120 − 110 = ` 10
■ Profit percentage = × 100 = × 100
CP CP Discount
Discount percentage = × 100
Loss CP − SP Marked Price
■ Loss percentage = × 100 = × 100
CP CP 10
SP × 100 SP × 100 = × 100 = 8.33%
■ CP = = (P=profit, L=loss) 120
(100 + P%) (100 − L%)
CP × (100 + P%) CP × (100 − L%) Example 3 By selling two articles for ` 180 each a
■ SP = =
100 100 shopkeeper gains 20% on one and looses 20% on the
Discount other, find the percentage profit / loss.
■ Discount percentage = × 100
Marked price (a) 4 % loss (b) 4% profit
(c) No loss (d) 10 % profit
■ If two successive discount of x% and y% is given on an
article, then Selling Price = Marked Price Sol. (a) Let cost price of two articles are ` x and ` y,
100 − x   100 − y 
× 
respectively.
 ×  x × (100 + 20)
 100   100  ∴ 80 =
Equivalent discount =  x + y −
xy  100
 180 × 100
 100  ⇒ x= = 150
■ If a false weight is used during selling of an article, then 120
Error y × (100 − 20)
Gain per cent = × 100% and 180 =
True value − Error 100
Here, Error = True weight − False weight
180 × 100
⇒ y= = 225
■ A person sells goods at a profit of x%. Had he sold it for ` X
80
more, y% would have been gained. Then, CP is given by ∴ Total cost price = 150 + 225 = ` 375
and total selling price = 180 + 180 = ` 360
= ` X × 100
y−x Hence, loss = 375 − 360 = `15
15
■ A person sells goods at a loss of x%. Had he sold it for `X ∴ Loss per cent = × 100 = 4%
375
more, he would have been gained y%. Then, CP is given by
X
=` × 100. When there is a gain of x% and a loss of y% the T Note In cases, where the selling price of two articles is same
y+x and one is sold at the loss of x % and another is sold at a profit
net effect is given by. Net effect =  x − y −
xy  of x%. Or in other words, the profit % and loss % is same and
%
 100  selling price is same. This transaction always yields a loss and
2
■ If CP of x articles is equal to SP of y articles where x > y.  Common loss or gain %
x−y such loss % =  
Then, profit percentage = × 100  10 
y 2
 20 
In the above case loss % =   = 4%
■ Ratio of MP and CP is given as  10 
MP = CP; 100 + P % = 100 − D%; P% → Profit %;
D% → Discount is
38 Self Study Guide SSC Constable (GD) Recruitment Exam

Practice Exercise
Profit and Loss 10. A tradesman sold an article at a 18. A dealer offered a machine for sale
1. The SP of a TV set is marked at loss of 20%. If the selling price had ` 27500, but even if had charged
` 17600 including sales tax at the been increased by ` 100, there 10% less, he would have made a
rate of 10%. Find the sales tax. would have been a gain of 5%. The profit of 10%. The actual cost of the
(a) ` 1600 (b) ` 1250 cost of the price of the article was machine is
(c) ` 1550 (d) ` 1660 (a) ` 200 (b) ` 25 (a) ` 24250 (b) ` 22500
(c) ` 400 (d) ` 250 (c) ` 22275 (d) ` 22000
2. A shopkeeper purchases 11 knives in
`10 and sells them at the rate of 10 11. By selling a basket for ` 19.50, a 19. A man sold a fan for ` 540 losing
knives for ` 11. He earns a profit of shopkeeper gains 30%. For how 10%. At what price should he have
(a) 11% (b) 15% much should he sell it to gain 40%? sold it to gain 10%?
(c) 20% (d) 21% (a) ` 21 (b) ` 21.50 (a) ` 660 (b) ` 648 (c) ` 650 (d) ` 594
(c) ` 24 (d) ` 23
3. A bought a computer system for 20. By selling 12 oranges for ` 10, a man
` 40000 and sold it to B at a loss of 12. A man sold his watch at a loss of losses 20%. How many oranges for
4%. If B sold it to C for ` 40320, 5%. Had he sold it for 56.25 more, `10, should he sell to get a gain of
profit per cent for B is he would have gained 10%. What 20%?
(a) 3 (b) 6 is the cost price of the watch (in (a) 9 (b) 10 (c) 7 (d) 8
(c) 4 (d) 5 `)?
21. One shopkeeper purchases a TV for
4. The difference between the selling (a) 370 (b) 365
(c) 375 (d) 390
` 6000 and sells it at 15% loss, then
price of a shirt sold at profits 15% the selling price of TV will be
and 17% is ` 3. Then, the cost price 13. A man buys a certain number of
(a) ` 5600 (b) ` 5100
of the shirt is oranges at 20 for ` 60 and an equal (c) ` 5900 (d) ` 5910
(a) ` 175 (b) ` 180 number at 30 for ` 60. He mixes
22. For what sum should goods worth
(c) ` 200 (d) ` 150 them and sells them at 25 for ` 60.
What is gain no loss per cent? ` 570 he insured at 5% so that in
5. A salesman expects a gain of 13% case of loss the owner may recover
on his cost price. If in a month his (a) Gain of 4%
(b) Loss of 4% the premium as well as the value
sale was ` 791000. What was his
(c) Neither gain or loss of the goods?
profit?
(d) Loss of 5% (a) ` 640 (b) ` 600
(a) ` 91000 (b) ` 97786
(c) ` 587.50 (d ` 595.40
(c) ` 85659 (d) ` 88300 14. The profit earned by selling an
article for ` 752 is 1.2 times the 23. Two articles are sold for ` 396.00
6. Peter buys a table for ` 450 and
spends ` 30 on its transportation. If loss incurred when the same each. On one, the seller gains 10%
he sells the table for ` 600, his gain article was sold for ` 400. What is and on other he loses 10%. Find
per cent will be the CP of the article? his gain or loss.
(a) 30 (b) 25 (a) ` 520 (a) 1% loss (b) 1% profit
(c) 28 (d) 24 (b) ` 580 (c) 2% profit (d) No loss no gain
7. A man sells rice at 10% profit and (c) ` 560 24. A tradesman marks his goods at
(d) Data inadequate such a price that after allowing a
uses weight 30% less than the
actual measure. His gain per cent 15. If an article is sold at 200% profit, discount of 15%, he makes a profit
is then the ratio of its cost price to its of 20%. What is the marked price of
2 1 selling price will be an article whose cost price is ` 170?
(a) 57 % (b) 57 % (a) ` 220 (b) ` 200 (c) ` 240 (d) ` 260
8 7 (a) 1 : 2 (b) 2 : 1
2 3 (c) 1 : 3 (d) 3 : 1
(c) 57 %
5
(d) 57 %
7 16. By selling an article for ` 102, there
Discount
8. A sells an article to B at a profit of is a loss of 15%, when the article is 25. If a shopkeeper marks the price of
10%. B sells the article back to A at sold for ` 134.40, the net result in goods 50% more than their cost
the transaction is price and allows a discount of 40%,
a loss of 10%. In this transaction
(a) 12% gain (b) 12% loss
what is his gain or loss per cent?
(a) A neither loses nor gains
(c) 10% loss (d) 15% gain (a) 15% loss (b) 10% loss
(b) A makes a profit of 11%
(c) A makes a profit of 20% (c) 10% gain (d) 15% gain
17. A toy car was sold at a loss for
(d) B loses 20% 26. A shopkeeper sells his goods at 10%
` 60. Had it been sold for ` 81, the
9. Some articles were bought at 6 for 3 discount on the marked price. What
` 5 and sold at 5 for ` 6. Gain is gain would have been of the price should he mark on an article
4 that costs him ` 900 to gain 10%?
(a) 5% (b) 6%
former loss. The cost of the toy is (a) ` 1050 (b) ` 1100
(c) 30% (d) 44%
(a) ` 72 (b) ` 80 (c) ` 1150 (d) ` 1000
(c) ` 65 (d) ` 81
Profit, Loss and Discount 39

27. After allowing a discount of 12% on At first, the price of one mango 44. If the profit on selling an article for
the marked price of an article, it is was ` 425 is the same as the loss on
sold for ` 880. Find its marked (a) ` 6 (b) ` 5 (c) ` 4 (d) ` 7 selling it for ` 355, then the cost
price. 36. A shopkeeper, in order to clear his price of the article is
(a) ` 1100 (b) ` 2000 [SSC Constable, 2012]
old stock of TV sets, offers 12%
(c) ` 1000 (d) ` 2100 (a) ` 370 (b) ` 380
discount on the TV sets. If the
28. A trader marks his goods at 20% (c) ` 390 (d) ` 400
marked price of TV set is ` 6500,
above the cost price. If he allows a the selling price of the TV set is 45. If the cost of 18 articles is equal to
discount of 5% for cash down (a) ` 5700 (b) ` 5720 the selling price of 16 articles, the
payment, his profit per cent for (c) ` 5400 (d) ` 6000 gain or loss is [SSC Constable, 2012]
such a transaction is (a) 25% gain (b) 25% loss
37. A dealer marks his goods 20% 1 1
(a) 15 (b) 12 (c) 12 % loss (d) 12 % gain
(c) 14 (d) 17 above cost price and allows a 2 2
29. A discount of 40% on the marked discount of 10% to his customers.
46. Dinesh bought two radios for
price of a trouser enables Ajit to His gain percentage is
`1920. He sold one at a profit of
(a) 9 (b) 6 (c) 8 (d) 7
purchase a shirt also which costs 2
38. A seller increases the cost price of an 20% and the other at a loss of 6 %.
him ` 320. How much did Ajit pay 3
for the trouser? article by 30% and fixed the marked If the selling prices of both radios
(a) ` 480 (b) ` 540 price as ` 286. But during sale he are same, the cost price of the two
(c) ` 800 (d) ` 400 gave 10% discount to the purchaser. radios are [SSC Constable, 2013]
30. When a discount of 12% on the The percentage of profit will be (a) ` 800 and ` 1120
marked price of an article is (a) 10 (b) 15 (b) ` 840 and ` 1080
allowed, the article is sold for (c) 20 (d) 17 (c) ` 860 and ` 1060
` 264. The marked price of an 39. An item was sold for ` 3600 at 25% (d) ` 900 and ` 1020
article is discount. Its marked price was 47. Peter buys a table for ` 450 and
(a) ` 300 (b) ` 276 (c) ` 312 (d) ` 325 (a) ` 4800 (b) ` 4500 spends ` 30 on its transportation. If
(c) ` 2700 (d) ` 2880
31. A shopkeeper marks his goods 30% he sells the table for ` 600, his gain
above his cost price but allows a Previous Years’ Questions per cent will be
discount of 10% at the time of sale. [SSC Constable, 2012]
His gain is 40. The ratio of cost price and selling (a) 30 (b) 25
(a) 21% (b) 20% price is 20 : 25. The profit per cent (c) 28 (d) 24
(c) 18% (d) 17% is [SSC Constable, 2013] 48. By selling an article for ` 960 a
32. A retailer gets a discount of 40% on (a) 25 (b) 20 man incurs a loss of 4%; what was
the printing price of an article. The (c) 125 (d) 80
the cost price?[SSC Constable, 2011]
retailer sells it at the printing price. 41. A trader purchased a ratio at 6 th (a) ` 1000 (b) ` 784
His gain per cent is 7 (c) ` 498.4 (d) ` 300
(a) 40 (b) 55 of its marked price and sold it at 49. A milkman brought 70 L of milk
2 2
(c) 66 (d) 75 14 % more than its marked price. for ` 630 and added 5 L of water. If
3 7 he sells it at ` 9 per litre, his profit
33. Ramesh bought a fan on 25% His gain is [SSC Constable, 2013] percentage is [SSC Constable, 2011]
discount on its price. He get profit 1
(a) 15 %
4
(b) 28 % 1
(a) 8 (b) 7
of 10% by selling it on ` 660. What 7 7 5
was the price of the fan? 1 2 1
(c) 33 % (d) None of these (c) 8 (d) 7
(a) ` 800 (b) ` 700 (c) ` 600 (d) ` 685 3 5 7
34. A shopkeeper sells a radio at 20% 42. By selling a car for ` 156600 a
discount on printed price and gets person suffers 13% loss. What was Answers
15% profit. What is the printed his loss? [SSC Constable, 2013]
1 a 2 d 3 d 4 d 5 a
price? (a) ` 20358 (b) ` 23400 b b b d c
6 7 8 9 10
(a) 35% more than cost price (c) ` 136242 (d) ` 18016 11 a 12 c 13 b 14 c 15 c
(b) 75% more than cost price 43. Mr. A spents 78% of his monthly 16 a 17 a 18 b 19 a 20 d
(c) 20% more than cost price income and saves the rest. If his 21 b 22 b 23 a 24 c 25 b
(d) None of the above 26 b 27 c 28 c 29 a 30 a
annual saving is ` 22440 his
35. A fruit merchant makes a profit of 31 d 32 c 33 a 34 d 35 c
monthly income (in `) is b c d a a
25% selling mangoes at a certain 36 37 38 39 40
[SSC Constable, 2013]
41 c 42 b 43 b 44 c 45 d
price. If he charges `1 more on (a) 7800 (b) 8500
46 b 47 b 48 a 49 d
each mango, he would gain 50%. (c) 9700 (d) 10125
40 Self Study Guide SSC Constable (GD) Recruitment Exam

Hints & Solutions


1. (a) Price of TV set = ` x, then 6. (b) Cost price of the table 12. (c) Let CP of the watch = ` x
x × 110 = 450 + 30 = ` 480 According to the question,
Given, = 17600
100 600 − 480 x × (100 + 10) x × (100 − 5)
∴ Profit percentage = × 100 − = 5625
.
17600 × 100 480 100 100
∴ x= = ` 16000
110 120 110 x 95 x
= × 100 = 25% ⇒ − = 5625
.
∴ Required sales tax = 17600 − 16000 480 100 100
= ` 1600 7. (b) Let the marked weight = 1 Kg ⇒
15 x
= 5625.
2. (d) Selling Price of 1 Knife = `
11 = 1000 g 100
10 Real weight = 70% of 1000 = 700 g . × 100 5625
5625
⇒ x= =
10 Let CP of 1g = ` 1 15 15
Cost Price of 1 Knife = `
11 ∴ CP of 700g = ` 700 ∴ x = ` 375
11 10 13. (b) Let the man buy 60 oranges (LCM of
Profit = SP − CP = − CP of 1000 g = ` 1000
10 11 SP = 110% of 1000 20 and 30) of each kind.
121 − 100 21 CP of the 60 oranges of the first kind
= = =
110
× 1000 = ` 1100
110 110 60
100 = × 60 = ` 180
21 20
Gain = 1100 − 700 = ` 400
Now, profit per cent = 110 × 100 400 1 CP of 60 orange of second kind
10 ∴ Gain = × 100% = 57 % 60 × 60
11
700 7 = = ` 120
8. (b) Suppose the cost price of article be 30
21 11
= × × 100 = 21% ` 100. Total CP of 120 oranges
110 10
Then, A sells it to B = ` 110 = `(180 + 120) = ` 300
3. (d) Cost price of B 60
and B sells it to Their SP = × 120 = ` 288
(100 − Loss) × CP of A
= 90 25
100 A = 110 × = ` 99 Loss = ` ( 300 − 288) = ` 12
100
(100 − 4) × 40000 12
= = ` 38400 So, gain for A = 10 + 1 = ` 11 ∴Loss per cent = × 100 = 4%
100 300
∴ Per cent gain = 11%
Now, CP of B = ` 38400 14. (c) Let the CP of the article be ` x.
5
B sell the computer to C at ` 40320 9. (d) CP of 1 article = ` 752 − x = ( x − 400) × 12
.
6
∴ SP for B = 40320 ⇒ 752 − x = 12
. x − 480
SP − CP 6
Now, profit per cent = × 100 SP of 1 article = ` 1232
5 ⇒ 2 .2 x = 1232 ∴ x = = ` 560
CP 2 .2
40320 − 38400
∴ Gain = `  − 
6 5
= × 100 = 5 % 15. (c) Let the CP = ` 100
38400  5 6
 36 − 25  = ` 11 SP = 100 + 200% of 100
4. (d) Let the cost price of shirt = ` x = `  200 × 100
Then, selling price at profit of 15%  30  30 = 100 +
100
(100 + Profit) × CP (100 + 15) × x 115 x  × 100 % = 44%
11 / 30
= = = Gain per cent =  = 100 + 200 = 300
100 100 100  5/6 
∴Required ratio = 1 : 3
Similarly, selling price at per cent of 17% 10. (c) Let the cost price of an article be ` x.
(100 + 17 ) × CP 16. (a) Here, SP = ` 102, loss per cent = 15
= 80 x 4x
∴ SP of the article = ` =` 100 × SP
100 100 5 CP of article =
117 × x 117 x 4x 100 − Loss per cent
= = New SP of the article = ` + 100
100 100 100 × 102
5 =
Now, by given condition 4x 105 x 100 − 15
∴ + 100 =
117 x 115 x 5 100 10200
− = 3, =
100 100 4x 21x 85
⇒ + 100 =
117 x −115 x 5 20 = ` 120
=3
21 − 16 
⇒ 
100 Now, the article is sold for ` 134.40.
 x = 100
100 × 3  20 
⇒ x= = ` 150 Here, SP > CP
2 100 × 20 SP − CP
⇒x= = ` 400 ∴Profit per cent = × 100
5. (a) Gain = 13%; Total SP = ` 791000 5 CP
100 × SP 100 × 791000 11. (a) Cost price of basket 134.40 − 120
Now, CP = = = × 100
(100 + Gain%) (100 + 13) = 19.50 ×
100
= ` 15 120
130 14.40 × 100
=
79100000
= ` 700000 =
113 On 40% profit, selling price of basket 120
140 1440
Gain = SP − CP = 791000 − 700000 = 15 × = ` 21 = = 12%
100 120
= ` 91000
Profit, Loss and Discount 41

17. (a) Let the cost of the toy be ` x. 24. (c) According to the question, 30. (a) Let the marked price of the article be
3 (100 − 15) 170 × (100 + 20) ` x.
∴ ( x − 60) = ( 81 − x ) Marked price × =
4 100 100 Then, 88% of x = 264
3 170 × 120 x × 88
⇒ x − 45 = ( 81 − x ) ∴Marked price = = ` 240 ⇒ = 264
4 85 100
7x 25. (b) Let the cost price = ` 100 264 × 100
⇒ = 81 + 45 ⇒ x= = ` 300
4 Then, marked price is 50% above CP 88
126 × 4 (100 + 50%) × 100 ∴Marked price of the article = ` 300
⇒ x= = ` 72 ∴ Marked price =
7 100 31. (d) If the price of a goods is first increased
18. (b) 10% of 27500 = ` 2750 150 × 100 by x % and decreased by y % . Then, effect on
= = ` 150
SP = 27500 − 2750 = ` 24750 100 the price of goods
=  x − y −
24750 × 100 2475000 Now, shop allowed 40% discount xy   300 
∴ CP = =  % =  30 − 10 −  % = 17%
100 + 10 110 ∴ Selling price  100   100 
(100 − Discount%) × Marked price 32. (c) Let the printing price be ` x.
= ` 22500 =
100 60 x 3x
19. (a) SP = ` 540 (100 − 40) × 150 ∴CP for the retailer = ` =`
= 100 5
Loss = 10% 100
On selling at printing rate, the profit = `  x −
3x
CP = x − 10% of x = 540 60 × 150 
=  5
x 9x 100
= x− = 540 = = 540 2x
= ` 90 =`
10 10 5
Clearly, shopkeeper had a loss as CP > SP.
540 × 10 ∴ Profit per cent =
2x/ 5
× 100%
x= = ` 600 CP − SP
9 ∴ Loss per cent = × 100 3x / 5
CP 200 2
10% profit, = 600 + 600 ×
10
= ` 660 100 − 90 = % = 66 %
100 = × 100 3 3
100
10 5 33. (a) Original Price = ` x
20. (d) SP of 1 orange = ` = ` = 10% loss
12 6 3x
26. (b) Let marked price be ` x. Cost price = (x − 25% of x) = `
5 4
× 100 ∴ SP = x ×
90
=`
9
x
Selling Price = 
500 25 3x 3 x  33 x
∴ CP of 1 orange = 6 = = ` 100 10 + 10% of  =
100 − 20 480 24 110  4 4 40
25  100 + 20  Again, SP = 900 × = ` 990 33 x
∴ SP of 1 orange for 20% gain =   100 ⇒ = 660
24  100  40
According to the question,
=`
5 660 × 40

9
x = 990 ∴ x= = ` 800
4 33
10
∴ Number of oranges to be sold for ` 10 990 × 10 34. (d) Ratio of MP and CP is given as
5 ∴ x= = ` 1100
= 10 ÷ = 8 9 MP 100 + P %
=
4 CP 100 − D %
27. (c) Suppose, marked price of the article
21. (b) Loss =15% be ` x. MP 100 + 15
∴ =
CP = ` 6000 Then, 88% of x = 880 CP 100 − 20
SP = 6000 − 15% of 6000 88 115 23
⇒ x× = 880 ⇒ x = `1000 ∴Required percentage = =
= 6000 −  6000 ×
15  100
 80 16
 100  28. (c) Suppose, cost price of the goods ∴Market Price = 23
= 6000 − 900 = ` 5100 be ` 100. Cost Price = 16
22. (b) Let the value of goods be ` x. ∴ Marked price of the goods According to the question,
According to the question, = `120 23 − 16
× 100
x − 5% of x = 570 ∴ After 5% discount, selling price of the goods 16
95
⇒ x− x×
5
= 570 = 120 × = ` 114 =
7
× 100
100 100 16
x ∴Profit per cent = 43 .75%
⇒ x− = 570 114 − 100
20 = × 100 = 14% ∴Printed Price is 43.75% more than cost price.
19 x 100
⇒ = 570 35. (c) Let cost price of a mango = ` x
20 29. (a) Suppose Ajit pay ` x for
(trouser + shirt) Fruit merchant makes a profit of 25%
570 × 20
∴ x= = ` 600 So, selling price
19 40
Then, x × = 320
125 5
23. (a) In this type of transaction there is 100 ⇒ of x ⇒ x
100 × 320 100 4
always a loss. ⇒x= ⇒ x = ` 800
∴ Required loss per cent 40 If selling price is increases by 1, then his profit
10 × 10 ∴Amount paid for trouser = 800 − 320 = ` 480 becomes 50%
= = 1% 5 150 5 3
100 x+ 1= of x, x + 1 = x
4 100 4 2
42 Self Study Guide SSC Constable (GD) Recruitment Exam

5 3 5x − 6x x × (100 − 25) 44. (c) Suppose, the cost price of the article
x − x = −1 , = −1 = 3600
4 2 4 100 =` x
− x = −4 , x = 4 3600 × 100 425 − x = x − 355
x= Then,
36. (b) Marked price of TV set 6500, discount 75 ⇒ 2 x = 780
= 12% x = ` 4800 ⇒ x = ` 390
We know that, CP 20
40.(a) = 45. (d) Required gain per cent
Marked Price − Discount = Selling Price SP 25
18 − 16
12 Let CP = 20 x and SP = 25 x = × 100
Selling Price = 6500 − 6500 × 16
100 ∴The required profit per cent
2 25 1
SP − CP = × 100 = = 12 %
= 6500 − 65 × 12 =  × 100 % 16 2 2
= 6500 − 780 = ` 5720  CP 
46. (b) Suppose, cost price of first radio = ` x
25 x − 20 x
37. (c) Let the cost price = `100 =  × 100 % ∴ Cost price of second radio
 20 x 
According to the question, = ` (1920 – x)
5x
A dealer marks his goods 20% above cost = × 100% 120 280
20 x Then, x × = (1920 − x ) ×
price. 100 300
= 25%
20 ⇒ 360 × x = (1920 − x ) × 280
Marked price = 100 + of 100 41. (c) Let the marked price is ` 700.
100 ⇒ x ( 360 + 280) = 1920 × 280
6 1920 × 280
= 100 + 20 = ` 120 Then, CP = 700 × = ` 600 ⇒ x=
7 640
The dealer allows a discount of 10% to his 100 1
customers. and SP = 700 + 700 × × ⇒ x = ` 840
7 100
We know that ∴ Cost price of second radio
= 700 + 100 = ` 800
MP (Marked Price) − Discount = ` 1920 − ` 840
So, the required gain
= Selling Price (SP) = ` 1080
SP − CP
=  × 100 % 47. (b) Cost price of the table
120 − 10% of 120 = SP, 120 − 12 = SP  CP 
= ` 450 + ` 30 = ` 480
Selling Price = `108 800 − 600
=  × 100 % ∴ Profit percentage
Gain = Selling Price − Cost Price  600  600 − 480
= 108 − 100 = ` 8 200 × 100 100 1 = × 100
= %= % = 33 % 480
Gain 600 3 3
Gain per cent = × 100 =
120
× 100 = 25%
Cost price SP × 100
42. (b) CP of the car = 480
8 100 − Loss per cent
= × 100 = 8% 48. (a) CP of article
100 156600 × 100 100
= = × SP
38. (d) Marked price of an article = `286 100 − 13 100 − Loss per cent
Cost price × (100 + 30) 156600 × 100 100
Q = 286 = = × 960 = ` 1000
100 87 96
286 × 100 = ` 180000 49. (d) CP of 75 L of mixture of milk and water
∴ Cost price = = `220
130 ∴ Loss = 180000 − 156600 = ` 630
= ` 23400
Selling price after discount SP of 75 L of mixture of milk and water
286 × (100 − 10) 43. (b) Let the monthly income of Mr. A = x = 9 × 75 = ` 675
= = ` 257.4
220 Saving of Mr. A = (100 − 78)% of x Gain = 675 − 630 = ` 45
257.4 − 220 = 22% of x Gain per cent =
45
× 100
∴Profit percentage = × 100 = 17%
220 22440 630
∴ 22% of x = (monthly saving)
12 50 1
39. (a) Let the marked price of on item = ` x = =7 %
1870 × 100 7 7
According to the question, ∴100% of x = = ` 8500
22
CHAPTER 10

Simple and Compound Interests


Interest (I) Compound Interest (CI)
Interest is the money paid by the borrower to the lender for It is the interest calculated on a sum of money which
the use of money. includes principal and interest calculated for previous year.

Simple Interest (SI) Important Formulae


If the interest is calculated on the original principal at any Let principal = P,rate = R%per annum, time = n yr and amount = A
rate of interest for any period of time, then it is called n
When interest is compounded annually A = P  1 +
R 
simple interest. ■

 100 
PTR
SI = 2n
When interest is compounded half-yearly A =P  1+
R /2 
100 ■

 100 
Where, SI = Simple Interest 4n
When interest is compounded quarterly A = P  1 +
R /4 
T = Time ■

 100 
P = Principal ■ When interest is compounded annually but time is in
R = Rate  2 
2  R 
3  R
Amount (A) = P + SI fraction, say 3 yr. Then, A = P  1 +  × 1 + 5 
5  100   100 
3  
Ex. 1 Calculate the simple interest on ` 7200 at 12 %
4 ■ If rates of interest are R1%,R 2 % and R 3 % for 1st, 2nd and
3rd yr respectively, then A = P  1 + 1   1 + 2   1 + 3 
per annum for 9 months R R R
(a) 700 (b) 688.5 (c) 710.5 (d) 680  100   100   100 
PTR
Sol. (b) SI = ■ On a certain sum of money, the difference between CI and
2
100
SI for 2 yr at r% in given by sum 
r 
7200 × 9 × 51  51 9   .
= = ` 688.50 Q R = %, T = yr  100 
100 × 12 × 4  4 12 
Ex. 3 A sum of money placed at CI doubles itself in
T Note: A sum of money of r1 % rate becomes n1 times of itself on a 4 yr. In how many years will it amount to 8 times itself?
certain time and becomes n2 time of itself at r 2 %, then (a) 12 (b) 14
n −1 (c) 16 (d) 10
r2 % = 2 × r1 %
n1 − 1 4 4
Sol. (a) Given, 2P = P 1 +
r   r 
 Or 1 +  =2
 
100   100
Ex. 2 A sum of money on a certain time at 10% rate 4 3
1  r 
becomes 3 times. Now, at what rate per cent it ⇒ 1 +   = (2) = 8
3

2 100 
 
1 12
becomes 2 times on that given time (certain time).  r 
2 ⇒ P 1 +  = 8P
 100 
(a) 5 % (b) 6 %
(c) 7 % (d) 18% Hence, required time is 12 yr.
m
Sol. (b) According to the formula, T Note If a sum becomes n times in t yr at CI then it will be (n )
 1  times in m yr.
n2 − 1  2 − 1 Thus, if a sum of money doubles itself in 4 yr, then it will become
Then r2 = × r1% ∴ r2 =  2  × 10% = 6%
n1 − 1  3 1 − 1 (2 )3 times in 4 × 3 = 12 yr.
 2 
44 Self Study Guide SSC Constable (GD) Recruitment Exam

Ex. 4 If the compound interest on a certain sum for SI 6 10000 200


= × =
2 yr, at 3% be ` 101.50, what would be the simple CI 100 609 203
interest? 200 200
∴ SI = × CI = × 1015
. = ` 100
(a) 100 (b) 100.5 203 203
(c) 90 (d) 99.5
2 Ex. 5 The difference between the CI and the SI on a
Sol. (a) CI on ` 1 = 1 +
3  609
 −1= certain sum of money at 5% per annum for 2 yr is ` 1 . 50.
 100  10000 Find the sum
2 ×3 6
SI on ` 1 = = (a) 600 (b) 500 (c) 700 (d)640
100 100 2
100 
Sol. (a) Sum = 15
.   = 15
. × 400 = ` 600
 5 

Practice Exercise
Simple Interest rate per cent per annum. The rate 12. A sum of ` 1550 was lent partly at
per cent is 5% and partly at 8% per annum
1. Neha borrowed ` 500 from her 2 3
(a) 6 % (b) 5 % simple interest. The total interest
friend at the rate of 12% per 3 5 received after 4 yr was ` 400. The
1 2 1
annum for 2 yr. The interest and (c) 7 % (d) 6 % ratio of the money lent at 5% to
2 3 3
that lent at 8% is
the amount paid by her 7. If the simple interest on ` 1200 be (a) 16 : 15 (b) 17 : 15 (c) 16 : 13 (d) 16 : 19
(a) ` 140, ` 540 (b) ` 130, ` 530 more than the interest on ` 1000 by
(c) ` 125, ` 525 (d) ` 150, ` 650 13. Ajay takes some loan from Rashmi
` 30 in 3 yr. The rate per cent per at the rate of 5% per annum and
2. If the simple interest on ` 1500 annum will be after 2yr, Ajay gave back `8800 to
increases by ` 30, when the time (a) 5% (b) 7% (c) 6% (d) 8% Rashmi and this way paid his
increases by 8 yr. The rate per cent 8. Arun borrowed a sum of money whole loan. Find the interest paid
per annum is from Jayant at the rate of 8% per by Ajay.
(a) 0.5% (b) 0.25% annum simple interest for the first (a) ` 825 (b) ` 975
(c) 0.75% (d) 1.25% (c) ` 800 (d) ` 850
4 yr, 10% per annum for the next 6
3. Mukul invested an amount of yr and 12% per annum for the
` 12000 at the simple interest rate period beyond 10 yr. If, he pays a Compound Interest
of 10 per cent and another amount total of ` 12160 as interest only at
14. A sum of money amounts to ` 6690
at the simple interest rate of 20 per the end of 15 yr, how much money
cent per annum. The total interest did he borrow? after 3 yr and to ` 10035 after 6 yr
on compound interest. The sum is
earned at at the end of one year on (a) ` 8000 (b) ` 10000
(a) ` 4460 (b) ` 3650
the total amount invested become (c) ` 12000 (d) ` 9000
(c) ` 4535 (d) ` 2800
14 per cent per annum. The total 9. A sum becomes 6 fold at 5% per
amount invested was 15. A sum of money doubles itself at
annum. At what rate, the sum compound interest in 15 yr. In how
(a) ` 22000 (b) ` 25000 becomes 12 fold?
(c) ` 20000 (d) ` 24000
many years will it become eight
(a) 10% (b) 12% times?
4. The simple interest on a sum of (c) 9% (d) 11%
(a) 20 yr (b) 40 yr (c) 35 yr (d) 45 yr
money is 1/25 of the principal and 10. Ramesh lent out 40% of a certain
the number of year is equal to the 16. A certain sum amounts to ` 7350
sum at the annual rate of 15%, he
rate per cent per annum. The rate lent 50% of the remaining at the in 2 yr and to ` 8575 in 3 yr. The
per cent is annual rate of 10% and the rest sum and the rate per cent are
(a) 3% (b) 4% (c) 2% (d) 2.5% 2 2
amount was lent out at 18% per (a) 16 %, ` 5400 (b) 16 %, ` 2400
3 3
5. The simple interest on a sum of annum. Find the annual rate on 1
money is equal to the principal and whole sum (c) 12 %, ` 5400 (d) None of these
2
the number of year is equal to the (a) 13.4% (b) 14.33%
rate per cent per annum. The rate (c) 14.4% (d) 13.33% 17. Seema invested an amount of
per cent is ` 16000 for two years at compound
11. What will be the ratio of simple interest and received an amount of
(a) 25% (b) 100% (c) 10% (d) 12% interest earned by certain amount ` 17640 on maturity. What is the
6. The simple interest on a sum of at the same rate of interest for 12 yr rate of interest?
money is 4/9 of the principal and and for 18 yr? (a) 8% (b) 5% (c) 4% (d) 3%
the number of years is equal to the (a) 2 : 5 (b)1 : 3 (c) 2 : 3 (d) 3 : 1
Simple and Compound Interests 45

18. The compound interest on ` 350 for 25. A sum of money lent at compound 31. Anil lent ` 1000 to Sourav for 4 yr
1 yr at 4% per annum, the interest interest for 2 yr at 20% per annum and ` 3000 to Vinod for 2 yr. If he
being payable half yearly, will be would fetch ` 964 more, if the received ` 900 as simple interest
(a) ` 36414
. (b) ` 36515
.
interest was payable half yearly altogether, the rate of interest is
than if it was payable annually. [SSC Constable, 2013]
(c) ` 1414
. (d) ` 1515
.
What is the sum? (a) 5% (b) 7%
19. The compound interest on ` 2000 (a) ` 40000 (b) ` 60000 (c) 8% (d) 9%
1 (c) ` 90000 (d) ` 500000 32. Which sum of money at compound
for 1 yr is 10% per annum, the
4 26. Find the least number of complete interest will amount to ` 650 at the
interest being payable quarterly year in which a sum of money put end of 1st year and ` 676 at the
will be out at 20% compound interest, will end of 2nd year?
(a) ` 2262.81 (b) ` 262.81 be more than double. [SSC Constable, 2012]
(c) ` 26181
. (d) None of these (a) ` 600 (b) ` 620
(a) 3 yr (b) 4 yr (c) 5 yr (d) 8 yr
(c) ` 625 (d) ` 630
20. ` 7500 is borrowed at CI at the rate 27. The simple interest on a certain 33. In what time will ` 1860 amount to
of 4% per annum. What will be the sum of money for 3 yr at 8% per
amount to be paid after 6 months, `2641.20 at simple interest 12% per
annum is half the compound
if interest is compounded annum? [SSC Constable, 2012]
interest on ` 8000 for 2 yr at 10% 1 1
quarterly? per annum. Find the sum placed (a) 3 yr (b) 3 yr (c) 4 yr (d) 4 yr
(a) ` 765075
. (b) ` 723025
. 2 2
on simple interest
(c) ` 7215.30 (d) ` 703516
. (a) ` 3500 (b) ` 3800 34. John invested a sum of money at
(c) ` 4000 (d) ` 3600 an annual simple interest rate of
21. The compound interest on a sum at
the rate of 5% for 2 yr is ` 512.50. 10%. At the end of four years the
Previous Years’ Questions amount invested plus interest
The sum is
(a) ` 5200 (b) ` 4800
earned was ` 770. The amount
28. A sum of ` 800 becomes ` 956 in
(c) ` 5000 (d) ` 5500
invested was [SSC Constable, 2011]
3 years at a certain rate of simple (a) ` 650 (b) ` 350
22. If the difference between the interest. It the rate of interest is (c) ` 550 (d) ` 500
compound interest and the simle increased by 4% what amount will
35. The compound interest on ` 6400
interest on a certain sum for 2 yr at the same sum become is 3 years? 1
8% per annum is 32 then the sum is [SSC Constable, 2015] for 2 yr at 7 % per annum is
(a) ` 1024 (b) ` 1025 2
(a) ` 5000 (b) ` 5500 [SSC Constable, 2011]
(c) ` 6000 (d) ` 5250 (c) ` 1052 (d) ` 1042
(a) ` 1016 (b) ` 996
23. SBI lent ` 1331 lakh to the TATA 29. A certain sum amounts to ` 1725 (c) ` 976 (d) ` 966
group at compound interest and at 15% per annum at simple
got ` 1728 lakh after 3 yr. What is interest and ` 1800 in the same Answers
the rate of interest charged, if time at 20% per annum at simple
compounded annually? interest. Find the sum. 1 d 2 b 3 c 4 c 5 c
(a) 11% (b) 9.09% [SSC Constable, 2015] 6 a 7 a 8 a 9 d 10 c
(c) 12% (d) 8.33% (a) ` 1650 (b) ` 1200 c a c a d
11 12 13 14 15
(c) ` 1700 (d) ` 1500
24. A sum of ` 400 amounts to ` 441 in 16 a 17 b 18 c 19 b 20 a
2 yr. What will be its amount, if the 30. Find the difference between SI and
21 c 22 a 23 b 24 a 25 a
rate of interest is increased by 5%? CI on ` 2000 in 2 yr at 6% per
annum [SSC Constable, 2013] 26 b 27 a 28 c 29 d 30 d
(a) ` 484 (b) ` 560
(c) ` 512 (d) ` 600 (a) ` 7.30 (b) ` 7.10 31 d 32 c 33 b 34 c 35 b
(c) ` 7.00 (d) ` 7.20

Hints & Solutions


500 × 5 × 12 3. (c) Let the total amount invested be ` x. According to the question,
1. (d) SI = = ` 150
2 × 100 x × 14 12000 × 10 ( x − 12000) × 20 P P×R×R
Then, = + = [QR = T ]
A = P + SI = 500 + 150 = ` 650 100 100 100 25 100
100
2. (b) t = 8 yr ⇒ 014
. x = 1200 + 02 . x − 2400 R2 = =4
25
SI increase by ` 30 in 8 yr ⇒ x = ` 20000
30 R = 2%
∴SI increases in 1 yr = = 375
. 4. (c) Let Principal = P
8 P P×R ×R
Let the rate be x% per annum Simple Interest = 5. (c) P = ⇒ R 2 = 100
25 100
1500 × 1 × x
. =
375 Rate = R % ∴ R = 10%
100
x = 0.25%
46 Self Study Guide SSC Constable (GD) Recruitment Exam

4 P×R ×R 400 P 2n
18. (c) A = P  1 +
6. (a) P= ⇒ R2 = P+ = 8800 R/2 
9 100 9 10 
 100 
20 2 11 P
∴ R = ⇒ R =6 % ⇒ = 8800 102 102
3 3 10 = 350 × × = ` 36414
.
(Difference in P ) × R × T 100 100
88000
7. (a) ⇒ P= ∴Compound interest
100 11 = ` 36414
. − ` 350 = ` 1414
.
= Difference in SI ∴ P =` 8000 5
4 ×
200 × 3 × r
19. (b) A = 2000  1 +
∴Interest paid = 8800 − 8000 = ` 800 10 
= 30 ⇒ R = 5% 
4
100  400 
14. (a) Let the sum be ` P.
8. (a) Interest for 4 yr @ 8% + interest for 6 yr 5
= 2000   = ` 2262.81
3 41
P  1 +
@ 10% + interest for 5 yr @ 12% R   40 
= 32 + 60 + 60 = 152%
Then,  = 6690 ...(i)
 100 
P × 152 6 ∴Compound interest = 2262.81 − 2000
12160 = ⇒ P = ` 8000
P  1 +
R  = ` 262.81
100 and  = 10035 ...(ii)
 100  4 ×
1
9. (d) SI at 5% = 6P − P = 5P 20. (a) A = 7500 1 +
4  2
On dividing Eq. (ii) by (i), we get 
P× 5×T 400 
∴ 5P = ⇒ T = 100 yr 3
 1 + R  = 10035 = 3
100   7500 × 101 × 101
 100  6690 2 = = ` 765075
.
Now, for new rate ( R ) 100 × 100
P × R × 100 3
11P = ∴ R = 11% From Eq. (i), P × = 6690  2 
21. (c) CI = P   1 +
R 
100 2  − 1
 100 
10. (c) Let the principal P be 100. ⇒ P = ` 4460  
15
P  1 +
R   2 
⇒ 512.50 = P   1 +
∴ 40% of P = ` 40 15. (d)  = 2P 5 
 100   − 1
 100 
Remaining sum = ` (100 − 40) = ` 60 15
 
⇒ 1 + R  =2 512.50 × 400
50% of ` 60 = 60 ×
50
= ` 30   ...(i) ∴ P= = ` 5000
 100  41
100
n
22. (a) Given, R = 8%, D = ` 32 and P = ?
P  1 +
R 
Remaining sum = [100 − ( 40 + 30)] = ` 30 Now,  = 8P
 100  2
By formula, D = P 
Total interest R 
n 
40 × 15 × 1 30 × 10 × 1 30 × 18 × 1  1 + R  = 8 = (2 )3  100 
= + + ⇒  
100 100 100  100  2
32 = P 
8 
3 ⇒ 
= ( 6 + 3 + 5.4) = ` 14.4  R  
15
 100 
P× R ×T =   1 +  
Now, we know SI =  100  32 × 100 × 100
100   ∴ P= = ` 5000
8× 8
100 × R × 1 [from Eq. (i)]
⇒ 14.4 = n 45 23. (b) According to the question,
100  R   R 
⇒ 1 +  = 1 +  3
1728 = 1331  1 +
R 
∴ R = 14.4%  100   100  
11. (c) If the principal = P and interest rate  100 
⇒ n = 45 1728  R 
= R% ⇒ = 1 + 
∴ Required time = 45 yr 1331  100 
P × R × 12
16. (a) SI on ` 7350 of 1 yr = ` ( 8575 − 7350) 3 3
Then, required ratio = 100  12  =  1 + R 
P × R × 18 = ` 1225 ⇒    
 11   100 
100  100 × 1225  2
∴ Rate =   % = 16 % R 12
=
12 2
= =2:3  7350 × 1  3 ⇒ 1+ =
18 3 100 11
Let the sum be ` x. R 12 1
12. (a) Let the sum lent at 5% = P 2 ⇒ = − 1=
 50  100 11 11
∴ Sum lent at 8% = (1550 − P ) Then, x  1 +  = 7350 100
P × 5 × 4 (1550 − P ) × 8 × 4  3 × 100  ∴ R = = 9.09%
Then, + 11
7 7
100 100 ⇒ x × × = 7350 24. (a) According to the given condition,
= 400 6 6 2
441 = 400  1 +
36 R 
⇒ 20P − 32 P + 1550 × 32 = 40000 ⇒ x = 7350 × = 5400 
 100 
⇒ − 12 P + 49600 = 40000 49
2
⇒ − 12 P = − 9600 ⇒ Sum = ` 5400 441  R 
n ⇒ = 1 + 
∴ P = ` 800 A 
17. (b) =  1 +
R  400  100 

Sum lent at 8% = 1550 − 80 = ` 750 P  100  2 2
⇒  21  =  1 + R 
∴ Required ratio 2    
17640  R   20   100 
= 800 : 750 = 16 : 15 ⇒ = 1 + 
16000  100  ⇒
21
= 1+
R
P × R ×T P × 5×2 P
13. (c) SI = = =
⇒1 +
R
=
21
∴R = 5%
20 100
100 100 10 21 R
100 20 ⇒ − 1=
According to the question, 20 100
Simple and Compound Interests 47

R 1 156 52 13 On dividing Eq. (ii) from Eq. (i), we get


⇒ = ∴ R = 5% 156 = 8 × r × 3, = r, = r, %=r
100 20 3× 8 8 2 676  r 
= 1 + 
∴ New rate per cent = 5 + 5 = 10% If the rate of interest is increased by 4%. New 650  100 
2 13 26 × 100
Rate of interest = +4
∴ Amount = 400  1 +
10  ⇒ r= = 4%
 2
 100  65
So, amount = P + SI From Eq. (i), we get
11 11
= 400 × × = ` 484
800 ×  + 4 × 3
13
650 = x  1 +
10 10 4 
2  
25. (a) Let the sum = x = 800 +  100 
100 650 × 25
Then, CI when compounded half yearly 21 ⇒ x= = ` 625
 4  = 800 + 8 × × 3 = 800 + 4 × 21 × 3 26
=  x ×  1 +
10  4641 2
 − x = x 33. (b) Simple interest = Amount – Principal
 100  10000 = 800 + 12 × 21 = 800 + 252 = ` 1052
  = ` 2641.20 − ` 1860
CI when compounded annually 29. (d) A certain sum amount to 1725 at 15%
per annum and in the same time the sum = `781.20
 2  11
=  x ×  1 +
20  Then,
 − x = x becomes 1800 at 20% per annum.
P× r×t
 100 
  25 We know that, Amount = P + SI 781.20 =
100
∴According to the question, According to the question,
1860 × 12 × t
P × 20 × T   P × 15 × T  ⇒ 781.20 =
P + 
4641 11
x− x = 964  −P+   100
10000 25  100   100 
781.20 × 100
4641 − 4400  ⇒ t =
⇒ x   = 964
⇒ 1800 − 1725 1860 × 12
 10000  Since, time is same for both so,
42 1
964 × 10000 20P 15P ⇒ t = or 3 yr
∴ x= = 40000 − = 75 , 5P = 75 × 100, P = ` 1500 12 2
241 100 100
2 34. (c) If the principal be ` x, then
26. (b) If the sum = P
30. (d) Using the formula, D = P × 
r 
 simple interest = ` (770 − x )
∴According to the question,  100 
SI × 100
n n
Here, P = 2000, r = 6% ∴ Principal =
P  1 +
20   6 Time × Rate
 > 2P ⇒   > 2 2
 100   5
D = 2000 × 
6  (770 − x ) × 100
∴  ⇒ x=
This is true for n = 4  100  4 × 10
As,  × × ×  > 2
6 6 6 6 36
= 2000 × = ` 7.20 ⇒ 2 x = (770 − x ) × 5
 5 5 5 5 10000
⇒ 2 x + 5 x = 770 × 5
Hence, n = 4 yr 31. (d) Here, p1 = 1000, p2 = 3000, rate = r%
⇒ 7 x = 770 × 5
  1 + 10  − 8000
2
t 1 = 4 yr,
27. (a) CI= `  8000 ×    770 × 5
 100  t 2 = 2 yr, ∴ x= = ` 550
  7
Sum of SI = ` 900  T 
 
35. (b) CI = P   1 +
11 11
P× r ×t R 
= `  8000 × × − 8000  − 1
 10 10  Using the formula, SI =  100 
100  
= ` ( 9680 − 8000) = ` 1680 p1 × t 1 × r p2 × t 2 × r
⇒ 900 = +  2 
= 6400   1 +
15 
 840 × 100  100 100  − 1
∴ Sum = `   = ` 3500  200 
 3×8  1000 × 4 × r 3000 × 2 × r  
⇒ 900 = +
QSI is half of CI  100 100   43  2 
900 = 6400   − 1
∴ SI = 1680 = 840 ⇒ 900 = 40r + 60r ⇒ r = % = 9%  
 40 
 2  100
32. (c) Suppose, principal amount = ` x 
= 6400 
1849 
− 1
28. (c) A sum of 800 becomes 956 in 3 yr
 1600 
We know that, A = P + SI
650 = x  1 +
r 
800 × r × 3
Then,  …(i) 6400 × 249
956 = 800 +  100  = = ` 996
100 2
1600
676 = x  1 +
r 
and  …(ii)
 100 
Self Study Guide SSC Constable (GD) Recruitment Exam

CHAPTER 11

Time and Work


Work is defined as the amount of job assigned or the Ex. 2 A man, a woman and a boy can do a work in 3
amount of job actually done. The work to be completed is days by working together. If a man and a boy can do
generally considered as one unit. that work in 6 days and 18 days respectively, then a
The following points are to be kept in mind while solving woman can do
questions based on time and work. it in
1. It is assumed that a person does same amount of work (a) 9 days (b) 21 days
each day. (c) 14 days (d) 27 days
2. Work and men are directly proportional. Sol. (a) Here,
3. Work and days are directly proportional 6 × y × 18
4. Men and days are inversely proportional 3=
6y + 18y + 6 × 18
Important Points Related to ⇒ 3 (24y + 108) = 108y
⇒ 72y + 324 = 108y
Time and Work as
324
⇒ y= =9
l If a person takes x days to complete a work, then the 36
1
work done in 1 day will be . Hence, a woman can do the same work in 9 days by working
x alone.
1
l If a person does th part of a work in one day, then
x Formula 3. If A and B can do a work in x days and y days
person will finish the work in x days. respectively. They started working together and after t days A
l If A is x% times more efficient than B to do a certain job, left the work, then time taken to finish the complete work will
1 y
then A will take times more time than B to do that be × (x − t ).
x x
work.
T Note If B leaves the work, then time taken to finish the complete
x
Formula 1. If A and B can do a work in x and y days work = × ( y − t ).
xy y
respectively, then they will complete the work in days
x+ y Ex. 3 A and B can do a work in 15 and 10 days
by working together. respectively. They started work by working together but
after 2 days due to some reasons B left the work and A
Ex. 1 A can do a work in 4 days while B can do that completed the remaining work. The complete work will
same work in 6 days. If they worked together, then that
be finished in
work would be completed in
4 3 2 2 (a) 10 days (b) 8 days
(a) 3 days (b) 2 days (c) 2 days (d) 3 days (c) 12 days (d) 15 days
5 5 5 5
Sol. (c) Q x = 15 days, y = 10 days and t = 2 days
Sol. (c) Q x = 4 and y = 6 x 15 × (10 − 2) 15 × 8
xy ∴ Required time = × (y − t ) = =
∴ Required number of days = y 10 10
x+ y
= 12 days
4 × 6 24 12 2
= = = =2
4 + 6 10 5 5 Formula 4. If M1 persons complete W1 work in d1 days by
working h1 hours per day earning R1 and M2 persons
Formula 2. If A, B and C can do a work in x, y and z days complete W2 work in d 2 days by working h2 hours per day
xyz m ×d × h m × d 2 × h2
respectively, then they will complete the work in earning R2 , then 1 1 1 = 2
xy + yz + zx W1R1 W2 R2
days by working together.
Time and Work 49

Ex. 4 15 men complete a work in 16 days. If 24 men 2×4


= × 3900
are employed, then the time required to complete that 2 ×3 + 3 × 4 + 4 ×2
work will be 8 8 × 3900
= × 3900 = = `1200
(a) 7 days (b) 8 days (c) 10 days (d) 12 days 6 + 12 + 8 26
Sol. (c) Let the work done be 1.
Here, M1 = 15, D1 = 16, W1 = W2 = 1, M2 = 24 and D2 = ?
Ex. 7 If 3 men or 4 women can reap a field in 43 days.
How long will 7 men and 5 women take to reap it?
Now, according to the formula,
(a) 3 days (b) 7 days (c) 12 days (d) 15 days
M1D1W2 = M2D2W1
Sol. (c) Given 3 m = 4 w
⇒ 15 × 16 × 1 = 24 × D2 × 1
Time taken by 3 m or 4w = 43 days
15 × 16 240
⇒ D2 = = = 10 days 4  28 + 15 43
24 24 Now, 7m + 5w =  × 7 + 5 w = w= w
3  3 3
Therefore, 10 days are required to complete the work.
43 3
∴Time taken by w = 43 × 4 × = 12 days
Formula 5. If A can do a work in x days and B can do y% 3 43
100 x
fast than A, then B will complete the work in
(100 + y )
days. Pipes and Cisterns
Problems on pipes and cisterns are based on the basic
Ex. 5 x can do a work in 16 days. In how many days
concept of time and work.
will the work be completed by y, if the efficiency of y is
60% more than that of x? Pipes are connected to a tank or cistern and are used to fill or
(a) 10 days (b) 12 days (c) 25 days (d) 30 days empty the tank or cistern.
Sol. (a) Here, x = 16 days and y = 60% faster Basic fundamental of this topic is similar to time and work.
100 x 100 × 16 1600 The only difference lies in the fact that here work can be
∴ Required days = = = = 10 days positive (filling the tank) as well as negative (emptying the
100 + y 100 + 60 160
tank) whereas in time and work, the work done is always
positive.
Formula 6. If A, B and C can do a piece of work in x, y
and z days, respectively and they received ` k as wages Inlet A pipe connected with a tank or cistern or a reservoir
by working together, then that fills it, is known as an inlet.
yz
Share of A = ` ×k Outlet A pipe connected with a tank or cistern or a
xy + yz + zx reservoir that empties it, is known as an outlet.
xz
Share of B = ` ×k T Note If a pipe empties a tank, then its work done is taken as
xy + yz + zx
negative.
xy
and share of C = ` ×k
xy + yz + zx Ex. 8 If two pipes A and B can fill a tank in 20 and 30
min separately. If both pipes are opened, then tank will
T Note If A and B can do a piece of work in x days and y days, be filled in
respectively and they received ` k as wages by working (a) 10 min (b) 12 min (c) 15 min (d) 25 min
together, then share of A Sol. (b) Q x = 20 min and y = 30 min
yk xk
=` and shares of B = ` ∴ Required time =
xy
x+ y x+ y ( x + y)
Wages are directly proportional to the work done and indirectly 20 × 30 20 × 30 60
= = = = 12 min
proportional to the time taken by the individual. 20 + 30 50 5

Ex. 6 A Completes a job in 2 days and B completes it Ex. 9 A tap fills the tank in 1 h and empties it in
in 3 days and C takes 4 days to complete it. If they work 30 min. If both pipes are opened together, then in how
together and get ` 3900 for the job, then how much many minutes will the tank be emptied?
amount does B get? (a) 60 min (b) 30 min (c) 40 min (d) 45 min
(a) ` 1800 (b) ` 1200 (c) ` 900 (d) ` 800 Sol. (a) x = 60 min , y = − 30 min
Sol. (b) Here, x = 2 days, y = 3 days and z = 4 days xy − 60 × 30
∴ Time taken = = = − 60 min
and k = ` 3900 x + y 60 + 30
xz
Now, amount of B = ×k T Note Here −ve sign shows that the tank is emptied.
xy + yz + zx
Self Study Guide SSC Constable (GD) Recruitment Exam

Practice Exercise
Time and Work 8. A and B together can complete a be allowed to go off so that the
1. Mohan can do a piece of work in work in 3 days. They started work may be finished in time?
33 days. What work will he do in 1 together but after 2 days. B left the
(a) 10 (b) 15
day? work. If the work is completed (c) 20 (d) 25
1 1 after, 2 more days, B alone could
(a) part (b) part do the work in how many days? 15. 36 work men are employed to
34 33
1 1 (a) 5 (b) 6 finish a certain work in 48 days.
(c) part (d) part But it is found that in 24 days only
39 41 (c) 7 (d) 10
2
2. 10 men can make 16 toys in 6 days 9. A and B can do a piece of work in work is done. How many more
72 days; B and C can do it in 120 5
working 3 h a day. In how many men must be taken to finish the
days; A and C can do it in 90 days.
days can 12 persons make 40 toys In what time can A alone do it? work in time?
working 8h a day? (a) 150 days (b) 120 days (a) 16 (b) 18
13 11 (c) 100 days (d) 80 days (c) 20 (d) 22
(a) 4 (b) 4
16 16 10. A can do a piece of work in 80 16. Two men A and B, started a job in
7 5 which A was thrice as good as B
(c) 4 (d) 4 days. He works at it for 10 days
16 16 and therfore took 60 days less than
and then B alone finishes the
3. A is twice as good a workman as B remaining work in 42 days. The B to finish the job. How many days
and together they finish a piece of two together could complete the will they take to finish the job, if
work in 18 days. In how many work in they start working together?
days will A alone finish the work? (a) 24 days (b) 25 days 1
(a) 20 days (b) 22 days
(a) 20 days (b) 25 days (c) 30 days (d) 35 days 2
(c) 23 days (d) 27 days (c) 25 days (d) 30 days
11. A and B can do a piece of work in
4. 2 men and 3 boys can do a piece of 5 days, B and C can do it in 7 days; 17. A group of labourers promise to do
work in 10 days while 3 men and A and C can do it in 4 days. Who a work in 12 days, but 5 of them do
2 boys can do the same work in 8 among these will take the least not trun up. If rest of the group do
days. In how many days can 2 men time, if put to do it alone? the work in 18 days, find the
and 1 boy do the same work? (a) A (b) B original number of men.
2 (c) C (d) Data inadequate (a) 15 (b) 25
(a) 12 days (b) 24 dyas
3 (c) 35 (d) None of these
1
1 12. A can do of the work in 5 days
(c) 14 days (d) 12 days 3 18. 20 women can do a job in 20 days.
2 After each day, a woman is
2
5. A can do a piece of work in 15 days and B can do of the work in replaced by a man and a man is
5
and B alone can do it in 10 days. 10 days. In how many days both A twice as efficient as a woman. On
B works at it for 5 days and and B together can do the work? which day does the job get
then leaves. A alone can finish the 3 4 completed?
(a) 7 (b) 8
remaining work in 4 5 (a) 14th day (b) 15th day
1 1 3 (c) 16th day (d) 11th day
(a) 6 days (b) 7 days (c) 9 (d) 10
2 2 8 19. If m men working m h per day, can
(c) 8 days (d) 9 days
13. A takes twice the time taken by B do m units of work in m days, then
1 n men working n h per day would
6. X can do of a work in 10 days. and thrice the time taken by C to
4 do a particular piece of work. be able to complete how many
Y can do 40% of the same work in Working together, they can units of work in n days?
1 n3 m3
40 days and Z can do of the complete the work in 2 days. Find (a) (b)
2
3 the number of days taken by A , B m n2
work in 13 days. Who will and C respectively to complete the m4 n4
(c) (d)
complete the work first? work alone. n2 m3
(a) X (b) Y (a) 12, 6 and 4
(c) Z (d) Both X and Z 20. There is sufficient food for 800
(b) 18, 9 and 6
(c) 24, 12 and 8
men for 62 days. After 56 days, 560
7. A and B can complete a work in men leave the place. For how many
8 days working together. B alone (d) 6, 3 and 2
days will the rest of the food last
can do it in 12 days. After working 14. 90 men are enagged to do a piece for the rest of the men?
for 4 days, B left the work. How of work in 40 days but it is found (a) 16 days
many days will A take to complete 2 (b) 20 days
the remaining work? that in 25 days, work is (c) 30 days
(a) 16 days (b) 18 days
3
(d) 15 days
(c) 20 days (d) 22 days completed. How many men should
Time and Work 51

Pipe and Cistern Previous Years‘ Questions (a) 9 days (b) 6 days
(c) 5 days (d) 3 days
21. Two pipes A and B can fill a tank in 26. 16 men are able to complete a 1
20 min and 30 men respectively. If piece of work in 12 days working 31. A can do of a work in 5 days and B
6
both pipes are opened together, the 14 hours a day. How long will 2
time taken to fill the tank is 28 men, working 12 hours a day, can do of the work in 8 days. In
(a) 50 min (b) 12 min take to complete the work? 5
(c) 25 min (d) 15 min [SSC Constable, 2015] how many days, can both A and B
(a) 7 days (b) 6 days together do the work?
22. A tap can fill a tank in one hour. (c) 10 days (d) 8 days [SSC Constable, 2012]
A second tap can empty it in 30 (a) 12 (b) 13
min. If both the taps operate 27. 8 men can paint a house in 5 days. (c) 15 (d) 20
simultaneously, how much time is How many men will be required to
32. A, B and C individually can do a
needed to empty the tank? paint the same house in 4 days?
[SSC Constable, 2013] work in 10 days, 12 days and 15
(a) 20 min (b) 60 min days, respectively. If they start
(a) 9 (b) 10
(c) 40 min (d) 45 min (c) 11 (d) 7 working together, then the number
23. 12 pumps working 6 h a day can 28. A and B together can do a work in 15 of days required to finish the work
empty a completely filled reservoir is [SSC Constable, 2012]
days. B alone can do the same work
in 15 days. How many such pumps (a) 16 days (b) 8 days
in 60 days. Then, the time taken by (c) 4 days (d) 2 days
working 9 h a day will empty the
same reservoir in 12 days? A alone to do the work (in days) is 33. Working 8 h a day, Anu can copy a
[SSC Constable, 2013]
(a) 15 (b) 9 (c) 10 (d) 12
(a) 30 (b) 45
book in 18 days. How many hours
24. A pump can fill a tank with water (c) 20 (d) 60 a day should she work so as to
in 2 h. Because of a leak in the finish the work in 12 days?
29. 45 men can complete a work in [SSC Constable, 2011]
1 16 days. Four days after they
tank it was taking 2 h to fill the (a) 12 (b) 10
3 started working, 36 more men (c) 11 (d) 13
tank. The leak can drain all the joined them. How many days will
water off the tank in they now take to complete the Answers
(a) 8 h (b) 7 h remaining work?
1 [SSC Constable, 2012] 1 b 2 b 3 d 4 d 5 b
(c) 4 h (d) 14 h
3 (a) 6 (b) 8 6 c 7 a 8 b 9 b 10 c
2 3
25. Having the same capacity 9 taps fill (c) 6 (d) 7 11 c 12 c 13 a 14 b 15 b
3 4
up a water tank in 20 min. How 16 b 17 a 18 b 19 a 20 b
many taps of the same capacity are 30. Some persons can do a piece of
21 b 22 b 23 c 24 d 25 b
required to fill up the same water work in 12 days. Two times the
number of such persons will do 26 d 27 b 28 c 29 c 30 d
tank in 15 min
(a) 10 (b) 12 half of the work in 31 a 32 c 33 a
(c) 15 (d) 18 [SSC Constable, 2012]

Hints & Solutions


1. (b) As we know that, if a person can do a 1 1 1 5 1
∴ + = 5. (b) Work done by B in 5 days = =
piece of work in x days, x 2 x 18 10 2
1 2+1 1 1 1
Then, person 1 days’s work = ⇒ = ∴ Remaining work = 1 − =
x 2x 18 2 2
Here, x = 33 days ⇒ x = 27 days A completes
1
work in 1 day
1 1 15
∴Required work done = part 4. (d) (2 M + 3B)’ s 1 day’s work =
10 1
33 ∴He will complete work in
2. (b) Given that M1 = 10, M 2 = 12, D1 = 6, 1 2
and ( 3M + 2 B)’ s 1 day’s work =
8 1 1
D2 = ? H1 = 3, H2 = 8, W1 = 16, W 2 = 40 = 15 × = 7 days
7 1 2 2
According to the formula, ⇒ M = ,B=
200 100 6. (c) Time taken to complete the work by
M1D1H1W 2 = M 2D2H2W1 X = 10 × 4
∴(2 M + 1B)’ s 1 day’s work
⇒ 10 × 6 × 3 × 40 = 12 × D2 × 8 × 16 = 40 days
=  2 ×
10 × 6 × 3 × 40 7 1  16 2
∴ D2 = + 1×  = =
 200 100  200 25 Time taken to complete the work by
12 × 8 × 16
5 × 3 × 5 75 100
= =
11
days = 4 days
Hence, 2 men and 1 boys together can finish Y = 40 × = 100 days
16 16 16 25 40
the work in days, i.e.
2 Time taken to complete the work by
3. (d) Let A does one work in x days Z = 13 × 3 = 39 days
1
Then, B will do the same work in 2 x days. = 12 days. Hence, Z will complete the work first.
2
52 Self Study Guide SSC Constable (GD) Recruitment Exam

7. (a) ( A + B)’ s 1 days work =


1 ∴( A + B + C )' s 1 day work 18. (b) 20 women 1 day's work =
1

=  + + 
8 1 1 1 1 20
B’s 1 day’s work =
1 2  5 7 4 Let the total work be 400 unit.
12 1 28 + 20 + 35  83 So, work done on first days = 20 unit
=  =
∴ A’s 1 days’ work 2 140  280 work done on second day
1 1 3–2 1 (19 w + 1m) = 21 unit
= − = = Hence, A' s 1 day work
8 12 24 24 83 1 43 Similarly, work done in 15 days
∴ A can complete the work in 24 days. = − =
280 7 280 = [20 + 21 + 22 ... + 34]
4 1 15
Now, B’s 4 day’s work = = B' s 1 day work =
83 1 13
− = = [2 × 20 + (15 − 1)]
12 3 280 4 280 2
1 2
Remaining work = 1 − = 83 1 55 = 15 × 27 = 405 units
3 3 C ' s 1 day work = − =
280 5 280 Hence, the work will be complete in 15 days.
As, time taken by A to complete the work = 24 19. (a) Let the required number of units of
Hence, C will take the least time.
days
12. (c) A does one work in 15 days. work = x
2
∴Time taken by A to do of the work B does one work in 25 days. According to the formula,
3
M1H1D1W 2 = M 2H2D2W1
(A+B)’s one day work = 
2 1 1
= × 24 = 16 days ∴ + 
 15 25  ⇒ m× m× m× x = n× n× n× m
3
5+ 3 8 m × n3 n3
8. (b) ( A + B)’s 2 days’ work = 2 × =
1 2 = = ∴ x = 3
= 2
3 3 75 75 m m
Hence, ( A + B) will complete one work in 20. (b) let the food lasts for x days.
A’s 2 days’ work =  1 −  =
2 1
75 3 ∴ M1 D1 = M 2 D2
 3 3 = 9 days
8 8 ⇒ 800 × 6 = x × 240

B’s 2 days’ work =  −  =
2 1 1
 3 3 3
13. (a) Let A completes the work in 6x days. 800 × 6
Then, B will take 3x days and C will take 2 x ⇒ =x
1 240
B’s 1 days’ work = days to complete the same work.
⇒ x = 20 days
6 1 1 1 1 6 1
∴ + + = ⇒ = 21. (b) x = 20min and y = 30min
∴B will take 6 days to complete the work alone. 6x 3x 2 x 2 6x 2
1 xy 20 × 30
9. (b) ( A + B)' s 1 day work = ⇒ x =2 Total taken time = =
72 Hence, A, B and C will take 12,6 and 4 days, x + y 20 + 30
( B + C )' s 1 day work =
1 respectively to complete this work alone. 20 × 30
= = 12 min
120 14. (b) According to the formula, 50
1 1
( A + C )' s 1 day work = M1D1W 2 = M 2D2W1 22. (b) One min’s work of first tap =
90 60
( 90 × 25) 1 −  = ( 90 − x ) × 15 ×
2 2
∴ 2( A + B + C )' s 1 day work 1
 3 3 and one min’s work of second tap = −
1 1 1 5+ 3+ 4 1 30
= + + = = 1
72 120 90 360 30 ⇒ 90 × 25 × = 10( 90 − x ) one min’s work of both tap
1 3 1 1 1− 2 − 1
⇒ ( A + B + C )' s 1 day work = = − = =
60 ⇒ 75 = 90 – x ∴ x = 90 − 75 = 15 60 30 60 60
1 1 1 15. (b) According to the formula,
∴ A' s 1 day work = − = Hence, it is clear that tank will be emptied in
60 120 120 M1D1W 2 = M 2D2W1 60 min.
( 36 × 24) 1 −  = ( 36 + x ) × 24 ×
Hence, A alone can complete the work in 2 2
23. (c) Pumps Hours Days
120 days.  5 5 12 6 15
10 1 3 2
10. (c) Work done by A in 10 days = = ⇒ 36 × 24 × = ( 36 + x ) × 24 × x 9 12
80 8 5 5 9: 6 
1 7 ⇒ 36 × 3 = 2( 36 + x )  ::12 : x
Remaining work = 1 − = work ⇒ 108 = 72 + 2 x 12 :15
8 8
∴ x =
36
= 18 men 6 × 15 × 12
7
work is completed by B in 42 days ⇒ x= = 10 days
2 9 × 12
8
16. (b) Let’s assume that A takes x days to 1
∴Whole work will be done by B in 24. (d) Work of pump in 1 h =
finish the job, then B will take 3x days.
 42 × 8  = 48 days 2
  ⇒ 3x − x = 60 ⇒ x = 30 days
 7 and work of pump with leak in 1 h = −
3
Work done by ( A + B) in one day 7
( A + B)’s one days work
= 
1 1 2
+  = and work of leak in 1 h
= 
1 1 1  30 90  45
+ = work −3 1 7 −1
 80 48  30 45 = + = − 6+ =
Hence, they together will do the job in days 7 2 4 14
Hence, both will finish the work in 30 days. 2
1 Hence, tank will emptied in 14 h.
11. (c) ( A + B)' s 1 day work = 1
= 22 days 25. (b) Taps Quantity
5 2
1 9 20
( B + C )' s 1 day work = 17. (a) Let the total number of workers be x,
7 x 15
then 20 x 9
( A + C )' s 1 day work =
1
12x = 18(x − 5) 15 :20 :: 9 :x ⇒ x = = 12
4 15
⇒ x = 15
Time and Work 53

26. (d) 16 men completes a work in 12 days So, the required days ∴ A can do whole work = 6 × 5 = 30 days
working 14 h a day. 1 60 2
= = = 20 Q B can do of work = 8 days
M1 = 16 men, D1 = 12 days, H1 = 14 1

1 4−1 5
15 60 5× 8
We have to find 28 men working 12 h a day ∴ B can do whole work = = 20 days
complete the work in D2 days. 29. (c) Given, 2
M 2 = 28 H2 = 12 D2 = ? M1 = 45, M 2 = 81, D1 = 12, D2 = ? ∴A and B together do the work
∴ M1D1 = M 2D2 20 × 30
We know that, = = 12 days
⇒ 45 × 12 = 81 × D2 20 + 30
M1D1H1 = M 2D2H2
45 × 12 32. (c) Given, x = 10, y = 12 and z = 15
16 × 12 × 14 = 28 × D2 × 12 ⇒ D2 =
81 ∴ Required number of days
16 × 12 × 14
= D2 20 2 x × y× z
28 × 12 ⇒ D2 = days or 6 days =
3 3 x × y+ y× z+ z× x
D2 = 8 30. (d) Suppose, initially x men do the piece 10 × 12 × 15
Hence, 28 men working 12 h a day completes of work. M1 = x 1, D1 = 12, w1 = 1, =
the work in 8 days. 10 × 12 + 12 × 15 + 15 × 10
1
27. (b) M1 = 8,W1 = W 2 = 1, D1 = 5, M 2 = x , D2 = 4 M 2 = 2 x , w 2 = , D2 = ? =
1800
= 4 days
2 450
Using the formula, M1D1 M 2D2
∴ = 33. (a) Days Working hours/day
M1 D1 H1 M 2 D2 H2 w1 w2
= 18 8
W1 W2 x × 12 2 x × D2
⇒ = 12 x
8× 5 x × 4 1 1
⇒ = ⇒ x = 10 12 8
1 1 2 ⇒ =
18 x
1 12
28. (c) The work of (A+B)of one day = ⇒ D2 = = 3 days ⇒ 12 x = 18 × 8
15 2 ×2 18 × 8
1 ⇒ x = = 12 h
B the work of one day = 1
31. (a)Q A can do of work = 5 days 12
60 6

CHAPTER 12

Time, Speed and Distance


The concept of time, distance and speed is related to a l To convert a speed given in km/h to m/s, multiply the speed
an object in motion. 5
by .
18
Speed l To convert a speed given in m/s to km/h, multiply the speed
The distance travelled or covered by a person or an 18
object in unit time, is called its speed. by .
5
Distance Covered
Speed = Relationship between Time, Distance and Speed
Time Taken
The unit of speed is m/s or km/h. Relationship between time, distance and speed is expressed by
Distance
Speed =
Time Time
The duration in hours, minutes or seconds spent to This expression shows that
cover a certain distance is called time. 1. Speed is directly proportional to distance. If the speed is
doubled, then distance travelled in the the same time, will
Distance also be doubled.
The length of path covered by a person or an object
2. Distance and time are directly proportional. If distance to
between two places, is known as distance. The unit of
be travelled is doubled, then the time taken would also be
distance is m or km.
doubled at the same speed.
54 Self Study Guide SSC Constable (GD) Recruitment Exam

3. Time is inversely proportional to speed. If the distance Relative Speed


remains the same and speed is doubled, then time taken
Relative Speed is the speed of a moving object in
to travel the same distance becomes half of the original
relation to other moving object. Let two objects are in
time taken at the original speed.
motion and their speeds are a km/h and b km/h
Following examples will help to clarify the above relationship. respectively, then
1 (a) Relative speed = ( a + b) km/h
Ex. 1 Arnav covers a distance of 150 km in 1 h. The
2 (if two objects are moving in opposite directions)
speed of Arnav is (b) Relative speed = ( a − b) km/h
(a) 50 km/h (b) 100 km/h (if two objects are moving in same direction and a > b)
(c) 1050 km/h (d) None of these
Sol. (b) Distance covered =150 km Ex. 4 Two persons are moving in the direction
1 3 opposite to each other. The speeds of the both
and time taken = 1 = h persons are 5 km/h and 3 km/h, respectively. Find
2 2
Distance Covered 150 150 × 2 the relative speed of the two persons in respect of
∴ Speed of Arnav = = = = 100 km/h each other.
Time taken 3 /2 3
(a) 15 km/h (b) 2 km/h
Ex. 2 If a man walks at the rate of 5 km/h, he misses a (c) 8 km/h (d) 4 km/h
train by only 7 min. However, if he walks at the rate of 6 Sol. (c) Required relative speed = 5 + 3 = 8 km/h
km/h, he reaches the station 5 min before the arrival of the
train.
Concept of Relative Speed in Motion of
Trains
what is the distance covered by him to reach the station?
(a) 4 km (b) 6 km (c) 8 km (d) 5 km 1. If two trains of length x km and y km are moving
Sol. (b) Let the distance travelled by d km and time taken be t in opposite directions at u km/h and v km/h,
hr. then time taken by the trains to cross each other
 7  x + y
Then, d = 5 × t +  …(i) = h
 60  u + v
 5
and d = 6 × t −  …(ii) 2. If two trains of length x km and y km are moving
 60 
in the same direction at u km/h and v km/h,
Now, using eq. (i) and (ii), we have where u > v), then time taken by faster train to
 7  5  x + y
5t +  = 6t − 
 60   60  cross the slower train =  h
u − v
35 30 65
⇒ 5t + = 6t − ⇒ t=
60 60 60 3. It two trains start at the same time from two
From Eq. (i), points A and B towards each other and after
 65 7  5 × 72 crossing they take a and b hours in reaching B and
∴ d =5  + = = 6 km
 60 60  60 A respectively. Then, A’s speed : B’s speed
= b: a
Average Speed
When a certain distance is covered at speed A and the same Ex. 5 A train 140 m long is running at 60 km/h. In
distance is covered at speed B, then the average speed during how much time will it pass a platform 260 m long?
2AB (a) 60s (b) 24s (c) 18s (d) 10s
the whole journey is given by .
A+B 5 50
Sol. (b) Speed of train = 60 × = m/s
18 3
Ex. 3 A person goes to Delhi from Mumbai at the speed Distance covered by train in crossing the platform
of 60 km/h and comes back at the speed of 50 km/h. = (140 + 260) = 400 m
Calculate the average speed of the person for the entire trip. 3
(a) 54.54 km/h (b) 50.5 km/h ∴ Time taken = 400 × = 24 s
50
(c) 64 km/h (d) 12 km/h
2 AB 2 × 60 × 50 Ex. 6 A man sets out to cycle from point P to Q and
Sol. (a) Average Speed = =
A+B 60 + 50 at the same time, another man starts to cycle from
[Q A = 60 km/h, B = 50 km/h] point Q to P. After passing each other, they complete
6000 their journeys in 9 h and 4 h respectively. What is the
= = 54.54 km/h ratio of speeds of 1st man to that of 2nd man?
110
(a) 3 : 2 (b) 9 : 4 (c) 4 : 9 (d) 2 : 3
Time, Speed and Distance 55

Sol. (d) Q a = 9 h, b = 4 h  u + v
l Speed of boat in still water =   km/h
1st man' s speed b 4 2  2 
∴ = = = =2:3
2nd man' s speed a 9 3  u − v
l Speed of stream =   km/h
 2 
Boats and Streams
The problems of boats and streams are also based on the Ex. 7 A man can row 15 km/h in still water. If it takes
basic relation of speed, distance and time him twice as long to row up as to row down the river.
Distance Find the rate of stream.
i.e. Speed = (a) 8 km/h (b) 4 km/h (c) 6 km/h (d) 5 km/h
Time
Sol. (d) Ratio of time taken upstream and downstream = 2 : 1
In these questions, the direction along the stream (water) is
called downstream and direction against the stream is Hence, ratio of speed upstream and speed downstream
would be 1 : 2
called upstream.
Let speed upstream be x km/h. Then, speed downstream
If the speed of a boat in still water is x km/h and the speed would be 2x km/h.
of the stream is y km/h, then u + v x + 2x 3x
Speed in still water = = =
l Downstream speed = ( x + y) km/h 2 2 2
3x
l Upstream speed = ( x − y) km/h Given, = 15 ⇒ x = 10 km/h
From the above relationship we conclude, if the 2
downstream speed is u km/h and upstream speed is v ∴ Downstream speed = 2 × x = 2 × 10 = 20 km/h
u − v 20 − 10 10
km/h then, ∴ Speed of the stream = = = = 5 km/h
2 2 2

Practice Exercise
Concept of time, speed 90 km/h and 80 km/h respectively. 10. A certain distance is covered at a
and distance How many kilometres from Delhi certain speed. If half of this distance
will the two trains be together? is covered in 4 times of the time,
1. Convert 90 km/h into m/s (a) 360 km (b) 320 km find the ratio of the two speeds.
(a) 5 m/s (b) 25 m/s (c) 270 km (d) 280 km (a) 1 : 8 (b) 1 : 4 (c) 4 : 1 (d) 8 : 1
(c) 35 m/s (d) 15 m/s
7. Prabhat covers a distance in 11. A man covers half of his journey at
2. Convert 30 m/s into km/h 40 min, if he drives at a speed of 6 km/h and the remaining half at 3
(a) 100 km/h (b) 105 km/h 60 km/h on an average. Find the km/h. Find his average speed.
(c) 101 km/h (d) 108 km/h speed at which he must drive at (a) 3 km/h (b) 4 km/h
3. A person riding a bike crosses a to reduce the time of the journey (c) 4.5 km/h (d) 9 km/h
bridge with a speed of 54 km/h. by 25%.
(a) 60 km/h (b) 70 km/h 12. The speeds of A and B are in the
What is the length of the bridge, if
(c) 75 km/h (d) 80 km/h ratio of 3 : 4 . A takes 20 min more
he takes 4 min to cross the bridge?
than the time taken by B to reach a
(a) 3600 m (b) 2800 m 8. Without stoppage, a train travels a
(c) 3500 m (d) 4500 m
destination. In what time does A
certain distance with an average
reach the destination?
4. A man covered a distance of 12 km speed of 60 km/h and with
1 2 2
in 90 min by cycle. How much stoppage, it covers the same (a) 1 h (b) 2 h (c) 1 h (d) 2 h
3 3 3
distance will he cover in 3 h, if he distance with an average speed of
rides the cycle at a uniform speed? 40 km/h. On an average, how 13. A is twice as fast as B and B is
(a) 36 km (b) 24 km many min per hour does the train thrice as fast as C. The journey
(c) 30 km (d) 27 km stop during the journey? covered by C in 56 min will be
(a) 20 min/h (b) 15 min/h covered by A in
3
5. Walking at of his normal speed, (c) 10 min/h (d) 12 min/h 1 1
4 (a) 5 min (b) 2 min
9. Salabh, during his journey, travels 3 3
Abhishek is 16 min late in
for 20 min at a speed of 30 km/h, 1 1
reaching his office. The usual time (c) 7 min (d) 9 min
taken by him to cover the distance another 30 min at a speed of 3 3
between his home and his office is 50 km/h, next 1 h at a speed of 14. A man is walking at a speed of 9
(a) 48 min (b) 60 min 50 km/h and last 1 h at speed of 60 km/h. After every km he takes rest
(c) 42 min (d) 62 min km/h. What is the average speed of for 9 min. How much time will he
the entire journey? take to cover a distance of 27 km?
6. Two trains for Bhopal leave Delhi (a) 51.18 km/h (b) 48 km/h
at 9 am and 8 : 30 am and travel at (a) 6 h (b) 6 h 45 min
(c) 63 km/h (d) 39 km/h
(c) 6 h 54 min (d) 6 h 35 min
56 Self Study Guide SSC Constable (GD) Recruitment Exam

15. A man covers a certain distance (a) 38 min (b) 39 min


between his house and office on (c) 45 min (d) 32 min
Previous Years’ Questions
scooter, having an average speed of 31. A student starting from his house
23. Two trains, one 160 m and the
60 km/h he is late by 20 min. 1
other 140 m long are running in walks at a speed of 2 km/h and
However, with a speed of 80 km/h 2
opposite directions on parallel
he reaches his office 10 min earlier. reaches his school 6 minutes late.
Find the distance between his rails, the first at 77 km an hour the
other at 67 km an hour. How long Next day starting at the same time
house and office. he increases his speed by 1 km/h
(a) 120 km (b) 90 km will they take to cross each other?
1 and reaches 6 minutes early. The
(c) 80 km (d) 60 km (a) 7 s (b) 7 s (c) 6 s (d) 10 s distance between the school and
2
16. Ram travels at the rate of 3 km/h his house is [SSC Constable, 2015]
and he reaches 15 min late. If he 24. A train 150 m long, passes a pole in 3 1
(a) 1 km (b) 3 km (c) 6 km (d) 4 km
travels at the rate of 4 km/h, he 15 s. It also passes another train of 4 2
reaches 15 min earlier. The the same length travelling in
opposite direction in 12 s. The 32. A plane can cover 6000 km in 8
distance Ram has to travel is
speed of the second train is hours. If the speed is increased by
(a) 1 km (b) 6 km (c) 7 km (d) 12 km
(a) 45 km/h (b) 48 km/h 250 km/h, then the time taken by
17. A man walks to a town at the rate (c) 52 km/h (d) 54 km/h the plane to cover 9000 km is
1 [SSC Constable, 2015]
of 5 km/h and rides back at the
2 25. Two trains, one from station A to (a) 9 h (b) 8 h (c) 6 h (d) 5 h
rate of 10 km/h. How far he station B and the other from B to A
33. A train 100 m long, running at
walked, if the total time of the start simultaneously. After meeting,
36 km/h, takes 25 s to pass a bridge.
journey is 6 h 12 min? the trains reach their destinations
The length of the bridge is
(a) 31 km (b) 29 km after 9 h and 16 h respectively. The [SSC Constable, 2013]
(c) 22 km (d) 17 km ratio of their speeds is (a) 100 m (b) 80 m
(a) 2 : 3 (b) 4 : 3 (c) 6 : 7 (d) 9 : 16 (c) 150 m (d) 120 m
18. A train leaves Manipur at 6 am and
reaches Dispur at 10 am. Another 34. The speed of a boat along the
train leaves Dispur at 8 am and Boat and Streams stream is 12 km/h and against the
reaches Manipur at 11 : 30 am. At 26. A man rows a boat 18 km in 4 h stream is 8 km/h. The time taken by
what time do the two trains cross downstream and return upstream the boat to sail 24 km in still water
each other? in 12 h. The speed of the stream is [SSC Constable, 2012]
(a) 7 : 56 am (b) 7 : 56 pm
(in km/h) is (a) 2 h (b) 3 h (c) 2.4 h (d) 1.2 h
(c) 8 : 56 am (d) 8 : 56 pm
(a) 1 (b) 1.5 (c) 2 (d) 1.75 35. In covering a distance of 30 km,
19. The average speed of a car is 75
km/h. The driver first decreases its 27. A man can swim 3 km/h in still Abhay takes 2 h more than Sameer.
average speed by 40% and then water. If the velocity of the stream If Abhay doubles his speed, then
increases it by 50%. What is the is 2 km/h, the time taken by him to he would take 1 h less than
new average speed now? swim to a place 10km upstream Sameer. Abhay’s speed
(a) 67.5 km (b) 60 km and back is (in km/h) is [SSC Constable, 2012]
1 1 (a) 5 (b) 6 (c) 6.25 (d) 7.5
(c) 90 km (d) 60.5 km (a) 9 h (b) 10 h (c) 12 h (d) 8 h
3 3
20. Two cars run to a place at the 36. A train of length 500 ft crosses a
speeds of 45 km/h and 60 km/h 28. A boat goes 6 km an hour in still platform of length 700 ft in 10 s.
respectively. If the second car takes water, but takes thrice as much The speed of the train is
5 h less than the first car for the time in going the same distance [SSC Constable, 2011]
journey, find the length of the against the current. The speed of (a) 70 ft/s (b) 85 ft/s
journey. the current (in km/hour) is (c) 100 ft/s (d) 120 ft/s
(a) 600 km (b) 300 km (a) 4 (b) 5 (c) 3 (d) 2 37. The speed of two cars are in the
(c) 900 km (d) 120 km ratio 5 : 4. The ratio between the
29. A man can row at 5 km/h in still
21. A man takes 8 h 32 min in walking water. If the velocity of current is 1 times taken by them to travel the
to a certain place and riding back. km/h and it takes him 1 h to row same distance is
He would have gained 2 h 14 min [SSC Constable, 2011]
to place and come back, how far is
by riding both ways. How long (a) 5 : 4 (b) 6 : 4 (c) 6 : 5 (d) 4 : 5
the place?
would he take to walk both ways? (a) 2.5 km/h (b) 3 km/h
(a) 10 h (b) 10 h 36 min
Answers
(c) 2.4 km/h (d) 3.6 km/h
(c) 10 h 46 min (d) 10 h 40 min 1 b 2 d 3 a 4 b 5 a
30. The speed of a motor-boat and the 6 a 7 d 8 a 9 a 10 d
22. Two cars A and B are running speed of the current of water is in 11 b 12 a 13 d 14 c 15 a
towards each other from two the ratio 36: 5. The boat goes along 16 b 17 c 18 c 19 a 20 c
different places 88 km apart. If the with the current in 5 h 10 min. It 21 c 22 d 23 b 24 d 25 b
ratio of the speeds of the cars A will come back in
26 b 27 c 28 c 29 c 30 c
and B is 5 : 6 and the speed of the 31 a 32 a 33 c 34 c 35 a
(a) 5 h 50 min (b) 6 h 36 d 37 d
car B is 90 km/h, after what time
(c) 6 h 50 min (d) 12 h 10 min
will they meet each other?
Hints & Solutions
1. (b) 90 km/h =  90 ×  m/s
5 10. (d) Let x km is covered in y h. 15. (a) Let the distance be x km.
 18  Then, original 1
Time difference = (20 + 10) min = 30 min = h
= 5 × 5 = 25 m/s x
speed = km/h
2
x x 1
2. (d) 30 m/s =  30 ×  km/h
18 y Then, − =
 5 x 60 80 2
Again, km is covered in 4y h. 4x − 3x 1
= 6 × 18 = 108 km/h 2 ⇒ = ⇒ x = 120 km
x 1 240 2
3. (a) Given that, speed = 54 km/h ∴ New speed =  ×  km/h 16. (b) Ratio of speeds = 3 : 4
5 2 4y
= 54 × = 3 × 5 = 15 m/s
18  x  ∴ Ratio of time = 4 : 3
=   km/h
Time = 4 min = 4 × 60 = 240 s  8y According to the question,
30
∴Required length = 15 × 240 = 3600 m x x 1 4x − 3x = ⇒ x = 1/ 2
Ratio of speeds = : = 1: = 8 : 1 60
Distance y 8y 8
4. (b) As we know that, speed = ∴when Ram travels at 3 km/h, he takes 2h.
Time 11. (b) According to the formula,
12 2 AB ∴Required distance = 6 km
12 × 60 Average speed = 17.(c) Let the distance between two points be
= 90 = = 8 km/h A+ B
60 90 x km.
Here, A = 6 km/h, B = 3 km/h 2x x 12 31
∴ Distance covered in 3 h = Speed × Time ∴ + =6+ =
∴Required average speed
= 8 × 3 = 24 km 11 10 60 5
2 × 6 × 3 36 31x 31
3 = = = 4 km/h ⇒ = ⇒ x = 22 km
5. (a) If the speed becomes th of the normal 6+ 3 9
4 110 5
4
speed, then time will become of the usual 12. (a) Let time taken by A = x h 18. (c) Let the distance between Manipur and
3 Dispur be 280 km.
Then, time taken by B =  x −
20 
4 h
time. Let the usual time be T, then T − T = 16  60  Then, speed of 1st train = 70 km/h
3 And speed of 2nd train = 80 km/h
=  x −  h
1
min
 3 Now, distance travelled by 152
∴ T = 48 min train in 2 h = 140 km
Ratio of speeds = Inverse ratio of time taken
6. (a) Distance travelled by the train at 80 ∴140 km is travelled by both trains with
 x − 1
km/h in 30 min = 40 km. This distance of 40   140 14
3  3 3x − 1 3 relative speed of 150 km/h = = h
km is to be covered with a relative speed of ∴ = ⇒ = 150 15
( 90 − 80) = 10 km/h 4 x 3x 4
Hence, required time
40 ⇒ 12 x − 4 = 9x ⇒ 12 x − 9x = 4 14
∴Time taken to cover this distance = =4h 4 1 = 8 : 00 am + × 60 = 8 : 56 am
10 ∴ x = h =1 h 15
3 3
Hence, the trains will together after 4 × 90 19. (a) Required average speed
IInd Method 60 150
= 360 km from Delhi = 75 × ×
A’s speed : B’s speed = 3 : 4 100 100
7. (d) Distance required to be travelled ∴ A’s time : B’s time = 4 : 3 3
= 60 ×
40
= 40 km = 45 × = 22 . 5 × 3
[As, speed is inversely proportional to time.] 2
60
Time available to cover this distance
According to the question, = 67. 5 km/h
4x − 3x = 20 ⇒ x = 20 min 20. (c) Let the distance be x km
= 40 × 075. = 30 min
20 1
40 ∴Time taken by A = 4x = 4 × =1 h Then, time taken by first car =
x
∴ Speed = = 80 km/h 60 3
30 45
13. (d) Let time taken by A = y min x
60 and time taken by second car =
8. (a) Due to stoppages the train travels 20 km Let speed of C = x 60
x x
less in one hour and the time taken to travel 20 Then, speed of B = 3x Given, − =5
km is the time of the train taken at stoppages. ∴ Speed of A = 6x 45 60
4x − 3x
Therefore, time taken to cover 20 km Now, ratio of speeds of A and C =5
180
=
20
× 60 = 20 min/h = Ratio of time taken by C and A ⇒ x = 900 km
60 6x 56
6x : x = 56 : y ⇒ = 21. (c) Walking time + Riding time
Total distance x y
9. (a) Average speed = = 8 h 32 min …(i)
Total time 56 1
∴ y= = 9 min Also, 2 × Riding time = 6 h 18 min …(ii)
1 1
30 × + 50 × + 50 × 1 + 60 × 1 6 3 From Eqs. (i) and (ii), we get
= 3 2 14. (c) Since, he is taking a rest after every
1 1 Walking time = (8 h 32 min) − (3 h 9 min)
+ + 1+ 1 one km. Therefore, time consumed in taking 23
3 2 = 5 h 23 min = 5 + = 5.38 h
rest in 27 km 60
6(10 + 25 + 50 + 60)
= = 26 × 9 = 234 min = 3. 9 h Hence, time taken to walk both ways
17
Time taken to cover 27 km without rest = 3 h = 2 × 5.38 = 1076
. h
= 51.18 km/h (approx)
Total time taken = 3 + 3.9 = 6.9 h or 6 h 54 min or 10 h 46 min (approxi)
58 Self Study Guide SSC Constable (GD) Recruitment Exam

22. (d) Ratio of speeds of car A and B = 5 : 6 Hence, time taken to cover 10 km Distance
Therefore, Time =
10 10 Speed
Speed of car B = 90 km/h = +
5 1 9000
∴Speed of car A = 90 ×
5
= 75 km/h Time taken by plane =
6 = 2 + 10 = 12 h 250 +
6000
Hence, required time 28. (c) Let speed of current = x km/h 8
88 88 9000 × 8
= h= h ∴Speed of boat upstream =
90 + 15 165 2000 + 6000
= ( 6 − x ) km/h
9000 × 8 9× 8
88 and speed of boat downstream = = = 9h
= × 60 min = 32 min 8000 8
165 = ( 6 + x ) km/h
33. (c) Speed to train = 36 km/h
23. (b) Relative speed of trains Let y km be the required distance.
5
= (77 + 67 ) km/h 3y y = 36 × = 10 m/s
∴ = 18
5
= 144 × m/s = 40 m/s 6+ x 6− x
Total distance
18 ⇒ 3( 6 − x ) = 6 + x = Length of train + Length of bridge
and total distance travelled = Length of first ⇒ 18 − 3x = 6 + x
train + Length of second train = (100 + x ) m
⇒ x=3 Time taken = 25 s
= 160 + 140 = 300 m Hence, speed of current = 3 km/h Distance
Total distance travelled ∴ Speed =
∴ Time taken = 29. (c) Let x km be the required distance. Time
Relative speed
Now, speed of boat upstream 100 + x
300 1 10 =
= =7 s = 5 − 1 = 4 km/h 25
40 2 and speed of boat downstream 250 = 100 + x
24. (d)Q In crossing of a pole, distance = 5 + 1 = 6 km/h ∴ x = 250 − 100 = 150 m
travelled = 150 m x x 2 x + 3x 34. (c) Suppose, speed of boat = x km/h and
∴ + = 1⇒ =1
and time taken = 15 s 6 4 12
speed of stream = y km/h
Travelled distance 12
∴ Speed of first train = ⇒ x= = 2.4 km/h Then, x + y = 12 …(i)
Time taken 5
150 x − y=8 …(ii)
= = 10 m/s 30. (c) Let speed of boat and current be 36 x
On solving Eqs. (i) and (ii), we get
15 km/h and 5 x km/h, respectively.
and let speed of second train = x m/s 2 x = 20
∴Distance travelled by boat downstream ⇒ x = 10 km/h
150 × 2 10 41x × 31
x + 10 = ⇒ x + 10 = 25 = ( 36 x + 5 x ) × 5 = km 24
12 60 6 ∴ Required time = = 2 .4 h
18 10
⇒ x = 15 × km/h = 54 km/h Hence, time taken by boat upstream
5 35. (a) Let the speed of Abhay be x km/h and
41x × 31 41x × 31 41
= = = Sameer be y km/h, respectively.
25. (b)Q a = 9, b = 16 6 ( 36x − 5x ) 6 × 31x 6 30 30
Speed of 1st train b Then, − =2 ...(i)
∴ = 5 50 x y
=6 =6 h = 6 h 50 min
Speed of 2nd train a 6 60 30 30
and − =1 ...(ii)
16 31. (a) Let the distance between the school y 2x
= = 4: 3
9 and the house is x.
On adding Eqs. (i) and (ii), we get
26. (b) Speed of boat downstream According to the question 30 30
− =3
18 x x 12 10x 10x 1
x= = 4.5 km/h − = , − = x 2x
2 − 1
30
4 2.5 3.5 60 25 35 5
⇒  =3
and speed of boat upstream, 2x 2x 1 14x − 10x 1  2x 
− = , =
18 5 7 5 35 5
y= = 15
. km/h ⇒ 30 = 6x
12 35 7 3 ⇒ x = 5 km/h
4x = , 4x = 7 , x = , x = 1 km
Speed of stream 5 4 4
x − y 4.5 − 15. 36. (d) Speed of train
= = 32. (a) A plane can cover 6000 km in 8 h. Length of (train + platform)
2 2 =
Distance Time taken to cross
3 We know that, Speed =
= = 1. 5 km/h. Time  500 + 700 
2 =  ft/s = 120 ft/s
So, speed of the plane is
6000
km/h  10 
27. (c)∴Speed of man = 3km/h 8
and speed of stream = 2 km/h
37. (d) Time and speed are inversely
If the speed of the plane is increased by 250 proportional.
∴Speed of man downstream km/h then it becomes
∴ Required ratio = 4 : 5
= 3 + 2 = 5km/ h
=  250 +
6000 

and speed of man upstream  8 
= 3 − 2 = 1km/ h
We have to find the time taken by plane to
cover 9000 km
Mensuration 59

CHAPTER 13

Mensuration
Mensuration is the branch of mathematics which deals l Perimeter = a + a + b = 2 a + b
with the study of measurement of geometric magnitudes
like length, areas and volume for different geometric
shapes.
a a
We classify this chapter into two topics h
1. 2D-Mensuration 2. 3D-Mensuration

Features of 2D-Figure b/ 2 b/ 2
b
Area Area is the total space enclosed by the boundary of a
2-dimensional figure. It is measured in square unit. e.g.The perimeter of an isosceles triangle is 32 cm while
Perimeter Sum of the sides of any enclosed figure is called its equal sides together measure 18 cm. Find the third
the perimeter of that figure. side and each of the equal sides.
Sol. Let the third side be x cm.
Triangle According to the question, x + 18 = 32
Triangle is a plane figure with three sides that are joined to ∴ x = 32 − 18 = 14 cm
form three angles. Sum of these angles are 180°. 18
Each equal side = = 9 cm
Equilateral Triangle 2
It has all three sides equal and each angle is of 60°. Scalene Triangle
3 3 2 It has three unequal sides.
l Area = (Side) 2 = a
4 4 Let a, b and c are the sides of the triangle,
a a
3 3 then
l Height = (Side) = a h a c
2 2 l Area = s ( s − a) ( s − b) ( s − c)
l Perimeter = 3 Side = 3a a a+b + c
Where, s = b
2
e.g. The perimeter of an equilateral triangle is 45 cm. Find l Perimeter = a + b + c
its area. e.g.Find the perimeter of a triangle with sides equal to 3
Sol. Given that, perimeter = 45 cm cm, 8 cm and 5 cm.
⇒ 3 a = 45 [∴ a = side] Sol. Required perimeter = Sum of the sides
∴ a=
45
= 15 cm = ( 3 + 8 + 5) cm = 16 cm
3 e.g.Find the area of a triangle whose sides are 26 cm, 28
3 2 3 225 3 cm and 30 cm.
∴ Area = a = × 15 2 = sq cm
4 4 4 Sol. Given that, a = 26 cm, b = 28 cm and c = 30 cm
Isosceles Triangle a + b + c 26 + 28 + 30 84
∴ s= = = = 42
It has any two sides and two angles equal and altitude 2 2 2
bisects the base. ∴ ( s − a) = 42 − 26 = 16 cm
Let a = Each of two equal sides and b = Third side, then ( s − b) = 42 − 28 = 14 cm
b ( s − c) = 42 − 30 = 12 cm
l Area = 4a 2 − b 2
4
∴ Area = s ( s − a) ( s − b) ( s − c) = 42 × 16 × 14 × 12
2
b 1
l Height = a2 −   = 4 a2 − b 2 = (14 × 3) × 16 × 14 × ( 4 × 3)
 2 2
= 14 × 4 × 2 × 3 = 336 sq cm
60 Self Study Guide SSC Constable (GD) Recruitment Exam

Right Angled Triangle Parallelogram


It is a triangle whose one angle is equal to Parallelogram is a four-sided figure having opposite sides
90°. parallel and opposite angles equal.
h
Let p = Perpendicular, b = Base p l Area of parallelogram = Base × Height = b × h
and h = Hypotenuse, then D C
1 1
l Area = × Height × Base = × p × b b
2 2
h
l Perimeter = p + b + h
Note Here, h 2 = p 2 + b 2 , which is known as Pythagoras theorem. A b B

l Area of parallelogram
Quadrilaterals
= Diagonal × perpendicular on the diagonal
A figure enclosed by four sides is called a quadrilateral. A
When diagonal and the perpendicular on the diagonal are
quadrilateral has four angles and sum of these angles is
given.
equal to 360°.
Rectangle Rhombus
Rhombus is a parallelogram that has four equal sides but
Rectangle is a four-sided figure with four angles and
no right angles.
opposite sides are equal.
If ‘a’ is the side of the rhombus and d1 and d2 are diagonals.
D C Then,
) 1
l (d breadth l Area of rhombus = × d1 × d2
gona 2
dia (b)
l Perimeter of rhombus = 4 × a
A length (l) B
a
l Area of rectangle = Length × Breadth = l × b d1 d2
l Perimeter of rectangle = 2 ( l + b) a a
l Length of the diagonal = l + b 2 2

a
l Area of 4 walls of a room = 2 ( l + b) × h
2 2
Area of Paths and Verandah d  d 
l Side of rhombus =  1  +  2 
If l = Length, b = Breadth and p = Width of path  2  2
l Trapezium
p
Trapezium is a four-sided figure in which no two sides are
parallel.
p p b D b C

l Area of path outside = ( l + b + 2 p) 2 p E


A a B
l Area of path inside = ( l + b − 2 p) 2 p
Area of cross roads = ( l + b − p) p, l
1
Area of trapezium = (Sum of parallel sides) × Height
where l = Length, b = Breadth, p = Width of path 2
Square 1
= ( a + b) × h
Square is a four-sided figure in which all the four sides are 2
equal and all the four angles are right angles. If a is the side Where a, b are sides.
of the square. Then,
D C Circle
l Perimeter of square = 4 × Side = 4 a al
on It is a plane figure enclosed by a line on, which every point
l Area of square = ( a) 2 sq unit di
ag
is equally distant from a fixed point (centre) inside the
A B
l Diagonal of square= 2 × Side = 2 × a curve.
= 2 × Area
Mensuration 61

l Circumference (Perimeter) of Circle = 2πr Surface Area The sum of area of at all the surfaces of the
where r is the radius r
object is called surface area. It is measured in m 2 , cm 2 .
l Area of circle = πr 2 O
A

l Diameter = 2 × radius Cube


A solid body having 6 equal faces with equal length,
Ex. 1 The perimeter of a rectangular field is 480 m and breadth and height is called a cube.
the ratio between the length and breadth is 5 : 3. Find Thus, each face of a cube is a square. Let

a
the area of the field. side or edge = a. Then , a
(a) 12000 sq m (b) 13500 sq m l Volume of a cube = a 3
(c) 14000 sq m (d) 13000 sq m
Sol. (b) Let the length = 5x and breadth = 3x, then l Total surface area = 6 a2
2(5x + 3x) = 480 or x = 30 l Length of diagonal = 3a, a
∴ l = 150 m and b = 90 m Where a is side of the cube.
∴ Area = 150 × 90 = 13500 sq m
Cuboid
Ex. 2 If the diagonal of a square is doubled to make
the diagonal of another square. Find the area of the new A rectangular solid body having 6 rectangular faces, is
square. called cuboid.
(a) 4 × Old area (b) 2 × Old area Let length =l, breadth = b

b
(c) 3 × Old area (d) 5 × Old area and height = h. Then,
1 2 h
Sol. (a) Let the diagonal be a.Then, area = a l Volume of cuboid = lbh
2 l Total surface area
New diagonal = 2a = 2( lb + bh + lh)
1 1
∴ New area = (2a) 2 = 2a 2 = 4 × a 2 l Length of the diagonal
2 2 l
∴ New area = 4 × Old area = l2 + b 2 + h 2
Ex. 3 If the side of a square is increased by 5 cm, the l Area of 4 walls or lateral surface area = 2( l + b) h, where l,
area increased by 165 sq cm. Find the side of the square. b and h are length, breadth and height respectively.
(a) 20 cm (b) 16 cm (c) 14 cm (d) 10 cm Cone
Sol. (c) Let, original side = x cm Cone is a solid or hollow body with a round based and
Then ( x + 5) 2 − x2 = 165 ⇒ 10 x = 140 pointed top.
∴ x = 14 Let r = radius of base, h = height and l = slant height
∴ Side of the square = 14 cm
Ex. 4 If the area of a rhombus is 15 sq cm and the
l
length of one of its diagonals is 5 cm, then find the h
length of the other diagonal.
(a) 8 cm (b) 5 cm (c) 10 cm (d) 6 cm r
1
Sol. (d) Area of rhombus = × d1 × d 2 Then,
2
1 1 2
15 = × 5 × x [other diagonal = x (say)] l Volume of a cone = πr h
2 3
∴ x = 6 cm l Total surface area = πr(r + l)
Ex. 5 If the area of trapezium, whose parallel sides are l Curved surface area = πrl
6 cm and 10 cm is 32 sq cm, then find the distance l Slant height ( l) = h 2 + r 2
between the parallel sides.
(a) 4 cm (b) 5 cm (c) 2 cm (d) 6 cm Cylinder
1
Sol. (a) Area of trapezium = ( a + b) × h A cylinder is a solid or hollow body that is
2
formed by keeping circles of equal radii one on
Where a and b are parallel sides and h is altitude (distance) another or by rolling a rectangular sheet.
1
32 = (6 + 10) × h ⇒ h = 4 cm Let r = radius of base and h = height. Then, h
2 l Volume of the cylinder = πr 2 h
Features of 3D-Mensuration l Total surface area = 2πr( h + r) r
Volume The amount of space occupied by the 3D-object is l Curved surface area = 2πrh
called its volume. It is measured in m 3 , cm 3 .
62 Self Study Guide SSC Constable (GD) Recruitment Exam

Sphere Ex. 7 Three cubes of iron whose edges are 6 cm, 8 cm


A sphere is like a solid ball in which any point on the
and 10 cm respectively are melted and formed into a
surface of sphere is equidistant from the centre of the
single cube. Find edge of the new cube.
sphere. Let r = radius. Then, (a) 12 cm (b) 14 cm (c) 10 cm (d) 15 cm
Sol. (a) Volume of the new cube = [63 + 83 + 10 3 ]
= 1728 cu cm
Let the edge of new cube be a cm.
r
Then, a3 = 1728 ⇒ a = 12 cm
∴ The edge of new cube = 12 cm
4
l Volume of a sphere = πr 3 Ex. 8 The volume of a right circular cone is 100 π cm 3
3 and its height is 12 cm. Find its slant height.
l Total surface area = 4 πr 2 (a) 18 cm (b) 15 cm (c) 13 cm (d) 20 cm
2 1
l Volume of a hemisphere = πr 3 Sol. (c) Volume of circular cone = πr 2h
3 3
1 2
l Total surface area of hemisphere = 3 πr 2 i.e. πr h = 100 π
3
l Curved surface area of hemisphere = 2 πr 2 ⇒ r 2 = 25 ⇒ r =5
∴ Slant height (l) = h 2 + r 2 = 12 2 + 5 2 ⇒ l = 13 cm
Ex. 6 A rectangular reservoir contains 42000 L of
water. If its length and breadth be 6 m and 3.5 m Ex. 9 In a shower, 10 cm of rain falls. What will be the
respectively, what is the depth of water in it? volume of water that falls on 1 hectare area of ground?
(a) 1 m (b) 6 m (c) 2 m (d) 4 m (a) 7508 m 3 (b) 1200 m 3
Sol. (c) Volume of reservoir = 42000 L = 42 m 3
(c) 1400 m 3 (d) 1000 m 3
Let the depth of water be x m.
7 Sol. (d) 1 hectare = 10000 m 2
Then, 6 × × x = 42
2 Volume of water = Area of base × Height
⇒ x=2m 10
= 10000 × m 3 = 1000 m 3
∴ Depth of water = 2 m 100
∴Volume of water = 1000 m 3

Practice Exercise
2D-Mensuration 4. The perimeter of a square is 16 cm, 7. If the side of an equilateral triangle
1. The perimeter of an equilateral then the area of the square is is increased by 30%, how much
triangle is 45 cm. Find its area. (a) 8 sq cm (b) 16 sq cm increase will take in its area?
(c) 32 sq cm (d) 64 sq cm (a) 60% (b) 90% (c) 45% (d) 69%
225 3 115
(a) cm 2 (b) cm 2
4 3 5. If the permeter of rectangle of a 8. The perimeter of a rhombus is 40
215 3 205 2 rectangle is 28 m. Whereas its area m and its height 5 m. Its area is
(c) cm 2 (d) cm 2
2 3 is 48 m 2. What is the length of its (a) 60 sq m (b) 50 sq m
diagonal? (c) 45 sq m (d) 55 sq m
2. The area of trapezium is 336 sq cm.
(a) 5 m (b) 10 m 9. The sides of a rectangular field are
If its parallel sides are in the ratio
(c) 12 m (d) 12.5 m in the ratio 3 : 4 and its area 7500
5 : 7 and the perpendicular distance
between them be 14 cm, then the 6. A rectangular lawn 80 m × 60 m sq m. The cost of fencing it at 25
smaller of the parallel sides is has two roads each 10m wide paise per m is
(a) 20 cm (b) 22 cm running in the middle of it , one (a) ` 87.50 (b) ` 77.50
(c) 24 cm (d) 26 cm parallel to the length and the other (c) ` 67.50 (d) ` 57.60
parallel to the breadth. Find the 10. The length of rectangle is 1 cm
3. A tile is size 9'' by 9'' . The number
cost of gravelling them at ` 20 per more than its width. Its perimeter
of tiles needed to cover a 12 ft wide
sq m? is 14 cm. The area of rectangle is
and 18 ft long floor will be
(a) ` 28000 (b) ` 30000 (a) 16 cm 2 (b) 14 cm 2
(a) 32 (b) 348 (c) 216 (d) 384
(c) ` 26000 (d) ` 25000 (c) 12 cm 2 (d) 10 cm 2
Mensuration 63

11. The perimeters of two squares are 21. The length of the longest pole that 12 π 8π
(a) cm3 (b) cm3
24 cm and 32 cm. The perimeter can be kept in a room 5 m long, 4 3 3
4 2
(in cm) of a third square equal in m broad and 3 m high is (c) π cm3 (d) π cm3
area to the sum of the areas of (a) 5 2 m (b) 6 2 m 3 3
these squares is (c) 7 2 m (d) None of these
(a) 45 (b) 40 (c) 32 (d) 48 Previous Years’ Questions
22. The surface area of a sphere is
12. The diagonal of a square is 4 2 cm. 616 sq m. What will be its 30. A solid right prism made of iron
diameter? has cross section of a triangle of
The diagonal of another square
(a) 15 cm (b) 14 cm sides 5 cm, 10 cm, 13 cm and of
whose area is double that of the
(c) 13 cm (d) 16 cm height 10 cm. If one cubic cm of
first square is
iron weighs 7 g, then the weight of
(a) 8 2 cm (b) 16 cm 23. A right circular cone has height of the prism is (approximately)
(c) 32 cm (d) 8 cm 12 cm and base diameter of 70 cm.
[SSC Constable, 2015]
13. The base of a triangle is 15 cm and Find the volume of the cone.
3 3 (a) 1570.8 gram (b) 1371.32 gram
height is 12 cm. The height of (a) 15800 cm (b) 15400 cm (c) 1100.68 gram (d) 1470.8 gram
another triangle of double the area (c) 16200 cm 3 (d) 16500 cm 3
having base 20 cm is 31. A right circular cone of height 20
24. A cylinder of height 21 cm has a cm and base radius 15 cm is
(a) 9 cm (b) 18 cm
base of radius 4 cm. Find the melted and casted into smaller
(c) 8 cm (d) 12.5 cm
volume of the cylinder. cones of equal sizes of height 5 cm
14. A hall 20 m long and 15 m broad is (a) 1048 cm 3 (b) 1050 cm 3 and base radius 1.5 cm. The
surrounded by a verandah of (c) 1120 cm 3 (d) 1056 cm 3 number of casted cones are
uniform width of 2.5 m. The cost of
flooring the verandah at ` 3.50 per 25. The surface area of a cube is 486 [SSC Constable, 2015]
sq m is cm 3. Find its volume. (a) 300 (b) 400 (c) 100 (d) 150
3 3
(a) ` 500 (b) ` 600 (c) ` 700 (d) ` 800 (a) 1331 cm (b) 1728 cm 32. If the edge of a cube is increased
(c) 729 cm 3 (d) 512 cm 3 by 100%, then the surface area of
15. The radius of a wheel is 1.75 m.
26. If the base radius and the height of the cube is increased by
How many revolutions will it make [SSC Constable, 2013]
in travelling 11 km? a right circular cone are increased
(a) 100% (b) 200%
(a) 10 (b) 100 by 20%, then the percentage (c) 300% (d) 400%
(c) 1000 (d) 10000 increase in volume is
approximately 33. The area of the shaded region in the
16. A wheel makes 10000 revolutions given figure is [SSC Constable, 2013]
(a) 60% (b) 68% (c) 73% (d) 78%
in covering a distance of 88 km.
The diameter of the wheel is 27. Height of a solid cylinder is 10 cm C
(a) 1.4 m (b) 2.4 m (c) 2.8 m (d) 2.2 m and diameter 8 cm. Two equal
conical holes have been made from
17. If the radius of circle is increased
its both ends. If the diameter of
by 50%, then what will be the 45° 45°
the hole is 6 cm and height 4 cm.
percentage increase in its area? A a B
The volume of remaining portion is O
(a) 125 (b) 100 (c) 75 (d) 50
(a) 24 π cm 3
(b) 36 π cm 3
(a) a 2( π − 1) sq units
18. The radius of the circle is 3 times (c) 72 π cm3 (d) 136 π cm3
a2
the radius of another circle, the (b) ( π − 2 ) sq units
circumference of the new circle will 28. X and Y are two cylinders of the 2
be how many times the same height. The base of X has (c) a (π − 2 ) sq units
2

circumference of the smaller circle? diameter that is half the diameter a2


(d) (π − 1) sq units
1 of the base of Y. If the height of X is 2
(a) 3 times (b) times
3 doubled, the volume of X becomes
34. A street of width 10 m surrounds
(c) 6 times (d) 9 times (a) double the volume of Y
from outside a rectangular garden
(b) equal to the volume of Y
whose measurement is
3D-Mensuration (c) greater than the volume of Y
(d) half the volume of Y 200 m × 180 m. The area of the
19. The area of the base of a rectangular path (in sq m) is
tank is 6500 cm 2 and the volume of 29. A hemisphere whose diameter is 4 [SSC Constable, 2012]
water contained in it is 2.6 cu m. The cm, made of wood. If a cone cut (a) 8000 (b) 7000 (c) 7500 (d) 8200
depth of water in the tank is out of this hemisphere whose base 35. The length of the three sides of a
(a) 2.5 m (b) 3 m (c) 5.5 m (d) 4 m is also 4 cm diameter. Find the right angled triangle are (x − 2) cm,
volume of wood in hemisphere. x cm and (x + 2) cm, respectively.
20. A wall 8 m long, 6 m high and 22.5
cm thick is made up of bricks, each Then, the value of x is
measuring 25 cm × 11.25 cm × 6 cm . [SSC Constable, 2012]
The number of bricks required is (a) 10 (b) 8
(c) 4 (d) 0
(a) 6400 (b) 45000
(c) 40000 (d) 25000 4 cm
64 Self Study Guide SSC Constable (GD) Recruitment Exam

36. The number of spherical bullets (a) 52 or 59 (b) 52 or 60 (a) 12 m (b) 17 m


that can be made out of a solid (c) 15 or 37 (d) 37 or 29 (c) 19 m (d) 21 m
cube of lead whose edge measures 38. A semicircular shaped window has
44 cm, each bullet being of 4 cm diameter of 63 cm. Its perimeter Answers
 22   22
diameter, is  take π =  equals to  π = 
 7  7
1 b 2 a 3 d 4 b 5 b
[SSC Constable, 2012] 6 c 7 d 8 b 9 a 10 c
[SSC Constable, 2011] 11 b 12 d 13 b 14 c 15 c
(a) 2541 (b) 2451
(a) 126 cm (b) 162 cm 16 c 17 a 18 a 19 d 20 a
(c) 2514 (d) 2415
(c) 198 cm (d) 251cm 21 a 22 b 23 b 24 d 25 c
37. The lengths of two sides of an
39. The length of longest pole that can 26 c 27 d 28 d 29 b 30 a
isosceles triangle are 15 and 22
be placed in a room of 12 m long, 8 31 b 32 c 33 b 34 a 35 b
respectively. What are the possible
m broad and 9 m high is 36 a 37 a 38 b 39 b
values of perimeter? [SSC Constable, 2011]
[SSC Constable, 2011]

Hints & Solutions


1. (b) Given that, perimeter = 45 cm  xy  13. (b) Area of the triangle
= x+ y+  %
⇒ 3a = 45 cm (Q a = side)  100  =
1
× 15 × 12 = 90 sq cm
⇒ a = 15 cm  900  2
=  30 + 30 +  % = 69% ∴ Area of another triangle = 180 sq m
3 2 3 225 3  100 
∴ Area = a = × 15 2 = cm 2 1
4 4 4 8. (b) The side of the rhombus Again, A = × b × h
1 2
2. (a) Area of trapezium = × Sum of parallel Perimeter 40
= = = 10 m 1
180 = × 20 × h
2 4 4 2
sides × Distance between them. ∴ Area of rhombus = 10 × 5 = 50 sq m ∴ h = 18 cm
1
336 = ( 5 x + 7 x ) × 14 9. (a) Let length = 3 x and breadth = 4 x 14. (c) Area of verandah
2
∴ 3 x × 4 x = 7500 = (25 × 20 − 20 × 15) m2 = 200 m2
⇒ x=4
⇒ x =
7500
= 625 ⇒ x = 25 m  7
∴ The smaller side = 20 cm
2
∴ Cost of flooring =  200 ×  = ` 700
12  2
3. (d) 1 ft = 12 "
∴The sides are 75 m and 100 m 15. (c) Circumference of the wheel = 2 πR
∴ Number of tiles ∴Perimeter of the field = 2( l + b ) = 350 m 22 175
12 × 12 × 18 × 12 =2 × × = 11 m
= = 384 Hence, cost of fencing 7 100
9×9 25
= 350 × = ` 87.50 Distance covered in 1 revolution = 11 m
4. (b) Perimeter of square = 16 cm 100 Total distance covered = 11000 m
16 10. (c) Let breadth = x, then length = ( x + 1) cm Number of revolution made =
11000
= 1000
Side of square = = 4 cm
4 ∴ 2( x + x + 1) = 14 11
∴ Area = ( 4 )2 = 16 sq cm ⇒ 2 x + 1 = 7 ⇒ x = 3 cm 16. (c) Distance covered in 1 revolution
88 × 1000
5. (b) 2 ( l + b ) = 28 ⇒ l + b = 14 ...(i) ∴ Length = 4 cm and breadth = 3 cm = = 8.8 m
10000
∴ Area = 12 cm2
lb = 48 ...(ii) Let the diameter of the wheel be d m.
( l − b )2 = ( l + b )2 − 4lb 24 πd = 8.8
11. (b) Side of 1st square = = 6 cm Then,
4 88 7
⇒ ( l − b )2 = 14 2 − 4 × 48 ⇒ d = × = 2 .8 m
∴ Area of 1st square = 36 cm 2
10 22
= 196 − 192 = 4 32
Side of 2nd square = = 8 cm 17. (a) Let the initial radius be r. Then, area
⇒ l− b=2 ...(iii)
4 = πr 2
On solving Eqs. (i) and (iii),
∴ Area of 2nd square = 64 cm2 150r 3r
diagonal = l2 + b2 = 8 2 + 6 2 = 10 New radius = =
By given condition, 100 2
diagonal = 10 m Area of 3rd square = ( 36 + 64) cm2 = 100 cm2 9r 2
10 m New area = π ×
6. (c) A B ∴Side of 3rd square = 100 cm = 10 cm 4
∴Perimeter of 3rd square = 40 cm  9 πr 2  5 πr 2
P Q Increase =  − πr 2  =
L N 12. (d) Diagonal of square  4  4
10 m
10 m

= 2 × Side = 2 × a Increase percentage


M R S O
4 2 = 2 ×a  5 πr 2 1 
∴ a = 4 cm = × 2 × 100 % = 125%
C 10 m D  4 πr 
∴ Area of 1st square = 16 cm2
Area to be gravelled = p( l + b − p)
18. (a) Let the radius of smaller circle be r.
By given condition,
Then, its circumference = 2 πr
= 10 ( 80 + 60 − 10) = 10 × 130 = 1300 sq m Area of another square = 2 × 16 = 32 cm2
New radius = 3r
∴ Cost of gravelling = ` 1300 × 20 = ` 26000 ∴Diagonal of another square New circumference = 2 π( 3r ) = 3 × (2 πr )
7. (d) Net increase in area = 2 × 32 cm = 8 cm = 3 × (Smaller circumference)
Mensuration 65

19. (d) l × b = 6500 cm2 Volume of one cone =


1
× π × 3 2 × 4 = 12 π cm3   100 + 100  2 
3 =   − 1 × 100%
∴ l × b × h = (2.6 × 100 × 100 × 100) cm   100 
3

∴ Volume of both cones = 24 π cm3
 2.6 × 100 × 100 × 100    200  2 
∴ h=   cm ∴ Volume of remaining portion
 6500  =   − 1 × 100%
= 160 π − 24 π = 136 π cm3   100  
 2.6 × 100 × 100 × 100 
=  m
 6500 × 100  28. (d) Volume of X = πr 2h = [(2 ) − 1] × 100% = ( 4 − 1) ×100% = 300%
2

h=4m Volume of Y = π(2 r )2 h = 4 πr 2h 33. (b) C


20. (a) Volume of wall If the height of X is doubled, then new volume
 45  = πr 2(2 h) = 2 πr 2h
=  800 × 600 ×  cm
3
a
 2  1 1
= ( 4 πr 2h) = volume of Y
 1125  45° 45°
Volume of 1 brick =  25 × × 6 cm2 2 2
 100  29. (b) Volume of wood in hemisphere
A a O B
∴Number of bricks 2 1 π 8π 2a
= π (2 )3 − π (2 )2 × 2 = (16 − 8) = cm3
800 × 600 × 45 × 100 3 3 3 3
= = 6400 The required area = Area of the semicircle
2 × 25 × 1125 × 6 30. (a) In a solid right prism − Area of ∆ABC
21. (a) Required length= Length of diagonal sides of triangle are 5 cm, 10 cm and 13 cm. πr 2 1
Height of prism = 10 cm = − × Base × Height
= 52 + 42 + 32 = 50 = 5 2 m 2 2
Volume of prism = Area of base × height π × ( a )2 1
22. (b) Surface area of the sphere = 616 sq m We know that = − × ( a + a) × a
2 2
Area of triangle = s( s − a )( s − b )( s − c )
⇒ 4 πr 2 = 616 πa 2
1 a2
a+ b+c = − × 2 a2 = ( π − 2 ) sq units
616 7 where, s= 2 2 2
⇒r =2
× ⇒ r 2 = 49 ⇒ r = 49 = 7 cm 2
4 22 34. (a) 10
here, a = 5 cm, b = 10 cm and c = 13 cm
∴ Diameter =2 r = 2 × 7 = 14 cm a + b + c 5 + 10 + 13 28 200m
23. (b)Given, height of the cone, h = 12cm s= = = = 14
2 2 2

180m
and diameter of base = 70 cm 10 10
So, area of triangle = s( s − a )( s − b )( s − c )
70
Radius of base ( r ) = = 35 cm = 14(14 − 5)(14 − 10)(14 − 13)
2
1 = 14 × 9 × 4 × 1
∴ Volume of the cone = πr 2h = 2 ×7 × 3× 3×2 ×2
3 10
1 22 = 3 × 2 2 × 7 = 6 14
= × × ( 35)2 × 12 = 15400 cm 3 Volume of prism = Area of base × Height ∴The area of the path
3 7
= 6 14 × 10 cm 3 = 60 14 cm 3 = (200 + 10 × 2 ) × (180 + 10 × 2 )
24. (d) Volume of the cylinder = π r 2h
According to the question, 1 cm 3 ⇒ 7 g − 200 × 180
22
= × 4 × 4 × 21 = 1056 cm 3 Weight of prism = 60 × 7 × 14 = 60 × 7 × 374 . = 220 × 200 − 200 × 180
7 = 44000 − 36000 = 8000 m2
486 = 1570.8 g
25. (c) 6a 2 = 486 ⇒ a 2 = = 81 cm 2
6 31. (b) Right circular cone's height = 20 cm 35. (b) In right angled triangle
⇒ a = 81 = 9 cm and base radius = 15 cm (Hypotenuse)2 = (Base)2
∴Volume of the cube = 9 3 = 9 × 9 × 9 = 729 cm 3 1 + (Perpendicular)2
Volume of big cone = × base area × height
26. (c) Let original radius of base and height 3 ⇒ ( x + 2 ) = x + ( x − 2 )2
2 2
1 1 22
are R and H, respectively. = × π × r × h= ×
2
× 15 × 15 × 20 ⇒ x + 4 x + 4 = x2 + x2 − 4 x + 4
2

Original height = H 3 3 7
The big is melted and casted into smaller ⇒ x2 = 8 x ⇒ x = 8
 120  6
New radius =  R = R cones of equal sizes of height 5 cm and base
 100  5 36. (a) Total number of spherical bullets
radius 1.5 cm. Volume of solid cube
and new height = H
6 1 =
5 Volume of smaller cone = × π × 15 . × 15. ×5 Volume of 1 bullet (spherical)
3 44 × 44 × 44
1 = = 2541
Original volume V1 = π R 2H Number of casted =
Volume of big cone
4 22
3 Volume of smaller cone × ×2 ×2 ×2
1 6 
2
6 216 1 3 7
New volume V2 = π  R  × H = V1 × π × 15 × 15 × 20 37. (a) Perimeter of isosceles triangle
3 5  5 125 = 3
1 = 15 + 15 + 22 or 15 + 22 + 22
Now, increase in volume × π × 15. × 15. ×5
216 91 3 = 52 or 59 units
= V1 − V1 = V1 1 38. (b) Perimeter of semicircular shaped
125 125 × π × 15 × 15 × 20 × 10 × 10
window
∴Percentage increase in volume = 3
91 1
1
× π × 15 × 15 × 5 = ( πr + 2 r ) cm = r( π + 2 ) cm
= V1 × × 100% = 72 .8% 3 63  22  63 36
125 V1 =  + 2  cm = × = 162cm
20 × 10 × 10 2  7 
= = 20 × 10 × 2 = 400 2 7
= 73% (approx.) 5 39. (b) Length of the longest pole
27. (d) Volume of cylinder = π( 4)2 × 10 32. (c) The required surface area increased by
= 12 2 + 8 2 + 9 2
= 160 π cm3   100 + x  2 
=   − 1 × 100% = 144 + 64 + 81= 289 = 17 m
  100  
Self Study Guide SSC Constable (GD) Recruitment Exam

CHAPTER 14

Data Interpretation
Directions (Ex. Nos. 3-5) The pie chart given here shows
Data Interpretation expenditures incurred by a family on various items and their savings,
which amount to ` 8000 in a month. Study the chart and answer the
It is nothing but drawing conclusions and inference from a questions based on the pie chart.
comprehensive data presented numerically in tabular form
or pictorial form by means of an illustration graphs, pie
charts etc. Thus, the act of organising and interpreting data Food
to get meaningful information is data interpretation. 120°
Others
45°
Data Table Education
It is the easiest and most accurate way of presenting data Housing 30°
in a non-graphical manner in which various rows and 105° Savings
columns are used to represent the data. 60°

Directions (Ex. Nos. 1 and 2) Number of toys of five types


(A to E) manufactured over the years (in thousands) is given
below. Study the table and answer the following question. Ex. 3 How much expenditure is incurred on
A B C D E
education
(a) ` 3000 (b) ` 5000 (c) ` 4000 (d) ` 7000
2002 200 150 78 90 68
8000
2003 150 180 100 105 70 Sol. (c) Total income = × 360 = ` 48000
60
2004 180 175 92 110 85 30
2005 195 160 120 125 75 Expenditure on education = × 48000 = ` 4000
360
2006 220 185 130 135 80
Ex. 4 The ratio of the expenditure on food to the
Ex. 1 The approximate percentage increase in savings is
production of D type of toys from 2003 to 2005 was (a) 3 : 2 (b) 2 : 1 (c) 4 : 3 (d) 3 : 4
(a) 5% (b) 19% (c) 29% (d) 25% 120
125 − 105 Sol. (b) Expenditure on food = × 48000 = ` 16000
Sol. (b) Percentage increase = × 100 360
105 16000 2
20 ∴ Ratio = = = 2 :1
= × 100 = 19% 8000 1
105
Ex. 5 What is the total expenditure of the family for
Ex. 2 The percentage drop in production of A type of
the month?
toys from 2002 to 2004 was.
(a) ` 40000 (b) ` 5000
(a) 10% (b) 20% (c) 25% (d) 30%
200 − 180
(c) ` 4000 (d) ` 7000
Sol. (a) Percentage decrease = × 200
200 Sol. (a) Total expenditure = Income − Savings = 48000 − 8000
20 = ` 40000
= × 100 = 10%
200
Bar Chart
Pie Chart A bar chart is a chart with rectangular bars with lengths
Pie chart is called so because of its shape. Each slice of pie proportional to the values that they represent.
is allowed to each category and shows the portion of the A bar is a thick line whose width is shown merely for
entire pie. The total quantity is distributed over a total attention. In this method, the data is plotted on the x and
angle of 360°. Here, the data can be plotted with respect to Y-axes as bars. One of the axis (normally the X-axis)
only one parameter. Hence, uses of pie charts are restricted represents a discrete variable while the other axis
to represent limited type of information. represents the scale for one or more continuous variable.
Data Interpretation 67

There are three main bar charts Ex. 7 The average production of 2004 and 2005 was
1. A simple bar chart relates to only one variable. exactly equal to the average production of which of the
2. A sub-divided bar chart is used to represent various parts following pairs of years ?
of total magnitude of a given variable. (a) 2006,2007 (b) 2005, 2006
3. In multiple bar chart, two or more bars are constructed (c) 2002, 2006 (d) 2001, 2005
adjoining each other to represent either different Sol. (c) From the graph. Required average production
25 + 55
components of a total or to show multiple variables. = 40000 tonnes
2
Directions ( Ex. Nos 6 and 7) Study the following graph and ∴2002 and 2006 will have same average production as of
answer the questions given below it. years 2004 and 2005.
120
110 Line Graph
Production (in 1000 tonnes)

100 A line graph shows the quantitative information or a


90
90 relationship between two changing quantities (variables)
80
80 with a line or curve that connects a series of successive data
70
70 points. It is also known as Cartesian graph.
60 55
50
50 Ex. 8 The adjoining diagram is frequency polygon for
40
40
30 the scores of students in a test. What is the total number
30 25 of students appeared in the test.
20
(a) 250 (b) 175 (c) 180 (d) 200
10
40
0
2001
2002
2003

2005

2008
2004

2006
2007

35

Number of Students
30
Years
25

Ex. 6 What was the percentage decline in the 20


production of salt from 2003 to 2004? 15
(a) 64.3 (b) 180 10
(c) 62.4 (d) 107
5
Sol. (a) Percentage decrease
0
25 − 70 4500
= × 100 = −
400 450 500 550 600 650 700 750 800
70 70 Years
= − 64.3% Sol. (b) From the figure,
− ve sign indicates decrement Total number of students = 15 + (30 × 2) + 35 + 25 + (20 × 2)
= 15 + 60 + 35 + 25 + 40 = 175
∴ 64.3% decline.

Practice Exercise
Directions (Q. Nos. 1-5) Study the table and answer the questions 1. In which year, the production of scooters of all factories
that follow. was equal to the yearly average number of scooters
produced during 1985-89?
Yearly Production (in thousand) (a) 1985 (b) 1986 (c) 1987 (d) 1988
of Scooters in Different Factories
2. Which factory/factories showed a decrease of 25% in the
Year production of scooters in 1989 as compared to 1988?
1985 1986 1987 1988 1989 (a) P (b) S (c) Q and R (d) P and T
Factory
3. The ratio of the production of scooters by Factory P to
P 20 15 24 13 17
that by Factory T in 1985 is
Q 16 23 41 20 15 (a) 2 : 3 (b) 1 : 2 (c) 3 : 2 (d) 2 : 1
R 14 21 30 16 12
4. In which year was the total production of scooters
S 25 17 15 12 22 maximum?
T 40 32 39 41 35 (a) 1989 (b) 1986
Total 115 108 149 102 101 (c) 1987 (d) 1985
68 Self Study Guide SSC Constable (GD) Recruitment Exam

5. In which year was the total production of scooters of all Directions (Q. Nos. 11-15) Study the following graph carefully to
factories 20% of the total production of scooters during answer the questions that follow.
1985-1989? Number of Students Passed (in thousand)
(a) 1988 (b) 1985 from Two Universities over the Years
(c) 1986 (d) 1989
University A University B

Number of Students (in thousand)


Directions (Q. Nos. 6-10) Study the following pie chart carefully 70
to answer the questions that follow.
60
Percentagewise Distribution of Teachers
who Teach Six Different Subjects 50

40
i 8%

Biology
12% 30
Hind

20
English
Chemistry 27% 10
23%
M

0
at 13

2003 2004 2005 2006 2007 2008


he %
m

Physics Years
at
ics

17%
11. What is the respective ratio of the number of students
passed from University A in the year 2007 and the number
of students passed from University B in the year 2004?
Total number of teachers = 1800 (a) 5 : 4 (b) 4 : 5 (c) 7 : 10 (d) 10 : 7
6. If two-ninth of the teachers who teach Physics, are
12. What is the difference between the total number of
female, then number of male Physics teachers is
students passed from both the universities in the year
approximately what per cent of the total number of
2007 together and the total number of students passed in
teachers who teach Chemistry?
year 2005 from both the universities together?
(a) 57 (b) 42
(a) 70000 (b) 37000 (c) 7000 (d) 3700
(c) 63 (d) 69
7. What is the total number of teachers teaching Chemistry, 13. What is the sum of students passed from
English and Biology? University B in the years 2003, 2005 and 2006 together?
(a) 80000 (b) 8000 (c) 800000 (d) 75000
(a) 1226 (b) 1116
(c) 1176 (d) 998 14. Number of students passed from University B in 2008 is
8. What is the difference between the total number of approximately what per cent of the total number of
teachers who teach English and Physics together and the students passed from University A over the years?
total number of teachers who teach Mathematics and (a) 30 (b) 25 (c) 20 (d) 35
Biology together? 15. What is the respective ratio of the number of students
(a) 352 passed in 2007, 2008 and 2005 from university A?
(b) 342 (a) 5 : 3 : 2 (b) 3 : 5 : 5 (c) 5 : 3 : 3 (d) 5 : 1 : 1
(c) 643
(d) 653 Directions (Q. Nos. 16-20) Study the following graph carefully and
answer the following questions that follow.
9. What is the respective ratio of the number of teachers
who teach Mathematics and the number of teachers who Imports and Exports (in million dollar) of a Country
teach Hindi? from the year 1995 to 2001.
(a) 13 : 8 y x Import Export
(b) 7 : 13
190
(c) 7 : 26 x
(d) 8 : 15 170
x
150
10. If the percentage of Mathematics teachers is increased by
50% and percentage of Hindi teachers is decreased by 130
x x
25%, what will be the total number of Mathematics and 110 x
Hindi teachers together? x
90 x
(a) 390
70
(b) 379
(c) 459 50
(d) 480 30
x
1995 1996 1997 1998 1999 2000 2001
Data Interpretation 69

16. What was the percentage increase in imports in the year Directions (Q.Nos. 25-28) The following bar diagram represents the
when percentage increase in exports over its preceding use of different modes of travel to school by students in a certain locality of
year is maximum? the town. Study the graph and answer the questions. [SSC Const., 2011]
(a) 21% (b) 13% (c) 9% (d) None of these y
24

Number of students (1 unit = 10 students)


17. Which of the following years witnessed maximum fall in
exports? 22
(a) 1998 (b) 1999 (c) 2000 (d) 2001 20

18. During which year, there was maximum increase in 18


imports over its preceding year? 16
(a) 1997 (b) 1998 (c) 1999 (d) 2000 14
19. What was the maximum percentage fall in imports over 12
the given period? 10
(a) 30% (b) 14% (c) 24% (d) None of these 8
20. If total trade is defined as the addition of imports and 6
exports, what was the ratio of exports to imports in the 4
year when total trade was maximum?
2
(a) 9 : 10 (b) 10 : 9 (c) 9 : 4 (d) 4 : 9
0 x
Car Bus Moped Bicycle Rickshaw
Previous Year‘s Questons
Model of travel
Directions (Q.Nos. 21 and 22) Study the data and answer the 25. How many students are coming from that locality?
questions. [SSC Constable, 2015] (a) 500 (b) 600 (c) 560 (d) 660
The Score of Students of a Class 26. How many students use Bicycle and Rickshaw combined?
IQ Score No. of Students (a) 240 (b) 340 (c) 140 (d) 440
80-90 6 27. What is the percentage of students using Bus from that
locality?
90-100 9 14 2 8
(a) 22 (b) 18 (c) 22 (d) 22
100-110 16 33 3 11
110-120 13 28. What is the ratio of the students using their means of
120-130 4 transport as Car with those using Rickshaw?
130-140 2 (a) 7 : 2 (b) 8 : 3 (c) 2 : 7 (d) 3 : 8

21. Number of students whose IQ score is 140 is Directions (Q.Nos. 29-32) The following diagram shows the
(a) 0 (b) 1 percentage of population of Hindus, Sikhs and Muslims with respect to
(c) undeterminable from given data (d) 2 total population in a town during 2005 to 2008. Study the diagrams
and answer the question. [SSC Constable, 2012]
22. The number of students whose IQ score is 100 and more is 75
Hindus Sikhs Muslims
(a) 36 (b) 46 (c) 35 (d) 29
70
65
Directions (Q. Nos. 23 and 24) The following graph shows the 65
number of people ownig two wheelers in 7 different towns. Study the 60
60
Percentage of total population

graph and answer the questions. [SSC Constable, 2013] 55


55

700 50
Numbers of People Owning

650 45
45
600
550 40
Two Wheelers

500 35
450 35
400 30
30
350 25 25
300 25
250 20
200 20
150 15 15 15
100 10
50 10
5
0 A B C D E F G 0
Towns HSM HSM HSM HSM
2005 2006 2007 2008
23. The town which has twice as many vehicles as Town E is Years
(a) Town A (b) Town C (c) Town B (d) Town D 29. If the total population in 2007 was 80 lakh, then the number
24. The number of towns that have at the most 400 vehicles is of Hindus in 2007 was (in lakh)
(a) 4 (b) 3 (c) 2 (d) 1 (a) 25 (b) 16 (c) 18 (d) 20
70 Self Study Guide SSC Constable (GD) Recruitment Exam

30. Percentage decrease in Hindu population form 2005 to 2008 is 33. The number of workers whose daily payment is ` 20 is
(a) 50 (b) 40 (c) 25 (d) 15 (a) 9
(b) 16
31. Difference of percentage of population of Hindus in 2005
(c) 20
and 2008 is
(d) 4
(a) 20 (b) 15 (c) 25 (d) 30
34. The total daily payment made to the group which
32. If the total number of Hindus in 2008 was 12 lakh, the contains 9 workers is
number of Muslims in 2008 was (in lakh)
(a) ` 400
(a) 18 (b) 12 (c) 24 (d) 16
(b) ` 315
Directions (Q. Nos. 33 and 34) Given here is a graph showing (c) ` 480
the number of workers with their daily payment by a workshop. Study (d) ` 135
the graph and answer the questions based on this graph.
[SSC Constable, 2011]
Answers
Daily payment to each worker (in `)

45
1 a 2 c 3 b 4 c 5 b
40
6 a 7 b 8 b 9 a 10 c
35
11 d 12 a 13 a 14 b 15 c
30
16 d 17 d 18 c 19 d 20 d
25
21 c 22 c 23 a 24 b 25 d
20
26 b 27 c 28 d 29 d 30 a
15
31 b 32 d 33 c 34 b
10
5

2 3 4 9 16 20 26
Number of workers

Hints and Solutions


1. (a) Average number of scooters produced during 1985-1989 8. (b) Required difference
115 + 108 + 149 + 102 + 101 575 (27 + 17 ) (13 + 12 )
= = = 115 = 1800 × −1800 ×
5 5 100 100
= production in year 1985 = 792 − 450 = 342
20 − 15 13
2. (c) Q = × 100 = 25% and 1800 ×
20 9. (a) Required ratio = 100 = 13 : 8
16 − 12 8
R= × 100 1800 ×
16 100
= 25% also. 10. (c) Required number
3. (b) Required ratio = 20: 40 = 1:2  13 150   8 75 
=  1800 × ×  +  1800 × × 
 100 100   100 100 
4. (c) The total production of scooters is maximum in year 1987.
5. (b) Total number of scooters = 575 = 351 + 108 = 459
20 50000
∴ Required number of scooters = × 575 = 115 11. (d) Required ratio = = 10 :7
100 35000
Number of scooters produced in 1985 = 115. 12. (a) Required difference
6. (a) Number of teachers in Physics = 1800 ×
17
= 306 = ( 50000 + 60000) −( 30000 + 10000)
100 = 110000 − 40000 = 70000
2 13. (a) Required sum
Female teachers in Physics = 306 × = 68
9 = ( 30000 + 10000 + 40000) = 80000
Male teachers in Physics = 306 − 68 = 238 14. (b) Required percentage
23 50000
Number of teachers in Chemistry = 1800 × = 414 = ×100%
100 20000 + 25000 + 30000
238
∴Required percentage = × 100 ≈ 57% +50000 + 50000 + 30000
414
50000
7. (b) Total number of teachers teaching Chemistry, English and Biology = × 100% ≈ 25%
205000
(23 + 27 + 12 )
= 1800 × = 1116 15. (c) Required ratio = 50: 30: 30 = 5: 3: 3
100
Data Interpretation 71

16. (d) In the year 1997, the increase in exports over the preceding year 25. (d) Number of students coming from the locality
was maximum. = 6 + 15 + 11 + 18 + 16
So, in the year 1997, the increase in imports = 66 units
110 − 90 20
× 100% = × 100% = 22 .2% = 66 × 10 = 660
90 90
26. (b) Number of students use bicycle and rickshaw
17. (d) Year 2001, = 18 × 10 + 16 × 10
80 − 60
i.e. × 100% = 25% = 180 + 160 = 340
80 15 8
18. (c) In the year 1997 = 110 − 90 = 20 million dollar 27. (c) Required percentage = × 100 = 22 %
66 11
In the year 1998 = 120 − 110 = 10 million dollar 28. (d) Required ratio = 6 : 16 = 3 : 8
In the year 1999 = 160 − 120 = 40 million dollar
29. (d) Total number of Hindus in 2007
In the year 2000 = 180 − 160 = 20 million dollar 25
= 8000000 ×
19. (d) In the year 2001, the percentage is maximum and is equal to 100
180 − 120
× 100% = 33. 33% = 2000000
180
30. (a) Required decrease percentage
20. (d) Total trade is maximum in the year 2000. 30 − 15
80 = × 100 = 50%
∴ Required ratio = = 4: 9 30
180
31. (b) Required difference = (30 – 15) % = 15%
21. (c) Number of students whose IQ score is 140 is undetermined as the
32. (d) According to the question,
exact figure is not given in the question.
15% = 1200000
22. (c) Number of students whose IQ score is 100 and more than 100 is
1200000
= 16 + 13 + 4 + 2 = 20 + 15 = 35 ∴ 20% = × 20
15
23. (a) Vehicles of E = 300
= 1600000
Twice of vehicles E = 2 × 200 = 600
33. (c) It is obvious from the graph.
So, the Town A has twice as many vehicles as Town E.
34. (b) Required total daily payment = 35 × 9
24. (b) The number of towns that have not more than or at the most 400 = ` 315
are 3 i.e. C, E, F.

You might also like